100% found this document useful (1 vote)
3K views316 pages

Tests 6 - 10 PDF

This document provides a practice listening comprehension test with 3 parts (A, B, C) and a total of 50 questions. Part A contains short conversations followed by a question. Part B contains longer conversations also followed by questions. Part C contains several talks also followed by questions. The test-taker is instructed to choose the best answer for each question based on what was stated or implied in the conversation or talk. They are not allowed to take notes during the test.
Copyright
© © All Rights Reserved
We take content rights seriously. If you suspect this is your content, claim it here.
Available Formats
Download as PDF, TXT or read online on Scribd
100% found this document useful (1 vote)
3K views316 pages

Tests 6 - 10 PDF

This document provides a practice listening comprehension test with 3 parts (A, B, C) and a total of 50 questions. Part A contains short conversations followed by a question. Part B contains longer conversations also followed by questions. Part C contains several talks also followed by questions. The test-taker is instructed to choose the best answer for each question based on what was stated or implied in the conversation or talk. They are not allowed to take notes during the test.
Copyright
© © All Rights Reserved
We take content rights seriously. If you suspect this is your content, claim it here.
Available Formats
Download as PDF, TXT or read online on Scribd
You are on page 1/ 316

1 /PRACTICE 1'EsT 1/ 1 1

PRACTICE TEST 1

SECTION 1
LISTENING COMPREHENSION
Time: Approximately 30 Minutes
50 Questions

Section 1 has three parts. Each part has its own set of directions.
Do not take notes while listening or make any marks on the test
pages. Notetaking, underlining, or crossing out will be considered
cheating on the actual TOEFL exam. Answer the questions
following the conversations or talks based on what the speakers
have stated or implied.
For Practice Test 1, insert your Listening Comprehension cas-
sette in your tape player. On the actual TOEFL, you will be given
extra ~ime to go on to the next page when you finish a page in the
Listening Comprehension section. Jn the following test, however,
you will have only the 12 seconds given after each question. Tum
the page as soon as you have marked your answer. Start your
cassette now.

Part A

DIRECTIONS

Jn Part A, you will hear short conversations between two


speakers. At the end of each conversation, a third voice will ask a
question about what was said. The question will be spoken just one
time. After you bear a conversation and the question about it, read
the four possible answers and decide which one would be the best
answer to the question you have beard. Then, on your answer sheet,
find the number of the problem and mark your answer.

315
1 1 1 1 1

1. (A) She's tired of teaching.


(B) She was dismissed from her job.
(C) She's changing jobs.
(D) The school is too hot.

2. (A) She got up later than usual.


(B) The bus was late.
( C) She forgot her class.
(D) Her clock was wrong.

3. (A) The weather report.


(B) The traffic report.
(C) Directions to Interstate 4.
(D) Their disgust with careless drivers.

4. (A) She thinks his lectures are boring.


(B) She thinks his tests are too long.
(C) She doesn't like his choice of test questions.
(D) She doesn't think he prepares well enough.

5. (A) A movie. (C) A soccer game.


(B) A documentary. (D) A comedy.

6. (A) America. (C) Switzerland.


(B) England. (D) Sweden.

7. (A) She haci not applied to Stetson.


(B) She had not worked very hard.
(C) She was certain to be admitted.
(D) She was not likely to be 2.dmitted.

8. (A) She got scratched in·the wild berry bushes.


( B) She got cut at the wild picnic celebration.
(C) She was allergic to the fruit that she had eaten.
(D) She was trying to get a suntan at the picnic.

316
1 1 1 l 1 /PRACTICETEsT1/ 1 1 1 1 l

9. (A) Wiwtner. (B) Wittner. (C) Wittmer. (D) litner.

10. (A) Go out of town.


(B) Help the woman prepare for her meeting.
(C) Work with the woman.
(0) Work when the woman was supposed to work.

11. (A) 5 (B) 3 (C) 2 (0) 8

12. (A) She went to the wrong class.


(B) She was late for class because she got lost.
(C) She missed the class.
(D) She had some trouble finding the class, but she arrived on
time.

13. (A) The man is not sure which type of flowers April sent.
(B) April received many kinds of flowers.
( C) The man received many kinds of flowers from April.
(0) The man appreciated April's sending him flowers.

14. (A) William slept all the way from Georgia to New York.
(B) George didn't sleep at all on the trip.
(C) William was half asleep all the time that he was driving.
(0) WiJJiam didn't sleep at all on the trip.

15. (A) Too many people came to the meeting.


(B) There were not enough people at the meeting to inspect
the documents.
( C) The man had expected more" people to come to the
meeting.
(0) There were not enough seats for all the people.

16. (A) He sold no magazines.


(B) He sold only one magazine.
( C) He has never sold as many magazines as he sold today.
(0) He sold five magazines at one house.

317
1 1 1

17. (A) Frank told the contractor to do the work in spite of the
cost.
(B) Frank told the contractor that the price was too high.
( C) Frank cannot afford the work on his house.
(D) Frank repaired his own house.

18. (A) He studied last night because he had to.


(B) He tried to study last night, but the material was too ha rd.
(C) He couldn't study last night because he was very tired.
(D) He studied last night because he was bored.

19. (A) She goes to a movie every year.


(B) She hasn't gone to a movie yet this year, but last year she
did.
(C) She doesn't go to a movie unless she has the time.
(D) She hasn't seen a movie for a long time.

20. (A) He turned around to answer the teacher's question.


(B) He is an intelligent student.
(C) .He must have been embarrassed.
(D) He looked in the red book for the answer to the question.

21. (A) On a train. (C) On a plane.


(B) On a boat. (D) On a bus.

22. (A) She is going on vacation.


(B) She is leaving her job temporarily for health reasons.
(C) During the summer, she often misses work because of
illness.
(D) She is sick of working all the time.

23. (A) She is disappointed with the results.


(B) She likes her job very much.
( C) She is hoping for some improvem~nts in her workplace.
(D) She is very pleased with the outcome of her meeting.

318
1 l 1 /PRAcnCE'lEST.1/ 1 l 1 { 1 ·I
24. (A) He said be was sorry that he bad not announced the test
sooner.
(B) He was sorry that he had forgotten to bring the tests to
class. '
(C) He was sorry that he hadn't given the test earlier.
(D) He said he was sorry that he had not given the results of
the test sooner.

25. (A) She is taking a leave of absence from her job because of her
health.
(B) She is not going to return to her job.
(C) She is right to quit her job.
(D) She did very good work, but now she is quitting her job.

26. (A) John will be able to buy groceries.


(B) John doesn't have enough money to buy groceries.
(C) Joi:n wouldn't buy groceries even if he had enough money.
(D) John can't find his grocery money.

27. (A) Eighty people came to the rally.


(B) Forty people came to the rally.
(C) One hundred sixty people camP. to the rally.
(D) One hundred people came to the rally.

28. (A) They are going to meet Fred and Mary at the movies if they
have time.
(B) They went to the movies with Fred and Mary, but the
theater was closed.
(C) They couldn't meet Fred and Mary at the movies because
they didn' t have anj money.
(D) Fred and Mary were supposed to meet them at the movies,
but their car broke down.

319
1 1 t' 1 1 /PRACTICE TEsT 1/ 1 1 1 1 1

29. (A) He was supposed to give the awards at the banquet, but he
didn 't.
(B) He was given an award, but he refused it.
(C) He didn' t go to the banquet.
(D) He went to the awards banquet, but he refused to give a
speech.

30. {A) He is out of sugar.


(B) He puts only sugar in his coffee.
{C) There isn't enough sugar in his cc.tree.
(D) He likes sugar, but the coffee he is drinking has \VO much.

Go ON TO PART B

PartB

DIRECTIONS

In Part B, you will hear longer conversations. After each


conversation, you will be asked some questions. The conversations
and questions will be spoken just one time. They will not be written
out for you, so you wi:l have to listen carefully in order to
understand and remember what the speaker says.
When you hear a question, read the four possible answers in your
test book and decide which one would be the best answer to the
question you have heard. Then, on your answer sheet, find the
number of the problem and fill in the space that corresponds to the
letter of the answer you have chosen.

31. {A) In a clothing store.


(B) In customs.
(C) At a bank.
(D ) In a liquor store.

32. (A} 1 (B) 2 (C) 3 (D) 4

320
... 1
J. ..... 1 l 1 /PRACTICE TEsT 1/

33. (A) To make a list of her purchases.


( B) To open her suitcase.
(C) To pay $300 duty.
(D) To show him the bottles of wine.

34. (A) Plants. (B) Wine. (C) Meat. (D) Cash.

35. (A.) The Incredible Adventures of the Martians.


(B) Mission to Mars.
(C) Martian Renaissance.
(D) Captivating Tales of Mars.

36. (A) Science fiction. (C) Biography.


(B) Adventure. (D) Documentary.

37. (A) One year. (C) Five years.


(B) Three years. (D) Seven years.

38. (A) The crew had some incredible adventures on Mars.


(B) The crew met some real Martians.
(C) The Martians captured the crew.
(D) The ship carried an aJl-male crew.

Go ON10 PART C

PartC

DIRECTIONS

In Part C, you will hear several talks. After each talk, you will be
asked some questions. The talks and questions wiJI be spoken just
one time. They will not be written out for you, so you will have to
listen carefully in order to understand and remember what the
speaker says.

321
1 1 I 1 1 /PRACTICE TEsT 1/ 1 1 l 1 1

When you hear a question, read the four possible answers in your
test book and decide which one would be the best answer to the
question you have heard. Then, on your answer sheet, find the
number of the problem and fill in the space that corresponds to the
letter of the answer you have chosen.

39. (A) Spain. ( C) Florida.


(B) Latin America. (D) America.

40. (A) Soccer. (C) Football.


(B) Handball. (D) Horse racing.

41. (A) Jai aJai is one of the fastest-moving games.


(B) Jai aJai requires a great deal of skill and endurance.
(C) Jai alai can be played as singles or doubles.
(D) It is illegal to bet on Florida jai alai games.

42. (A) Baseball. ( C) Handball.


(B) Ping-Pong. (D) Badminton.

43. (A) Multiple telegraph. (C) Aviation.


(B) Telephone. (D) Acoustics.

44. (A) Acoustical science. (C) Adventure.


(B) Aviation. (D) Architecture.

45. (A) He worked very hard, but never achieved success.


(B) He spent so many years working in aviation because he
wanted to be a pilot.
(C) He dedicated his life to science and the well-being of
humankind.
(D) He worked with the deaf so that he could invent the
telephone.

322
1 1

46. (A) Bell was born in the eighteenth century.


(B) Bell worked with the deaf.
(C) Bell experimented with the science of acoustics.
(D) Bell invented a multiple telegraph.

47. (A) 100 (B) 25 (C) 35 (0) 50

48. (A) It is less expensive than term insurance.


(B) It can have a fixed premium for lite.
(C) It may result in the insured's being able to discontinue
premium payments.
(0) It may result in retirement income.

49. (A) Because the insured must pay for the agent's retirement
fund.
(B) Because it is pure insurance.
( C) Because part of the money is invested.
(D) Because it is based on the age of the insured.
',

50. (A) It is inexpensive.


(B) One can borrow from the fund that is built up.
( C) The premium is paid by the interest.
(D) It requires an easier medical examination than does cash
value insurance.

STOP. TuJS IS TIIE END OF TIIE LISTENING COMPREHENSION SEGnON.


Go ON TO SECTION 2.

323
2 2 2 2 2 / PRAcnCE TEST 1/ 2 2 2 2 2

SECTION 2
STRUCTURE AND WRITTEN EXPRESSION
Time: 25 Minutes

r
40 Questions

Part A

DIRECTIONS

Questions 1-15 are incomplete sentences. Beneath each sentence


you will see four words o r phrases, marked {A), (B), (C), anrl (D).
Choose rhe one word or phrase that best completes the sentence.
Then, on your answer sheer, find the number of the question and fill
in the space that corresponds to the letter of the answer you have
chosen. Fill in the space so that the letter inside the oval cannot be
seen.

1. After the funeral, the residents of the apartment building

(A) sent faithfully flowers all weeks to the cemetery


( B) sent to the cemetery each week flowers faithfully
(C) sent flowers faithfully to the cemetery each week
(D ) sent each week faithfully to the cemetery flowers

2. Because the first pair of pants did not fit properly, he asked for

{A) anorher pants


( B) others pants
( C) the others ones
(D) anorher pair

324
2 2 2 2 2 /PRACilCETusrl/ 2 2 2 2 2

3. The committee has met a n d - - - - -


(A) they have reached a decision
(B) it has formulated themselves some opinions
(C) its decision was reached at
(0) it has reached a decision

4. Alfred Adams has not _ _ _ __


(A) lived lonelynessly in times previous
(B) never before lived sole
( C) ever lived alone before
(0) before lived without the company of his friends

5. John's score on the test is the highest in the class;-----


(A) he should study last night
(B) he should have studied last night
(C) he must have studied last night
(0) he must had to study last night

6. Henry will not be able to attend the meeting tonight because

(A) he must to teach a class


(B) he will be teaching a class
( C) of he will teach a class
(0) he will have teaching a class

7. Having been served lunch, _ _ _ _ _


(A) the problem was discussed by the members of the
committee
(B) the committee members discussed the problem
(C) it was discussed by the committee members the problem
(0) a ~iscussion of the problem was made by the members of
the committee

325
[ 2 2 ! 2 2 /PRAcnCE'I'EsTl/ 2 2 2 2 2

8. Florida has not yet ratified the amendment, a n d - - - - -


(A) several other states hasn't either
(B) neither has some of the others states
(C) some other states also have not either
(D) neither have several other states ·

9. The chairman requested t h a t - - - - -


(A) the members studied more carefully the problem
(B) the problem was more carefuJnessly studied
(C) with more carefulness the problem could be studied
(D ) the members study the problem more carefully

10. California relies heavily on income from fruit crops, and

(A) Florida also


(B) Florida too
(C) Florida is as well
(D) so does Florida

11. The professor said that _ _ _ __


(A) the students can turn over their reports on the Monday
( B) the reports on Monday could be received from the stu-
dents by him
( C) the students could hand in their reports on Monday
(D) the students will on Monday the reports tum in

12. This year will be difficult for this organization because

(A) they have less money and volunteers than they had last
year
(B) it bas less money and fewer volunteers than it bad last year
( C) the last year it d id not have as few and little volunteers and
money
(D) there are fewer money and volunteers that in the last year
there were ·

326
2 2 2 2 2 /PRACITCETEsTl/ 2 2 2 2 2

13. The teachers have had some problems deciding - - - - -


(A) when to the students they shall return the final pa pers
(B) when are they going to return to the students the final
papers
(C) when they sho uld return the final papers to the students
(D) the time when the final papers they should return for the
students

14. She wanted to serve some coffee to her guests; however,

(A) she hadn' t many sugar


( B) there was no t a great amount of the sugar
( C) she did not have much sugar
(D) she was lacking in amo unt of the sugar

15. There has not been a great response to the sale, _ _ _ __ ?


(A) does there
( B) hasn't there
(C) hasn't it
(D) has there

Go ON TO PART B

PartB

DIRECTIONS

In questions 16-40, each sentence has four underlined words or


phrases. The four underlined parts of the sentence are marked (A),
(B), (C), and (D). Identify the one underlined word or phrase tha:
must be changed in order for the sente nce to be correct. Then, on
your answer sheet, find the number of the question and fill in the
space that corresponds to the letter of the answer you have chosen.

327
2 2 l 2 2 /PRACTICE TEsT 1/ 2 2 2 2 2

16. The main office of the factory can be found in Maple Street in
A B C 0
New York City.

17. Because there are le~ meP1bers present tonight than there
A B
were last night, we must wait until the next meeting to vote.
C D

18. David is particularly fond of cooking, and he often cooks


A B C
really delicious meals.
0

19. The progress made in space travel for the early 1960s is
A B C 0
remarkable.

20. Sandra has not rarely missed ~ or concert since she


A 8 C
was seventeen years old.
0

21. The governor has not decided how to deal with the new
A B C
problems already.
0

22. There was~ very interesting news on the radio this momin~
A B C
about the e:!rthquake in Italy.
D

23. The professor had already given the homework assignment


A
when he had remembered that Monday was a holiday.
8 r, 0

328

---r
2 2 2 2 2 /PRACTICETEsTl/ 2 2 2 2 2

24. Having been beaten by the police for striking an officer,


A B
the man will cry out in pain.
C D

25. This table is not sturdy enough to support a television, and


A B
that one probably isn't neither.
C D

26. The bridge was hitting by a large ship during a sudden storm
A - B- --C-
last wee k.
D

27. The company representative sold to the manager a


A B
~ewi ng machine for forty dollars.
C D

28. The taxi driver told the man to don't allow his disobedient son
A B C
to hang out the window.
D

29. These televisions are quite popular in Europe, but those ones
A B C
are not.
D

30. Harvey seldom pays his bills on time, and his brother does too.
A B C 0

31. The price of crude o il used to be a great deal lower than now,
A B C
wasn't it?
D

329
2 2 2 ' 2 2 /PRACTICETESTl/ 2 2 2 2 2

32. When an university formulates new regulations, ~


~ . B C
must relay its decision to the students and faculty.
D

33. Jim was upset last night because he had to do too


A B C
many homeworks.
D

34. There is some scissors in the desk drawer in the bedroom if


A B c D
you need them.

35. The Board of Realtors doesn't have any informations


A
about the increase in rent for this area.
B C D

36. George is not enough intelligent to pass this economics class


A B C D
without help.

37. There were so much people trying to leave


A B
the burning building that the police had a great deal of trouble
C D
controlling them.

38. John lived -~ New York since 1960 to 1975, but he is now living
A B C
in Detroit.
D

39. The fire began in the fifth floor of the hotel, but it
A B C
soon spread to adjacent floors.
D

330
2 2 2 2 2 / PRAcnCETESTl/ 2 2 2 2 2

40. Mrs. Anderson bought last week a new sports car; however,
A B
she has yet to learn how to operate the manual gearshift.
c 0

STOP. THIS IS THE END OF THE STRUCTURE ANO WRITTEN EXPRES-


SION SECTION. IF YOU FINISH BEFORE T IME IS UP, CHECK YOUR WO RK
ON PARTS A AND B OF THIS SECTION ONLY. D o NOT WORK ON ANY
OTHER SECTION OF THE TEST.

33 1
'
3 3 3, 3 3 /PRACTICE TEsT 1/ 3 3 3 3 3

SECTION 3
READING COMPREHENSION
Time: 55 Minutes
50 Questions

DIRECTIONS

In this section, you will read a number of passages. Each one is


followed by approximately ten questions about it. For questio ns
1-50, choose the one best answer, (A), (B), (C), o r (D), to each
question. Then, find the number of the question o n your answer
sheet, and fill in the space that corresponds to the letter of the
answer you have chosen. Answer a ll qf the questions following a
passage 0,1 the basis of what is stated or implied in that passage.

Questions 1 through JO are based on the following passage.

The Sto ne Age was a period of history which began in


approximately 2 million B.C. and lasted until 3000 n.c. Jts
name was derived from the stone tools and weapons that
modern scieutists fou nd. This period was divided into the
(5) Paleolithic, Mesol:thic, and Neolithic Ages. During the first
period (2 million to 8000 B.c .), the first hatchet and use of
fire for heating and cooking were developed. As a result of
the Ice Age, which evolved abou t 1 millio n y~a rs into the
Paleolithic Age, people were fo rced to seek shelter in caves,
(10) wear clothing, and develop new tools.
During the Mesolithic Age (8000 to 6000 B.C.) , people
made crude pottery and the first fish hooks, took dogs
hunting, and developed the bow and arrow, which were
used until the fo urteenth century A.D.
(15) The Neolithic Age (6000 to 3000 s.c.) saw humankind
domesticating s heep, goats, ptgs, and cattle, being less
nomadic than in previous eras, establishing permanent
settlements, and creating governments.

332
3 3 3 3 3 / PRACTICE T EST l/ 3 3 3 3 3

1. Into how many periods was the Stone Age divided?


(A ) 2 ( B) 3 (C) 4 (0 ) 5

2. In line 3, the word " derived" is closest in meaning to


(A) o rigi nated .(C) hallucina ted
(B) destroyed (D) discussed

3. Which of the following was developed earliest?


(A) F ish hook (C) Bow and arrow
(B) Hatchet (0 ) Pottery

4. Which of the following developments is NOT related to the


conditions of the Ice Age?
(A) Farming (C) Living indoors
(B) Clothing (D) Using fire

5. The word "crude" in line 12 is closest in meaning to


(A) extravagant (C) vulgar
( B) complex ( D ) primitive

6. The author states that the Stone Age was so named because
(A) it was very durable like stone
(B) the tools and weapons were made of sto ne
(C) there was little vegetation
(D) the people iived in stone caves

7 In lin~ 17, " nomadic" is closest in meaning to


(A) sedentary (C) primitive
( B) wandering ( 0 ) inquisitive

8. With what subject is the passage mainly conce rned?


(A) The Neolithic Age (C) The Stone Age
(B) The Paleolithic Age (D) The Ice Age

333
3 3 3 ' 3 3 I PRACTICE TEST 1/ 3 3 3 3 3

9. Which of the following best describes the Mesolithic Age?


(A) People were inventive.
(B ) People stayed indoors all the time.
(C) People were warriors.
(D) People were crude. ·

10. In line 17, the word "eras" is closest in mea ning to


(A ) families (B) periods (C) herds (D) tools

Questions 11 through 23 are based on the following passage.

Hot boning is an energy-saving technique for the meat


processing industry. It has received significant attention in
recent years when increased pressure for energy conserva-
tion has accentuated the need for more efficient methods of
(5) processing the bovine carcass. Cooling an entire car:;::::;s
requires a considerable amount of refrigerated space, since
bone and trimmable fat are cooled along with the -muscle. It
is a lso necessary to space the carcasses adequately in the
refrigerated room for better air moveme nt and prevention
(10) of microbial contamination, thus adding to the volume
requirements for carcass chillers.
Conventional handling of meat involves holding the beef
sides in the cooler for 24 to 36 hours before boning. Chilling
in the traditional fashion is also associated with a loss of
(15) carcass weight ranging from 2 percent to 4 percent due to
evaporation of moisture fro m the meat tissue.
Early excision, or hot boning, of muscle prerigor followed
by vacuum packaging has several potential advan tages. By
removing only the edible muscle and fat prerigor, refrigera-
(20) tion space and costs are minimized, boning labor is de-
creased, and storage yields increased. Because hot boning
often results in the toughening of meat, a more recent
approach, hot boning following electrical stimulation, has
been used to ! educe the necessary time of rigor mortis.

334
3 3 3 3 3 /PRACI1CETEsT1/ 3 3 3 3 3

(25) Some researchers have found this method beneficial in


maintaining tender meat, while others have found that the
meat also becomes tough after electrical stimulation.

11. The word " accentuated" in line 4 is closest in meaning to


(A) de-emphasized (C) caused
(B) speeded up (D) highlighted

12. All of the following are mentioned as drawbacks of the


conventional method of boning EXCEPT
(A) Storage space requirements
( B) Energy waste
(C) Loss of carcass weight
(D) Toughness of meat

13. In line 3, the word " pressure" is nearest in meaning to


(A) urgency (C) fiavor
( B) weight (D) cooking texture

14. Hot boning is becoming very popular because


(A) it causes meat to be very tender
(B) it helps conserve energy and is less expensive than conven-
tional methods
(C) meat tastes better when the bone is adequately seared
along with the meat
(D) it reduces the weight of the carcass

15. In line 11, " carcass chiller" is nearest in meaning to


(A) a refrigerator for the animal body
(B) a method of boning meat
(C) electrical stimulation of beef
(D) early excision

16. In line 17, " early excision" is closest in meaning to


(A) vacuum packaging (C) carcass chilling
(B) hot boning (D) electrical stimulation

335
I3 3 3 \ 3 ·3 /PRACTICE Tusrl/ 3 3 3 3 3 J
17. The toughening of meat during hot boning has been combatted
by
(A) following hot boning with electrical stimulation
(8) tenderizing the meat
(C) using electrical stimulation before hot boning
(D) removing only the edible muscle and fat pn:rigor

18. The word " bovine" in line 5 is nearest in meaning to


(A) cold (C) beef
(8) electrically stimulated (D) pork

19. The word " this" in line 25 refers to


(A) hot boning
( ~) hot boning fol lowing electrical stimulation
(C) rigor mortis
(0) removing edible muscle and fa t prerigor

20. In line 5, the word "carcass" is closest in meaning to


(A) deboned meat (C) refrigerator
(B) body (D) fat

21. The word "considerable" in line 6 is closest in meaning to


(A) frigid (B) kind (C) lesser (D) substantial

22. One reason it is recommended to remove bones before refriger-


ating is that
(A) it makes the meat more tender
(8) the bones are able to be used for other purposes
(C) it increases chilling time
(D) it saves cooling space by not refrigerating parts that will be
discarded

23. Tl1e word "trimmable" in line 7 is nearest in meaning to


(A) u!lsaturated (C) unhe:ilthy
( B) removable (D) chillcl>le

336
3 3 3 3 3 /PRAcnCE TEsT 11 3 3 3 3 3

Questions 24 through 31 are based on the following passage.

In 1920. after some thirty-nine years of problems with


disease. high co~t:5, and politics, the Panama Canal ~as
officially opened, finally linking the Atlantic an~ Pacific
o~ 9 .,na by <>-llowin~ ships to pass through the fifty-mile canal
(5) zone instead of traveling some seven thommnd miles around
Cape Hom. It takes a ship approxlmaccly eight hours to
~ .... ..,. ·~ ••• , , ·.,, ,\.,..~. . . si."- tb.a c<u1.al :ind costs an average of
fifteen thousand dollars, one tenth of what it would cost an
average ship to round the Horn. More than fifteen thousand
(10) ships pass through its locks each year.
The French initiated the project but sold their rights to
the United States, which actually began the construction of
the p'r oject. The latter will control it until the end of the
twentieth century when Panama takes over its duties.

24. Who currently controls the Panama Canal?


(A) France (C) Panama
(B) United States (D) Canal Zone

25. The word "locks" in line 10 is closest in meaning to


(A) securities (C) lakes
(B) latches (D) canal gates

26. On the average, how much would it cost a ship to travel around
Catie Horn?
(A) $1,500 (B) $15,000 (C) $150,000 (D) $1,500,000

27. In what year was construction begun on the canal?


(A) 1881 (B) 1920 (C) 1939 (D) 1999

337
3 3 3 3 3 /PRACTICE TEST1/ 3 3 3 3 3

28. It can be inferred from this passage that


(A) the canal is a costly project which should be reevaluated
( B) despite allthe problems involved, the project is beneficial
(C) many captains prefer to sail around Cape Horn because it
is less expensive
(D) problems have made it necessary for three governments to
control the canal over the years

29. In line 3, the word " linking" is closest in meaning to


(A) controlling (C) detaching
(B) dispersing (D) joining

30. In line 11, " initiated" is nearest in meaning to


(A) purchased (C) forfeited
(B) launched (D) forced

31. All of the following are true EXCEPT


(A) it costs so much to pass through the locks because very few
ships use them
( B) the United States received the rights to the canal from the
French
(C) a ship can pass through the canal in only eight hours
(D) passing through the canal saves thousands of miles of
travel time around Cape Horn

Questions 32 through. 41 are based on the following passage.

In 776 a.c., the first Olympic Games were held at the foot
of Mount Olympus to honor the Greeks' chief god, Zeus.
The warm climate for outdoor activities, the need for
preparedness in war, and their lifestyle caused the Greeks
(5) to create competitive sports. Only_the elite and military
coti.ld participate at first, but later the games were open to
all free Greek males who had no criminal record. The
Greeks emphasized physical fitness and strength in their
education of youth. Therefore, contests in running, jump-

338
3 3 3 3 3 /PRACTICETEsTl/ 3 3 3 3 J

(IO) ing, discus and javelin throwing, boxing, and horse and
chariot racing were held in individual cities, and the winners
competed every four years at Mount Olympus. Winners
were greatly honored by having olive wreaths placed on
their heads and having poems sung about their deeds.
(15) Originally these contests were held as games of friendship,
and any wars in progress were halted to allow the games to
take place. They also helped to strengthen bonds among
competitors and the different cities represented.
The Greeks attached so much importance to the games
(20) that they calculated time in four-year cycles caJled " Olympi-
ads," dating from 776 e.c. The contests coincided with
religious festivities and constituted an all-out effort on the
part of the pa rticipants to please the gods. Any who
disobeyed the rules were dismissed and seriously punished.
(25) These athletes brought shame not only to themselves, but
also to the cities they represented.

32. Which of the following is NOT true?


(A) Winners placed olive wreaths on their own heads.
( B) The games were held in Greece every four years.
(C) Battles were interrupted to participate in the games.
(D) Poems glorified the winners in song.

33. The word " elite" in line 5 is closest in meaning to


(A ) aristocracy (C) intellectuals
(B) brave (D) muscular

34. Why were the Olympic Games held?


(A) To stop wars
( B) To honor Zeus
(C) To crown the best athletes
(D) To sing songs about the athletes

35. Approximately how many years ago did these games originate?
(A) 800 years (C) 2,300 years
(B) l,200years (D) 2,800years

339
3 3 J.. 3 3 / PRACTICE TEST 1/ 3 3 3 3 3

36. What conclusion can .we draw about t he ancient G reeks?


(A) They were pacifists.
(B) They believed athletic events were important.
(C) They were very simple.
(D) They couldn't count, so they used " Olympiads" for dates.

37. What is the main idea of this passage?


(A) Physical fit ness was an integral part of the lives of the
ancient Greeks.
(B) The Greeks severely punished those who did not partici-
pate in physical fitness programs.
(C) The Greeks had always encouraged everyone to partici-
pate in the games.
(0) The Greeks had the games coincide with religious festivi-
ties so that they could go back to war when the games were
over.

38. In line 14, the word " deeds" is closest in mea ning to
(A) accomplishments (C) d ocuments
( B ) ancestors (D ) property

39. Which of the following was ultima tely required of all athletes
competing in the Olympics?
(A ) They must have completed milita ry service.
( B ) They had to attend special training sessions.
(C) They had to be Greek males with no criminal record.
(D) They had to be very religious.

40. The word " halted" in line 16 meads most nearly the same as
(A) .;nrouraged (C) curtailed
(B) sta~ed (D ) fixed

41. What is an " Olympiad"?


(A) The time it took to finish the games
(B) Th~ tim~ ~~t._e~n games
( C) The time it took to finish a war
(D) The time it took the athle tes to train

340
L3 3 3 3 3 / PRACTICE TEsT 1/ 3 3 3 3 3

Questions 42 through 50 are based on the following passage.

Tampa, Florida, owes a great deal of its growth and


prosperity to a Cuban cigar manufacturer named Vicente
Martinez Ybor. When civil war broke out in 1869, he was
forced to flee his country, and he moved his business to
(5) south Florida. Sixteen years later, labor union problems in
Key West caused him to seek a better location along the
west coast of the state. He bought a forty-acre tract of land
and made plans to set up his cigar factory on the site. This
original sixteen-block stretch of land later expanded to one
(10) hundred acres near Tampa. This newly developed area was
called Ybor City in his honor. Spanish, Italian, and Cuban
immigrants flocked to the area as the demand for workers in
the cigar factory increased. One fifth of the city's twenty
thousand residents enjoyed the high-paying jobs there. At
(15) the e nd of the 1800s, Jose Martf, a Cuban poet and freedom
fighter, organized a revolution from Ybor City and managed
to get considerable support for his movement. Teddy
Roosevelt's " Rough Riders" were stationed there during
the Spanish-American War in 1898. Much of the prosperity
(20) of this region is due to Ybor's cigar factory established more
than one hundred years ago.

42. Where is Ybor City located?


(A) South Florida (C) West Florida
( B) Cuba (D) Marti

43. The word " flee" in line 4 means most nearly the same as
(A) escape (C) fight
(B) return to (D) disembody

44. The word "seek" in line 6 is closest in meaning to


(A) purchase (B) pursue (C) e lude (D) develop

341
3 3 J 3 3 / PRACTICETESTl / 3 3 3 3 3

45 . Why will people probably continue to remember Ybor's name?


(A) He suffered a great deal.
(B) An area was named in his honor.
(C) He was a Cuban revolutionary.
(D ) He was fo rced to flee his homeland.

46. In line 12, the word " flocked" is closest in meaning to


(A) came in large numbers
( B ) escaped hurriedly
( C) increased rapidly
(D) prospered greatly

47. In the early years, how many residents of Ybor City worked in
the cigar factory?
(A) 4,000 ( B) 5,000 (C) 10,000 (D ) 20,000

48. What is the best title for the passage?


(A ) The Spanish-American War
( B ) C uban Contributions in the Development of Ybor City
( C) Ybor's Contribution to Deve loping Part of the Tampa
Area
(D) The Process of Cigar Manufacturing

49. In line 8, "site" is closest in meaning to


(A) location (C) vision
( B ) view (D) indebtedness

50. Who was Jose Martf?


(A ) A good friend of Ybor
( B) One of Teddy Roosevelt's " Ro ugh Riders"
( C) A Cuban write r who sought to free his country
(D ) A worker in the cigar factory

STOP. Tu.ls IS TiiE ENO OF THE EXAMINATION. IF YOU FINISH BEFORE


T IME JS UP, CHECK YOUR WORK IN THIS SECTION O NLY. DO NOT WO RK
ON ANY OTHER SECTION OF THE TEST.

342
1 1 1 1 1 / PRAcnCE TEsT 2/ 1 1 1 t"JJ
PRACTICE TEST 2

SECTION 1
LISTENING COMPREHENSION
Time: Approximately 30 Minutes
50 Questions

Section 1 has three parts. Each part has its own set of directions.
Do not take notes while listening or make any marks on the test
pages. Notetaking, underlining, or crossing out will be considered
cheating o n the .actual TOEFL exam. Answer the questions
following the conversations or talks based on what the speakers
have stated or implied.
For Practice Test 2, restart your Listening Comprehension
cassette immediately following Practice Test 1. On the actual
TOEFL, you will be given extra time to go on to the next page when
you finish a page in the Listening Comprehension section. In the
following test, however, you will have only the 12 seconds given after
each question. Turn the page as soon as you have marked your
answer. Start the cassette now.

Part A

DIRECTIONS

In Part A, you will hear short conversations between two


speakers. At the end of each conversation, a third voice will ask a
question about what was said. The question will be spoken just one
time. After you hear a conversation and the question about it, read
the four possible answers and decide which one wo uld be the best
answer to the question you have heard. Then, on your answer sheet,
find the number of the problem and mark your answer.

343
/PRACTICE TEsT 2/ 'l 1 1 1 1

1. (A) Hand the man a drink.


(B) Drink a diet soft drink.
(C) _Go to the store to buy a drink.
(D) Go without a soft drink.

2. (A) They both liked it.


(B) Neither liked it.
(C) The mother didn't like it, but the father did.
(D) The mother didn't like it because it wasn't in E nglish.

3. (A) A supermarket.. (C) A pharmacy.


( B) A department store. (D) A car repair sh0p.

4. (A) The teacher postponed the conference.


( B) There won't be a test this afternoon.
(C) The students will be attending the conference.
(D) The students took a science test that afternoon.

5. (A) The program was on too late.


(B) The rain didn't let up until after the speech.
(C) He doesn't like thP. president.
(D) He had a late class.

6. (A) Lawyer-client. (C) Dentist-patient.


(B) Doctor-patient. (D) Bank teller-customc!r.

7. (A) There is a quieter place available.


(B) He doesn't care for tennis matches.
(C) The noise should die down shortly.
(D) It's even louder in the meeting room.

8. (A) She's not hungry.


(B) She's at the orthodontist's.
(C) The food tastes like an old shoe.
(D) She's in too much pain .

344
1 1 1 1 I. /PRAcnCE TEsT Z/

9. (A) Packing her own groceries.


(B) A lack of variety in meats.
(C) The unreasonable prices.
(D) The attitude of the employees.

10. (A) She does not feel well enough to return to wo rk.
(B) She hates her work.
(C) She hasn't finished the assignme nt.
(D ) She is still unable to walk.

11. (A) H ome economics. (C) M icrobiology.


( B ) Business administration. (D ) Histor,'.

12. (A) It is no longer delicious.


(B) It makes delicious butte r.
(C) It is the best cheese.
(D ) There are many better cheeses.

13. (A) The game is temporarily delayed because of rain.


(B) T here will be no game if it rains.
(C) There will be a game regardless of the weather.
(D ) It rains every time there is a game.

14. (A) She knew the answer to the question.


( B) She had read t he material, but she didn't know the answer.
(C) She was not prepared for class.
(D) Even though she hadn't read the material, she knew the
answer.

15. (A ) Thirty people returned the evaluation forms.


( B) Sixty people filled o ut the evaluatio n forms.
(C) Eight people returned their forms.
(0) O nly thirty people received the evaluation forms.

345
1 1 1\ 1 1 /PRACTICETEsT2/ 1 1 1 l 1

16. (A) He is a professiohal musician.


(B) He is very talented, but he will never be a professional
musician because he doesn't practice.
(C) He practices every day, but he will never be a professional
musician.
(D) He doesn't want to be a professiona l musician because he
wants to practice law.

17. (A) Stay home if the weather is nice.


(B) Spend the weekend at the beach if the nice weathe r holds
out.
(C) Stay home because the weather will not be pleasant.
(D) Go to the beach if the weather improves.

18. (A) Only he saw the terrible accident.


(B) No one at all saw his terrible accident.
( C) He saw no one in the accident.
(D) No one in the terrible accident saw him.

19. (A) She writes and speaks Spanish equally well.


(B) She both writes and speaks Spanish, but she writes it
better.
(C) Even though she writes Spanish, she speaks it better.
(D) She doesn 't like to write Spanish, but she speaks it.

20. {A) They missed the homework assignment, but they turned it
in later.
(B) They hate each other since their dispute.
(C) They caught a baby squirrel, but they soon let it go.
(D) They had an argument, but now they are friends again.

21. (A) A taxi. (C) A boat.


(B) Aplane. (D) A bus.

346
1 1 l 1 1 /PRACTICETEST2/ 1 1 l 1 l

22. (A) He does no t want to be helpful.


(B) He does not understand the math problem.
(C) He hasn't had a chance to work on the math calculation.
(D) He has already figured out the problem.

23. {A) Mary works in a nursery.


(B) Mary's children stay in a nursery while she works.
(C) Mary takes her children to work with her.
( 0 ) Mary's children are ill today.

24. (A) He will move to Florida when he quits his job here.
(B) As soon as his new job in Florida is confirmed, he will move
there.
(C) He wants to move to Florida, but he can't find a job there.
(D) He plans to move to Florida when he retires.

25. (A) He doesn't like fishing on a hot, summer day.


(B) Although he likes fishing, he doesn't want to do it on a hot,
summer day.
(C) Fishing is his favorite enjoyment on a hot, summer day.
(D) He loves to eat hot fish for breakfast in the summer.

26. (A) When the production had begun, they realized that they
should have practiced more.
( B) Before the production began, they reviewed their lines one
more time.
(C) Although they had practiced for months, the production
was a flop.
(D) They went to the theater in two separate cars.

. 27. (A) She gave the class an assignment.


(B) She gave the students a hand with their assignments.
(C) She asked the students to turn in their assignments.
(D) She asked the students to raise their hands if they wanted
to ask a question about the assignm ent.

347
1 1 }, 1 1 /PRACTICE TEsT 2/ 1 1 1 1 1

28. (A) Stacey will buy their dog.


(B) After they return from vacation, they are going to buy a
dog.
(C) Stacey will take care of their dog while they are on
vacation.
(D) Stacey will be very tired after the long vacation.

29. (A) It originated in the United States.


(B) It's very popular in Scotland.
(C) It originated in the United States, but now it's more
popular in Scotland.
(D) It originated in Scotland, but now it's more popular in the
United States.

30. (A) He saw them thirteen years ago.


(B) They arrived thirty years ago.
( C) He has not seen them for thirty years.
(D) He sees them every thirteen years.

Go ON TO PART 8

Part B

DIRECTIONS

In Part B, you will hear longe r conversations. After each


conversation, you will be asked some questions. The conversations
and questions will be spoken just one time. They will not be written
out for you, so you will have to listen carefully in order to
understand and remember what the speaker says.
When you hear a question, read the four possible answers in your
test book and decide which one would be the best answer to the
question you have heard. Then, on your answer sheet, find the
number of the problem .and fill in the space that corresponds to the
letter of the answer you have chosen.

348
[ 1 1 1 1 1 . / PRACTICE TEST 2/ 1 1 1 1 l

31. (A) She was sick.


(B) She couldn't make up her mind as to which countries she
should visit.
(C) She couldn't think of a topic for her composition.
(D) She was totally disorganized.

37.. (A) That she ta:Ce a cruise.


( B) That she try to get o rganized.
(C) That she ride a camel.
(D) That she write about her trip.

33. (A) H1..nga ry. (C) Egypt.


(B) No rth Africa. (D) The Holy Land.

34. (A) To pack his bags for his trip.


(B) To write his own composition.
(C) He's no t feeling well.
(D) To pick up some photographs.

35. (A) Type his paper.


(B) Help him with his research.
(C) Prese nt his findings at the July conference.
(D) Verify his findings.

36. (A) He's about to leave for a new job.


( B) He wants to present it at a conference.
(C) His employer has requested it.
(D) It's very important for his livelihood.

37. (A) July. (C) May.


(B) September. (D) February.

349
I• 1 1 l 1 i /PRACTICE TEsT 2/ 1 1 1 1 1

38. (A) He's completed typing his 11ot.:s.


(B) He's completed the research.
(C) He's still performing research.
(D) He's begun typing.

Go ON TO PART C

Part C

DIRECTIONS

In Part C, you will hear several talks. After each talk, you will be
asked some questions. The talks and questions will be !;poken just
one time. They will not be written out for you, so you will have to
listen carefully in order to understand and remember what the
speaker says.
When you hear a question, read the four possible answers in your
test book and decide which one would be the best answer to the
question you have heard. Then, on your answer sheet, find the
number of the problem and fill in the space that corresponds to the
letter of the answer you have chosen.

39. (A) Nathaniel Bacon and his friends fought against Indian
marauders.
(B) Bacon and his friends were Piedmont farmers.
(C) Bacon and a few farmers marched on the capital to protest
the Indian raids.
(D) Governor Berkeley did not listen to the demands of the
farmers.

40. (A) Less than 1 year. ( C) 10 years.


(B) 5 years. (D) 23 years.

350
1 1 1 t 1 /PRACTICE TEsT 2/ ·t 1 1 l 1

41. (A) He was killed by Indians.


( B) Governor Berkeley had him hanged.
(C) He succumbed to malaria.
(0) He w;i" accidP.ntly shot by one of the farmers.

42. (A) Death of its sculptor.


( B) Lack of funds.
(C) Disinterest in the project.
(D) Too many Indian raids.

43. (A) Abraham Lincoln. (C) Thomas Jefferson.


(B) Franklin Roosevelt. (0) George Washington.

44. (A) 27 years old. (C) 60 years old.


(B) 41 years old. (D) 74 years old.

45. (A) They bear little resemblance to the people they represent.
(B) The figures are gigantic, but too serious.
(C) They portray the people they represent.
(0) Because they are old and weatherbeaten, the facc-
disfigured.

46. (A) This magnificent work of art is located very high in the
Black Hills.
(B) Four American presidents have been sculpted as a lasting
memorial to their leadership.
( C) It took fourteen years to complete the project.
(0) Gutzon Borglum was near retirement age when he began
this project.

47. (A) In a chemistry class.


(B) At a gas station.
(C) Near an oil well.
(D) In a nuclear plant.

351
I 1 1 1 1 1 /PRACTICET'EsT2/ 1 1 1 1 JJ
48. (A) Refined oil.
(B) Unrefined oil.
(C) A mixture of simple inorganic compounds.
(D) The product of burning.

49. (A) By the percentage of nitrogen.


(B) By the percentage of oxygen.
( C) By the percentage of hydrogen and carbon.
(D) By the percentage of sulfur.

50. (A) Oil that has been separated by distilling.


(B) Oil that has greater than one percent sulfur content.
(C) Oil that has less than one percent sulfur content.
(D) Oil that is in its simplest form.

STOP. THIS IS TIIE END OF THE LISTENING COMPREHENSION SECTION.


Go ON TO SECTION 2.

352
2 2 2 2 2 / PRACITCE TEsT 2/ 2 2 2 2 2

SECTION 2
STRUCTURE AND WRITTEN EXPRESSION
Time: 25 Minutes
40 Questions

Part A

DIRECTIONS

Questions 1- 15 are incomplete sentences. Beneath each sentence


you will sec fo ur wo rds o r phrases, marked (A), (B), (C), and (0).
Choose the one word o r phrase that best comple tes the sentence.
Then, on your answer sheet, find the number of the question and fill
in the space that corresponds to the Jetter of the a nswer you have
chosen. Fill in the space so that the letter inside the oval cannot be
seen.

1. Captain Henry, - - -- - crept slowly through the under-


brush.
(A) being remote from the enemy,
(B) attempting to not encounter the enemy,
(C) trying to avoid the enemy,
(0) not involving himself in the enemy,

2. Tommy was one _ __ __


(A) of the happy childs of his class
( B) of the happiest child in the class
( C) child who was the happiest of all the class
(0) of the happiest children in the class

3. he began to make friends more easily.


(A) Having ente red school in the new city, it was fou nd that
( B) After entering the new school,
(C) When he had been entering the new school,
(0) Upon entering into the new school,

353
(

2 2 2 2 ·2 /PRACilCE TEsT2/ 2 2 2 2 2

4. It is very difficult to stop the cultivation of marijuana because

(A) it grows very carelessly


(B) of it's growth without attention
(C) it grows well with little care
(D) it doesn't care much to grow

5. The fact that space exploration has increased dramatically in


the past thirty years------
(A) is an evidence of us wanting to know more of our solar
system
(B) indicates that we are very eager to learn all we can about
our solar system
(C) bow we want to learn more about the solar system
(D) is pointing to evidence of our intention to know a lot more
about what is called our solar system

6. Many of the international problems we are now facing

(A) linguistic incompetencies


(B) are the result of misunderstandings
(C) are because of not understanding themselves
(D) lacks of the intelligent capabilities of understanding each
other

7. Mr. Roberts is a noted chemist _ _ _ __


(A) as well as an effective teacher
(B) and too a very efficient teacher
( C) .but he teaches very good in addition
(D) however he teaches very good also

354
2 2 2 2 2 /PRACTICE TEsr2/ 2 2 2 2 2

8. Public television stations are different from commercial stations

(A) because they receive money differently and different types


of shows
( 8 ) for r.ioney and program types
(C) in the areas of funding and programming
(D) because the former receives money and has programs
differently from the latter

9. Manufacturers often sacrifice quality _ _ __ _


(A) for a larger profit margin
( B) in place of to earn more money
( C) to gain more quantities of money
(D) and instead earn a bigger amount of profit

10. Automobile production in the United States------·


(A) have taken slumps and rises in recent years
(B) has been rather erratic recently
(C) has been erratically lately
(D) are going up and down all the time

11. A major problem in the construction of new buildings

(A) is that windows have been eliminated while air condition-


ing systems have not been perfected
(B) is they have eliminated windows and still don't have good
air conditioning
(C) is because windows are eliminated but air conditioners
don't work good
(D) is dependent on the fact that while they have eliminated
windows, they are not capable to produce efficient air
conditioning systems

355
2 2 ! 2 2 / PRACTICE TEsT 2/ 2 2 2 2 2 J
12. John said that no o ther car could g o - - -- -
(A ) so fast like his car
( B) as fast like his car
(C) as fast like the car of him
( D ) as fast as his car

13. Her grades have improved, but only--- - - -


(A) in a small amount
(B) very slightly
(C) mmunum
(D) some

14. While attempting to reach his home before the storm,

(A ) the bicycle of John broke down


(B) it happened that Jo hn's bike broke down
(C) the storm caught John
(D) Jo hn had an accident on his bicycle

15. The changes in this city have occurred------


(A ) with swiftness
(B) rapidly
(C) fastly
(D ) in rapid ways

Go ON TO PART B

PartB
DIRECTIONS

In questions 16-40, each sentence has four underlined words or


phrases. The four underlined parts of the sentence are marked (A),
(B), (C), and (D). Identify the one underlined word or phrase that
must be changed in order for the sentence to be correct. Then, o n
your answe r sheet, find the number of the question and fill in the
space that corresponds to the letter of the answer you have chosen.

356
2 2 2 2 2 /PRACTICE TEsT2/ 2 2 2 2 2

16. The officials object to them wea ring long dresses for the
A B C
inaugural dance at the country club.
D

17. Janet is finally used to cook o n an electric stove after having a


A B
gas one fo r so long.
C D

18. He knows to repair the carbure tor without taking the whole
- A- B -c-
car a part.
D

19. Stuart stopped to write his letter because he had to leave


A B C
for the hospita l.
D

20. She must retyping the report before she hands it in


A B C
~the director of financing.
D

21. How much times did Rick a nd Je nnifer have to do the


A B
experiment before they obtained the results they had
c
been expecting?
D

22. Each of the students in the accounting class has to type their
A - B- -C
own research pape r this semester.
D

357
2 2 2 ~ 2. /PRACTICE TEsT 2/ 2 2 2 2 2

23. Mrs. Stevens, along with her cousins from New Mexico,
A B
are planning to attend the festivities.
c 0

24. They are going to have to leave soon, and so do we.


A B C 0

25. All the students are looking forward spending their free:: time
A B C D
relaxing in the sun this summer.

26. Dresses, skirts, shoes, and children's clothing are advertised at


A B C
great reduced prices this weekend.
0

27. Mary and her sister just bought two new winters coats at the
A B C D
clearance sale.

28. A lunch of soup and sandwiches do not appeal to all of the


A B C D
students.

:?.9. Some of us have to study their lessons more carefully if we


A B C
expect to pass this examination.
D

30. Mr. Peter:> :.:sed to think of hisself as the only president


A B C
of the company.
D

358
2 2 2· Z 2 /P.RACTICE Tusr2/ .2 2 2 2 2

31. The instructor advised the students for the procedures to


A B
follow in writing the term paper.
C D

32. Although both of them are trying to get the scholarship, she
A B C
has the highest grades.
D

33. The new technique calls for heat the mixture before applying
--;::- -6- c
it to the wood.
D

34. The pilot and the crew distributed the life preseivers
A B
between the twenty frantic passengers.
C D

35. A five-thousand-dollars reward was offered


-A - -B - C
for the capture of the escape d criminals.
D

36. The equipment in the office was badly in need of


A B C
!9 be repaired.
n

37. ~ lit er is one of the metric measurements, aren't they?


A B C D

38. We thought he~ planning ~o go on vacation after


A B C
the first of the month.
D

359
2 2 2 2 2 /PRACTICE TEsT2/ 2 2 2 2 ~
39. There are a large supply of pens and notebooks
A B
in the storeroom to the left of the library entrance.
C D

40. The president refuses to accept either of the four


A B
new proposals made by the contractors.
C D

STOP. THIS IS THE END OF THE STRUCT1JRE AND WRITTEN EXPRES-


SION SECTIO N. IF YOU FINISH BE FOR E TIME IS UP, CHECK YOU R WORK
ON PARTS A AND B OF THIS SECTION O NLY . D o NOT WO RK ON ANY
OTHE R SECTION O F TH E TEST .

360
3 3 3 3 3 / P RACI1CE TEsT 2/ 3 3 J 3 3

SECT ION 3
READING COMPREHENSION
Time: 55 Minutes
SO Questio ns

DIRECTIONS

In this sectio n, you will read a number of passages. Each o ne is


followed by approximately ten questions about it. For questions
1-50, choose the one best answer, (A), (B). (C), or (D), to each
question. Then, find che number of the question o n your a nswer
s h eet , a nd fill in the space that corresponds to the lette r of the
answer you have chosen. Answer all of the questions fo llowing a
passage on the basis o f what is stated or implied in that passage.

Questions I through I 0 are based on the following passage.

Napo leo n Bo naparte's ambitio n to control all the area


arou nd the Mediterranean Sea led him and his French
soldiers to Egypt. After losing a nava l battle, they were
forced to remain there for three yea rs. In 1799, while
(5) constructing a for t, a soldier discovered a piece o f stele (a
stone pillar bearing an inscription) known as t he Rosett a
stone , in commemoration o f the town near the fort. This
famous stone, which would eventually lead to the decipher-
ing of ancient Egyptian hieroglyphics dating to 3100 B.C.,
(10) was written in three languages: hieroglyphics (picture writ-
ing), demotic (ii shorthand version of Egyptian hie roglyph-
ics), and Greek. Scientists discovered that the characters,
unlike those in English, could be written from right to left
and in other directions as well. The direction in which they
( 15) were read depended o n how the characters were arranged.
Living elements (animals, people, and body parts) were
ofte n the first symbols, and the direction that they faced
indicated the direction for reading them.

361
3 3 3' 3 3 /PRACTICETEST2/ 3 3 3 3 3

Twenty-three years after the discovery of the Rosetta


(20) stone, Jean Fram,;ois Champollion, a French philologist
fluent in several languages, was able to decipher the first
word-Ptolemy-the name of an Egyptian ruler. This name
was written inside an oval called a " cartouche." Further
investigation revealed that cartouches contained names of
(25) important people of that period. Champollion painstakingly
continued his search and was able to increase his growing
list of known phonetic signs. He and an Englishman,
Thomas Young, worked independently of each other to
unravel the deeply hidden mysteries of this strange lan-
(30) guage. Young believed that sound values could be assigned
to the symbols, while Champollion insisted that the pictures
represented words.

1. All of the following languages were written on the Rosetta


stone EXCEPT
(A) French (C) Greek
(B) demotic (D) hieroglyphics

2. All of the following statements are true EXCEPT


(A) cartouches contained names of prominent people of the
period
(B) ChampolJion and Young worked together in an attempt to
decipher the hieroglyphics
( C) one of Napoleon's soldiers discovered the Rosetta stone
(D) Thomas Young believed that sound values could be
assigned to the symbols

3. The word "deciphering" in line 8 is closest in meaning to


(A) decoding (C) discovery
(B) downfall (D) probing

4 The first word deciphered from the Rosetta stone was


(A) cartouche (C) demotic
(B) Ptolemy (D) Champollion

362
3 3 3 3 3 /PRACI19E T:EsT·2/. 3 3 3 3 3·.

5 . Napoleon's soldiers were in Egypt in 1799 because they were


(A) celebrating a naval victory
(B) looking for the Rosetta stone
(C) waiting to continue their campaign
(D) trying to decipher the hieroglyphics

6. The person responsible for deciphering the first word was


(A) Champollion (C) Ptolemy
(B) Young (D) Napoleon

7. Why was the piece of newly discovered stele called the Rosetta
stone?
(A) It was shaped like a rosette.
(B) It was to honor Napoleon's friend Rosetta.
(C) The town near the fort was called Rosetta.
(D) The fort was called Rosetta.

8. In line l , "ambition" is nearest in meaning to


(A) aspiration (C) indifference
(B) indolence (D) apathy

9. What is the best title for the passage?


(A) Napoleon's Great Discovery
(B ) Deciphering the Hieroglyphics of the Rosetta Stone
(C) Thomas Young's Great Contribution
(D) The Importa nce ofCartouches

10. In which lines of the reading passage is the direction for reading
hieroglyphics discussed?
(A) Lines 5-8 ( C) Lines 19-22
(B) Lines 14-18 (D) Lines 25-27

363
3 3 3' 3 3 / PRACTICE TEST2/ 3 3 3 3 3

Questions 11 through 20 are based o n the following p assage.

Sequoyah was a young Cherokee Indian. son of a white


trader and an Indian squaw. At an early age, he became
fascinated by "the talking leaf," an expression that he used
to describe the white man's written records. Although many
(5) believed this "talking leaf' to be a gift from the Gr<:;at Spirit,
Sequoyah refused to accept that theory. Like othe r Indians
of the period, he was illiterate, but his determination to
remedy the situa tion led to the invention of a unique
eighty-six-character alphabet based on the sound patterns
(10) that he heard.
His family and frie nds thought him mad. but while
recuperating from a hunting accident, he diligently and
independently set out to create a form of communication for
his own people as well as for othe r Indians. In 1821, after
(15) twelve years of work, he had successfully developed a
written language that would enable thousands of Indians to
read and write.
Sequoyah's desire to preserve words and events for later
gene ra tions has caused him to be remembered among the
(20) important inventors. The giant redwood trees of California,
called "5equoias" in his hono r, will further imprint his name
in history.

11. What is the most important reason that Sequoyah will be


remembered?
(A) California redwoods were named in his ho nor.
( B) He was illiterate.
(C) He created a unique alphabet.
(D) He recovered from his madness and helped humankind.

12. The word "squaw" in line 2 is closest in meaning to


(A) wcm.a?: (B) teacher (C) cook (D) trade r

364
3 3 3 3 3 . /PRACTICETEST2/ 3 3 3 3 3

13. How did Sequoyah's family react to his idea of developing his
own "talking leaf' ?
(A) They arranged for his hunting accident.
(B) They thought he was crazy.
(C) They decided to help him.
(0) They asked him to teach them to read and write.

14. What prompted Sequoyah to develop his alphabet?


(A) People were writing things about him that he couldn't
read.
(B) He wanted to become famous.
(C) After his hunting accident, he needed something to keep
him busy.
(0) He wanted the history of his people preserved for future
generations.

15. In line 7, the word " illiterate" means most nearly the same as
(A) fierce
(B) poor
(C) abandoned
(0) unable to read or write

16. It is implied that Sequoyah called the written records " the
talking leaf' because
(A) they played music
(B) when be observed white people reading, they seemed to
understand what was written
(C) he was going mad, and he thought the leaves were talking
to him
(0) it was the only way that the Great Spirit had of communi-
cating with them

17. Sequoyah could best be described as


(A) determined (C) backwards
(B) mad (0) meek

365
3 3 3, 3 3 /PRAcnCETEST2/ 3 3 3 3 3

18. What is the best title for the passage?


(A) Sequoyah 's Dete rmination to Prese rve the Cherokee Lan-
guage
(B) The Origin of the Cherokee Language
\ C) Scquoyah's Madness Leads to a New Language
(D) The Origin of the " Sequoia" Trees in California

19. In line 3, "fascinated" is closest in meaning to


(A) absorbed (C) confused
(B) exasperated (D) imaginative

20. All of the following are true EXCEPT


(A) Sequoyah developed a form of writing with the help of the
Cherokee tribe
(B) Sequoyah was a very obseivant young man
(C) Sequoyah spent twelve years developing his alphabet
(D) Sequoyah was honored by having some trees named after
him

Questions 21 through 30 are based on the following passage.

The mighty, warlike Aztec nation existed in Mexico from


1195 to 1521. The high priests taught the people that the sun
would shine, the crops would grow, and the empire would
prosper only if the gods were appeased by human sacrifices
(5) and blood offerings from all levels of their society. The
pr iests practiced forms of self-mutilation, such as piercing
their tongues with thorns and flagellating themselves with
thorn branches. They collected the small amount of blood
produced by these practices and offered it to Huitzilopochtli
(10) and Quetzalcoatl, their chief gods. They insisted that all
Aztecs needed to make some sort of daily sacrifice. Warriors
were promised a place of honor in the afterlife if they died
CO"...:~: ::::sly in battle.
The Aztecs were constantly at war in order to have
(15) enough captives from battle to seive as sacrificial victims.

366
3 3 3 3 3 /PRACTICE TusT2/ 3 3 3 3 3 ,,

The prisoners were indoctrinated before their deaths in to


believing that they, too, would find a place of honor in the
afterlife and that their death insured the prosperi~y of the
great Aztec nation. After being heavily sedated with mari-
(20) juana or a similar drug, they were led up the steps t" the top
of the ceremonial centers where they accepted their fate
passively, and their palpitating hearts were removed from
their bodies as an offering to the gods.

21. Why did the Aztecs offer human sacrifices?


(A) They were cruel and inhuman.
(B) They believed they had to pacify the gods.
(C) They wanted to force the citizens to obey.
(D) They wanted to deter crime.

22. Before the sacrifices, the victims were


(A) tortured and harassed
(B) fed and entertained
(C) brainwashed and drugged
(D) interrogated and drugged

23. In what manner did the victims accept their destiny?


(A) Submissively (C) Violently
(B) Rebelliously (D) Notoriously

24. The word "appeased" in line 4 is closest in meaning to


(A) glorified (C) angered
(B) assaulted (D) satisfied

25. What is the best title for the passage?


(A) The Aztecs' Need to Offer Human Sacrifice
(B) Aztec Victims
(C) The History of the Mighty Aztec Nation
(D) Aztec High Priests

367
11

3 3 3 3 3 / PRACTICE TEsT2/ 3 J 3 3 3

26. What did the Aztecs b el ieve t h e gods c r aved in o rder to e nsure
the people's survival?
(A) Sunshine (B) Blood (C) Thorns (D) Drugs

27. Which of the following is NOT given as a reason fo r offering


human sacrifice?
(A) The sun would not rise.
(B ) The crops would not grow.
(C) The warriors would not be famous.
(D ) The empire would not be successful.

28. Why-were the victims willing to accept their fate?


(A) They liked to see the sun shine.
(B ) They wante d everyone to see them at the top of the
ceremonial centers.
(C) They were made to believe they would have a place of
honor in eternity.
(D) They liked to take drugs.

29. Which of the following is described as a form of self-torture that


the high priests practiced?
(A) Indoctrination
( B) Heavy sedation
(C) Piercing their tongues
(D) Sacrificing victims

30. In line 1, the word " mighty" is closest in meaning to


(A) primitive ( C) meticulous
(B) unimposing (D) powerful

368
3 3 3 3 3 /PRACTICETEsT2/ 3 3 3 3 3

Ques;ions 31 through ;.<J are based on the following passage.

P etroleum products, such as gasoline, kerosene, home


heating oil, residual fuel oil, and lubricating oils, come from
one source--crude oil found below the earth's surface, as
well as under large bodies of water, from a few hundred feet
(5) below the surface to as deep as 25,000 feet into the earth's
iwerior. Sometimes crude oil is secured by drilling a hole
imo the earth, but more dry holes are d rilled than those
producine oil. ~ither pressure at the source or pumping
forces crude oil to the surface.
(:Q) Crude oil wells flow at varying rate:;, f-o m about ten to
thousands o f barrels per hour. Petroleum products are
always measured in forty-two-gallon barrels.
Petroleum products vary greatly in physical appearance:
thin, thi..:k, transparent, or opaque, l:iut regardless, their
(15) chernicc.I composition is made up of only two elements:
carbo11 und hydrogen, which form compounds called hydro-
:::arbons. Other chemical elements found in union with the
hydrocarbons are few and are classified as impurities. Trace
elements are also found, but in such minute quantities that
(20) thev 4\re disregarded. The combination of carbon and
hydrogen forms many thousands of compounds which are
pos~ib lc because 0f the various positions and unions of
these two atoms in the hydrocarbon molecule.
'fhe variot.s petroleum products are refined by heating
(25) crude oil arid then condensing the vapors. These products
arc the so-called light o ils, such as gasoline, kerosene. and
distillate oil. The residue remaining after the light oils are
distilled is known as heavy or residua l fuel oil and is used
mostly for burning under boilers. Additional complicated
~'.:0) refining processes rearrange the chemical structure of the
hydrocarbons to produce other products, some of which are
used 10 upgrade and increase the c.ctane rating of various
types of gasoline.

369
3 3 3 3 3 / PRACTICE TEsT2/ 3 3 3 3 3

31. All of the following are true EXCEPT


(A) crude oil is found below land and \W'iter
(B) crude oil is always fo unrl a few hundred feet below the
surface
(C) pumping and pressure force crude oil to the surface
(D) many petroleum products are obtained from crude oil

32. The word " minute" in line 19 is closest in meaning to


(A) instant (B) huge (C) in significant (D) timely

33. Many thousands of hydrocarbon compounds are possible be-


cause
(A) the petroleum products vary greatly in physical appearance
(B) complicated refining processes rearrange the chemical
structure
(C) the two atoms i:i the molecule assume ma ny positions
(D ) the pressure needed to force it to the surface causes
molecular transformation

34. In line 32, the word " upgrade" is closest in meaning to


(A) improve (C) charge
( B) counteract (D) unite

35. Which of the following is true?


(A) The various petroleum products are produced by filtratiun.
(8) Heating and condensing produce the various products.
(C) Chemical separation is used to produce the various prod-
11.cts.
(D) Mechanical means, such as centrifuging, are nsed to
produce the various products.

36. The word " opaque" in line 14 means most nearly the same as
(A) transparent (B) turbid (C) ligt.t (D) crude

370
3 3 3 3 3 /PRACTICETEsT2/ 3 3 3 3 3

37. How is crude oil brought to the surface?


(A) Expansion o f the hydrocarbons
(B) Pressure and pumping
(C) Vacuum created in the drilling pipe
(D) Expansion and contraction of the earth's surface

38. All of the following are listed as light oils EXCEPT


(A) Distillate oil (C) Lubricating oil
(B) Gasoline (D) Kerosene

39. What are the principal components of all petroleum products?


(A) Hydrogen and carbon
(B) Residual fuel oils
( C) Crude oils
(D) Refined substances

40. The word " condensing" in line 25 is nearest in meaning to


(A) cooling (C) diluting
( B) expanding (D) refuting

i' 1. The word " they" in line 20 refers to


(A) impurities (C) hydrocarbons
( B) minute quantities (D) trace elements

Questions 42 through 50 are based on the following passage.

In the U nited States, presidential elections are held in


years evenly divisible by four (1884, 1900, 1964, etc.). Since
1840, American presidents elected in years ending with zero
have died in office, with one exception. William H. Harri-
(:)) S<'n, the man who served the shortest term, died of pneumo-
nia on.1y several weeks after his inauguration.
Abraham Lincoln was one of four presidents who were
assr..ssinated. He was elected in 1860, and his untimely death
came just five years later. James A. Garfield, a former

3"11
3 3 3, 3 3 /PRACTICETEST2/ 3 3 3 3 3

(10) Union army general from Ohio, was shot during his first
year in office (1881) by a man to whom he wouldn't give a
job. While in his second term of office (1901), William
McKinley, another Ohioan, attended the Pan-American
Exposition in Buffalo, New York. During the reception, he
( 15) was assassinated while shaking hands with some of the
guests. John F. Kennedy was assassinated in 1963 in Dallas
only three years after his election.
Three years after his election in 1920, Warre n G. Harding
died in office. Although it was never proved, many believe
(20) he was poisoned. Franklin D . Roosevelt was elected four
times (1932, 1936, 1940, and 1944), the only man to serve so
long a term. He had contracted polio in 1921 and eventually
died of the illness in 1945.
Ronald Reagan, who was elected in 1980 and re-elected
(25) four yea rs later, suffered an assassination attempt but did
not succumb to the assassin's bullets. He was the first to
break the long chain of unfortunate events. Will the
candidate in the election of 2000 also be;: as lucky?

42. All of the following were election years EXCEPT


(A) 1960 (B) 1930 (C) 1888 (D) 1824

43. Which presider.t served the shortest term in office?


(A) Abraham Lincoln (C) William McKinley
(B) Warren G. Harding (D) William H. Harrison

44. Which of the following is true?


(A) All presidents elected in years ending in zero have died in
office.
(B) Only presidents from Ohio have died in office.
(C) Franklin D. Roosevelt completed four terms as president.
(D) Four American presidents have been assassinated.

45. How many presidents e lected in years ending in zero since 1840
have died in office?
(A) 7 (B) 5 (C) 4 (D) 3

372
3 3 3 3 3 /PRACTICE TEsT2/ 3 3 3 3 3

46. 1he wo rd " in a ug uration" in line 6 m eans m ost nearly the sam e
as
(A) election
(B) acceptance speech
(C) swearing-in ceremony
(0) campaign

47. Ali of the following presidents were assassinated EXCEPT


(A) John F. Kennedy (C) Abraham Lincoln
(B) Franklin 0 . Roosevelt (0) James A. Garfield

48. The word " whom" in line 11 refers to


(A) Garfield
( B) Garfield's assassin
(C) a Union army general
(D) McKinley

49. The word "assassinated" in line 8 is closest in meaning to


(A) murdered (C) honored
(B) decorated (D) sickened

50. In line 22, "contracted" is closest in meaning to


(A) communicated about (C) agreed about
(B) developed (D) notified

STOP. THIS IS THE END OF THE EXAMINATION. IF YOU FINISH BEFORE


TIME IS UP, CHECK YOUR WORK IN THIS SECTION ONLY. Do NOT WORK
ON ANY OTHER SECTION O F THE TEST.

373
1 1 ~ l 1 / PRACITCE T EST 3/ 1 1 1 1 1

PRACTICE TEST 3

SECTION 1
LISTENING COMPREHENSION
Time: Approximately 3e Minutes
50 Questions

Section l has three pa rts. Each pan 11as its own sel. of directions.
Do not take notes while liste ning or make any marks on the test
pages. Notc taking, underlining, o r crossing out will be considered
cheating on the actual TOEFL exam. Answer the questions
following the conversations or talks based on what the speakers
have stated or implied.
For Practice T est 3. restart your Listening Compre hension
cassette immediate ly following Practice Test 2. O n the actual
TO EFL, you will be given extra time to go on to the next page when
you finish a page in the Listening Comprehensio n section. In the
following test, however, you will have only the 12 seconds given afte r
each question. Turn the page as soon as you have mar kt>d your
answer. Start the cassette now.

Part A

DIRECTIONS

In Part A, you will hear short conversations between two


speakers. At the end of each conversation, a third voice will ask a
question about what was said. The question will be spoken just one
time. After you hear a conversation and the question about it, read
the fo ur possible answers and decide which one would be the best
answer to the question you have heard. Then, on your answer shee t,
find the number of the problem and mark your answer.

1. t'.A) April. (B) May. (C) June. (D) July.

374
1 1 1 1 1 / PRACTICE TEST 3/ 1 1 1 1 1

2. (A) Philadelphia. (C) Doctors.


( B) Chapmans. (D ) Arizona.

3. (A) He'll see if he can get the computer going.


( B) It's a very good computer.
( C) Dana has a copy of the manual in the back office.
(D) The woman was wise to have copied her data.

4. {A) A gas station.


(B) A police station.
{C) A lost-and-fo und department.
(D) A bar.

5. (A) J ason Daniels isn't ho me right now.


( B) The caller dialed the wrong number.
(C) Jason Daniels can't come to the phone 1ight now.
(D) Jason Daniels doesn't want to spe.ik to the caller.

6. (A) She's o n a committee.


(B) She's been working late.
( C) She exercises too much.
(D) She's trying to budget her sleep.

7. (A) Be tter. (B) Sick. (C) Fine. {D) Tired.

8. (A) No, because it's not for sale.


(B) Yes, because he has plenty of money.
(C) Yes, if he bo rrows the money from the woman.
(D) No, because he didn 't bring enough money.

9. (A) Europe. (C) Canada.


( B) Where the speakers are. (D) California.

375
1 1 1 l 1 /PRACTICE TEsT 3/ 1 1 l 1 1

10. (A) Her car is being repaired at the gas station.


(B) Frank is going to the gas station to pick up her car.
( C) She has gone to get her gas tank filled with gasoline.
(D) Her car isn't working properly because of the type of
gasoline that she is using.

11. {A) Although they knew there was going to be a meeting, they
didn't come.
(B) They didn't want to attend the meeting, but they d.id
anyway.
(C) They didn't know about the meeting.
(D) They didn't let anybody know about the meeting, so
nobody attended.

12. (A) He made the best grade in his class.


(B) He is an exceptionally good student.
(C) His classmates made good grades, but he didn't.
(D) He is one of the better students in his class.

13. (A) The dean was asked to question several students.


(B) The humanities professor questioned several students.
(C) The humanities professor was able to answer the students'
questions.
(D) The humanities professor has asked the dean a question
about some students.

14. {A) Refuse to work.


(B) Leave early.
( C) Request to work overtime.
(D) Ask for assistance.

15. (A) He went to the concert because he didn't want to work.


(B) He didn't go to the concert because he had too much work
to do.
(C) Although he had a lot of work to do, he went to the
concert.
(D) He never goes to a concert if he has work to do.

376
[1 1 1 1 1 /PRACTICETEsT3/ 1 1 1 1 1

16. (A) He lost the library's new books.


(B) He is going to the new library to look for some books.
( C) He may keep the library books longer.
(D) He had to pay a late fee for the books.

17. (A) He is afraid to start smoking because of the hazardous


effects.
(B) He is afraid he'll become fat if he stops smoking.
(C) He is afraid that he will become more nervous if he stops
smoking.
(D) He doesn 't realize the possible dangers of smoking.

18. (A) He studies regularly, but his grades are suffering.


(B) He is so lazy that he never gets good grades.
(C) He hasn't studied lately but will likely get good grades.
(D) He probably will not pass because he hasn't studied.

19. (A) His meat wasn't tender.


(B) The speaker did not have a good character.
(C) It was difficult to meet new people in the crowd.
(D) The meeting was cut short.

20. (A) He never forgets when he has a meeting.


( B) It seems that he forgot about their meeting.
(C) He should have canceled the meeting.
(D) He has to come to the meeting.

21. (A) Vegetables. (B) Fruit. (C) Meat. (D) Cookies.

22. (A) They couldn't afford a honeymoon.


(B) They went to Puerto Rico.
(C) They went to St. Augustine.
(D ) They are still planning on going to Puerto Rico.

377
l 1 1-. 1 1 / PRACTICE TEST 3/ 1 1 1 1 I.

23. (A) The first hot dog· came from Germany.


( B) Hot dogs o riginated in the United States.
(C) Some hot dogs are made from reindeer meat.
(D) Even countries like Finland have a food similar to hot
dogs.

24. (A) His work is too simple to keep him interested.


(B) He has no time to relax.
(C) He has a flat tire.
(D) He has no work to do.

25. (A) Tiffany is Stephanie's mother.


( B) Tiffany and Stephanie are sisters.
(C) Tiffany is older than Stephanie:
( D) Tiffany is younger than Stephanie.

26. (A) To find out how long it will take to repair the car.
(B) To find a different repairman.
(C) To find out what it will probably cost before the work is
done.
( D) To repair it himself.

27. (A) She watched TV last night instead of working on her


paper.
(B) She didn' t watch TV last night because she had to write a
paper.
(C) She wrote her paper last night while she was watching TV.
(D) She is writing a TV script.

28. (A) Franklin admired the deer's beauty from his bedroom
window.
( B) Franklin closed the door quickly.
(C) Franklin shot a deer with a rifle.
(D) Franklin took a photograph of a deer.

378
1 1 1 1 1 /PRACTICETEsT3/ 1 1 1 I. 1

29. (A) Start typing immediately.


(B) Have her paper typed by somebody else.
( C) Change her topic.
(D) Find a different typing service.

30. (A) Yolanda injured Anna.


(B) Yolanda had to run downtown last week.
(C) Yolanda went downtown to exercise.
(D) Yolanda met Anna downtown unexpectedly.

Go ON TO PART B

Part B

DIRECTIONS

In Part B, you will hear longer conversations. After each


conversation. you will be asked some questions. The conversations
and questions will be spoken just one time. They will not be written
out for you, so you will have to listen carefully in order to
understand and remember what the speaker says.
When you hear a question, read the four possible answers in you r
test book and decide which one would be the best answe r to the
question you have heard. Then, on your answer sheet, find the
number of the problem and fill in the space that corresponds to the
letter of the answer you have chosen.

31. (A) He was crazy.


(8) They thought he was dead.
(C) He had many broken bones.
(D) He fell out of a plane.

32. (A) On a plane.


(B) On television.
(C) On the ground.
(D) In a hospital.

379
1 .1

33. (A) He fell out of a plane.


(B) His two parachutes didn't open.
( C) He fell while walking.
(D) A parachute fell on him.

34. (A) He died.


(B) He jumped from a plane again.
( C) He broke his leg.
(D ) He went crazy.

35. (A) Dentist-patient. (C) Teacher-student.


(B) Doctor-patient. (D) Pharmacist-customer.

36. (A) In a few days.


(B) Before leaving the office.
(C) Very slowly.
(D) Soon enough.

37. (A) Some medicine.


(B) Some tests.
(C) Exhaling slowly.
(D) FiJling her lungs with air.

38. (A) She does not have enough air in her lungs.
(B) She's exhaling too slowly.
(C) She didn't do well in her tests.
(D) She has a little congestion.

Go ON TO PART C

380
1 1 1 1 1 /PRACTICETEST3/ 1 1 1 1 1

PartC

DIRECTIONS

In Part C, you will hear several talks. After each talk, you will be
asked some questions. The talks and questions will be spoken just
one time. They will not be written out for you, so you will have to
listen carefully in order to understand and remember what the
speaker says.
When you hear a question, read the four possible answers in your
test book and decide which one would be the best answer to the
question you have heard. Then, on your answer sheet, find the
number of the problem and fill in the space that corresponds to the
letter of the answer you have chosen.

39. (A) 2 (B) 5 (C) 3 (D) 7

40. (A) They attracted the attention of a private airplane.


(B) They ran out of gas.
(C) Some fishermen spotted them.
(D) Their families finally found them.

41. (A) They knew that they had run out of fuel.
(B) Their families bad reported them missing.
(C) They hadn't met the private airplane when it was due to
arrive.
(D) It was starting to get dark.

42. (A) 15 miles. (B) 7Yi miles. (C) 2 miles. (D) 5 miles.

43. (A) To plan a special diet for the patient to lose weight.
(B) To show someone how to read an x-ray.
(C) To get the patient to join an exercise class.
(D) To inform the patient about how to deal with his illness.

381
1 1 t 1 1 /PRACTICETEsT3/ 1 1 1 1 1

<14. (A) She talked with a c hiropractor.


(B) She read the x-rays.
(C) She did some back stretching exercises.
(D) She took some pain killers.

45. (A) Osteoarthritis. ( C) Pinched nerves.


(B) Curvature of the spine. (D) Muscle spasms.

46. (A) A painter. (C) An art critic.


(B) A museum guide. (D) A friend of Dali.

47. {A) Perfume. (C) Furniture.


(B) Leather goods. (D) Jewelry.

48. (A ) A to reador. (C) Columbus.


(B) Gala. (D) Lincoln.

49. (A) Landscapes.


(B) Seascapes.
(C) Hand craftsmanship.
(D) Surrealism.

50. {A) Toreadors. (C) Landscapes.


(B) Slave markets. (D) Limp watches.

STOP. THIS JS THE END OF THE LISTENING COMPREHENSION SECTION.


Go ON TO SECTION 2.

382
2 2 2 2 2 / PRACI1CE TEST J/ 2 2 2 2 Z

SECTION 2
STRUCTURE AND WRITIEN EXPRESSION
Time: 25 Minutes
40 Questions

Part A

DIRECTIONS

Questions 1-15 are incomplete sente nces. Beneath each sentence


you wi ll see four words o r phrases, marked (A), (B), (C), and (D).
Choose the one word or phrase that best completes the sente nce.
Then, o n your answer sheet, fi nd the number of the question and fill
in the sp ace that corresponds to the letter of the answer you have
chosen. Fill in the space so that the letter ins ide the oval canno t be
seen.

1. The attorney told his clien t that _ _ _ _ __


(A) they had little chance of winning the case
( B) the case was of a small chance to win
(C) it was ne1rly impossible to win him the case
(D) the case had a minimum chance to be won by him

2. One of the professor' s greatest attributes is _ _ _ __


(A) when he gives lectures
(B ) how in the ma nner that he lectures
( C) the way to give lectures
(D ) his ability to lecture

383
2 2 2' 2 2 / PRACTICE TEsT 3/ 2 -2 2 2 2

3 . The bank sen t a no tice to its cus to mers which contained

(A) a remembrance that interest rates were to raise the


following month
(B) a reminder that a raise in interest rates was the month
following
(C) to remember that the interest rates were going up next
mo nth
(D) a reminder that the interest rates would rise the followi ng
monch

4. was the day before yesterday.


(A) The Fran ce's Independence Day
(B) The day of the French independence
(C) French's Independence Day
(D) France's Independence Day

5. It was not until she had arrived home - - - - - remem-


bered her appointment with the doctor.
(A) when she
(B) that she
(C) and she
(D) she

6. George would certainly have attended the proceedings

(A) if he didn' t get a flat tire


( B) if the fiat tire hadn't happened
(C) had he not had a flat tire
(D) had the tire not flattened itself

7. received Jaw degrees as today.


(A) Never so many women have
(B ) Never have so many women
( C) The women aren't ever
(D) Women who have never

384
2 2 2 2 2 /PRACTICETEsT3/' 2 2 2 2 2 ~I

8. The students liked that professor's course because

(A) thr>re was few if any homework


(B) not a lot of homework
(C) of there wasn't a great amount of homework
(D) there was little or no homework

9. George he could improve his test scores, but he


did not have enough time to study.
(A) knew to
(B) knew how
( C) knew how that
(D) knew how to

10. he would have come to class.


(A) If Mike is able to finish his homework
(B) Would Mike be able to finish his homework
(C) If Mike could finish his homework
(D) If Mike had been able to finish his homework

11. Lee contributed fifty dollars, but he wishes he could contribute

(A) one other fifty dollars


(B) the same amount also
( C) another fifty
(D) more fifty dollars

12. The people at the party were worried about Janet because no
one was aware she had gone.
(A) where that
(B) ofwhere
( C) of the place where
(D) the place

385
..
2 2 2 2 2 /PRACTICETEST3/ 2 '2 2 2 2

13. Since he cha nged professions, Fred's yearly income h as

(A) ne a rly tripled


(B) got a lmost three times bigger
(C) almost grown by three times
(D) just about gone up three times

14. Nancy h '.!sn ·~ begun working on her Ph.D.

(A) still because she is yet


(H) yet<>:; a result she is still
('' ) ye. u.:.:ause she is still
(9) still wilile she is 2.lready

15. The director of this organizatio n must know------


(A) mo ney management, selling, a nd able to satisfy the stock-
holders
(B) how to manage money, selling his prod uct, and be able to
satisfy stockholders
(C) how to manage mont>y, sell his product, and satisfy the
stockholders
(D) money management. selling, the idea of being able to
satisfy the stockholde rs

JO ON TO PART B

PartB

Ia questio n:. l6-4r .ach sente~ce has four ur:derlincd words o r


. •i,1as;;.s. The fo ur u11 de1imc/ ;n:r,s o!· the senter.cc a re marked (A),
{ f!; :r:;, ar.d (D). ~- '1!:-itify the <;;..e :.a.ce;; . ..::1 we re x pr.~a:;e :h:.:t
• ' : :t be cha;1ged i:: ~rder for the St'm.;: ~r tc be correct. : !"!:!:i. on
~ • •r :!!!S\"''I sheet, f..nd the ni ·. mlJ .. , "t:estior. :.:nJ till in the
...! C.:•~ ...· , . ...,l ·..... .·tespnnds to th'! ,e:,. . •
2 2 2 2 2 /PRACTICETEsT3/ 2 2 2 2 2

16. She wishes that we didn't send her the candy yesterday
A B
because she's on a diet.
c 0

17. They are planning on attending the convention next month,


A B C
and so I am.
0

18. Today was such beautiful day that I couldn't bring myself
A B
to complete all my chores.
c 0

19. While they were away at the beach, they a1Jow1!d


A B
their neighbors use their barbeque grill.
c 0

20. The artist tried stimulate interest in painting by taking his


A B C
students to the museums.
0

21. Mumps are a very common disease which usually affects


A B C 0
children.

22. Nancy said that she went to the supermarket


A B C
before coming home.
0

387
2 2 2 2 2 / PRACTICE TEST 3 / 2 2 2 2 2

23. Be fore she moved here, Arlene had been president


~-
A~ ~8-

of the organization since four years.


C D

c D

25 . The athlete, together with his coach and several relatives,


~-A~ f3 C
are traveling to the Olympic Games.
D

26. Professor Duncan teaches both anthropology as well as


A B C
sociology each fall.
D

27. My brother is in California on vacation, but I wish he was here


A B C
so that he could help me repair my car.
D

28. I certainly appreciate him telling us about the delay in


A B C
delivering the materials because we had planned to begin work
D
tomorrow.

29. The chemistry instructor explained the experiment in


A
such of a way that it was easily understood.
B C D

388
2 2 2 2 2 / PRACTICE TEsT3/ 2 2 2 2 2

30. Rudo lph Nureyev has become one of the greatest da ncer that
A B C
the ba llet world has ever known.
0

31. He has less friends in his classes now than he had last year.
A B C 0

32. The town we visited was a four-days journey from our hote l, so
A B
we took the train instead of the bus.
~-
c
- -0 -

33. The influence of the nation's literature, art, and science


A -- B-
have captured widespread attention.
c 0

34. The leader emphasized the need fo r justice and equa lity
A B
between his people.
c 0

35. Many of the popula tion in the rural a reas is


A B
com posed of manual laborers.
C D

36. Several people have apparent tried to cha nge


A B
the man 's mind, but he re fuses to listen.
c 0

37. Keith is one of the most inte lligent boys of the science class.
A B C 0

389
2 2 2 2 2 / PRACTICETEST3/ 2 2 2 2 2

38. The girl~: were sorry to had missed the singers when they
A B C
arrived a! the airport.
D

39. When K~ith visited Alaska, he lived in~ igloo in the winter
A B
mo nths as well as in the spring.
C D

40. The harder he tried, the worst he danced before the


-A- - -B- --C-
large audience.
D

STOP. THIS IS THE END OF THE STRUCfURE AND WRITTEN EXPRES-


SION SECTION. IF YOU FINISH BEFORE TIME IS UP, CHECK YOUR WORK
ON PARTS A AND B OF THIS SECTION ONLY. D o NOT WORK ON ANY
OTHER SECTION OF THE TEST.

390
3 3 3 3 3 I P~CTICE TEST 3I 3 3 3 3 3

SECTION 3
READING COMPREHENSION
Time: 55 Minutes
50 Questions

DIRECTIONS

In this section, you will read a number of passages. Each one is


rollowed by approximately ten questions about it. For questions
1-50, choose the one best answer. (A), (B), (C), or (0), to each
question. Then, find the number of the question on your answer
sheet, and fill in the space that corresponds to the letter of the
answer you have chosen. Answer all questions following a passage
o n the basis of what is stated or implied in that passage.

Questions 1 through I 0 are based on the following passage.

Elizabeth Blackwell was born in England in 1821 and


emigrated to New York City when she was ten years old.
One day she decided that she wanted to become a doctor.
That was nearly impossible for a woman in the middle of the
(5) nineteenth century. After writing many letters seeking
admission to medical schools, she was finally accepted by a
doctor in Philadelphia. So determined was she that s he
taught school and gave music lessons to earn money for her
tuition.
(10) In 1849, after graduation from medical school, she de-
cided to further her education in Paris. She wanted to be a
surgeon, but a serious eye infection forced her to abandon
the idea.
Upon returning to the United States, she found it difficult
(15) to start her own practice because she was a woman. By 1857.
Elizabeth and her sister, also a doctor, along with another
female doctor, managed to open a new hospital, the first for
women and children. Besides being the first female physi-

391
3 3 3 3 3 /PRACTICE TEST 3 / 3 3 3 3 3

cian in the United States and fou nding her own hospital, she
(20) also established the first medical school for women.

1. Why couldn't Elizabeth Blackwell real ize her dream of becom-


ing a surgeon?
(A) She could n't get admitted to medical school.
(B) She decided to further her educatio n in Pa ris.
( C) A serious eye infection halted her quest.
(D) It was difficult fo r her to start a practice in the United
States.

2. What ma in obstacle almost destroyed Elizabeth's chances for


becoming a doctor?
(A) She was a woman.
( B) She wrote too many letters.
(C) She couldn't graduate from medical school.
(D) She couldn 't establish her hospital.

3. How many years elapsed between her graduation from medical


school and the opening of her hospital?
(A) 8 (B) 10 (C) 19 (D) 36

4. All of the following are "firsts" in the life of Elizabeth Blackwell


EXCEPT
(A) she became the first female physician in the United States
(B) she was the first woman surgeon in the United States
(C) she and severa l other women founded the first hospital for
women and children
(D) she established the first medical school for wome n

5. How old was E lizabeth Blackwe ll when she graduated from


medical school?
(A) 10 (B) 21 (C) 28 (D) 36

6. The word "abandon" in line 12 is closest in meaning to


(A) undertake (C) continue
(B) give up (D) look into

392
3 3 3 3 3 / PRACTlCETEsT3/ 3 3 3 3 3

7. What is the main idea of this passage?


(A) Elizabeth Blackwell overcame serious obstacles to become
the first woman doctor in the United States.
(B) Elizabeth Blackwell had to abandon her plans to become a
doctor because of an eyt!'infection.
(C) Elizabeth Blackwell even taught music to pay for her
medical studies.
(D) Elizabeth Blackwell founded the first medical school for
wo men.

8. The word "founding" in line 19 means most nea rly the same as
(A) locating (C) establishing
(B) looking for (D) buying

9. Why was it nearly impossible for Elizabeth Blackwell to get into


medical school?
(A ) She had a serious eye infection.
(B) She had iittle or no money to pay tuition.
( C) She wamed to be part of a profession that no woman had
ever entered before.
(D) Her fa mily didn't want her to be a doctor.

lO. The reason Elizabeth Blackwell could not become a surgeon is


explained in lines
(A) 4-5 (B) 8-9 (C) 11-13 (D) 14-15

Questions 11 through 21 are based on the following passage.

Glands manufacture and secrete necessary substances.


Exocrine glands secrete their products through ducts, but
endocrine glands, or ductless glands, release their products
directly into the bloodstream.
(5) Orte important endocrine gland is the thyroid gland. It is
in the neck and has two lobes. one on each side of the
windpipe. The thyroid gland collects iodine from the blood
and produces thyroxine, an important hormone, which it

393
3 3 3, 3 3 / PRACTICETEST3/ 3 3 3 3 3

stores in an inactive ·fo rm. When thyroxine is needed by the


(10) body, the thyroid gland secretes it directly into the blood-
stream. Thyroxine is combined in the body cells with other
chemicals and affects many functions of the body.
The thyroid gland may be underactive o r overactive,
resulting in problems. An underactive thyroid causes hypo-
(15) thyroidism, while an overactive o ne causes hyperthyroidism.
The former problem, called myxedema in adults and cretin-
ism in children. causes the growth process to slow down. A
cretin's body and mind do not grow tc, •.;;.:;ir full potential.
Hyperthyroidism, on the other hand, results in excreme
(20) nervousness, an increase in heart action, and other prnb-
lems.
Either hypot hyroidism or hype rthyroidism may result in
goiter, or an enlarged thyroid gland. A goiter will appear
when the body is not getting enough iodine. Goiter is less
(25) common today, since most people use iodized salt.

11. The thyroid gland is called an endocrine gland because it


(A) has ducts
(B) has lobes
(C) secretes directly into the bloodstream
(D) is located in the neck

12. The word "it" in line 8 refers to


(A) thyroxine (C) iodine
( B) blood (D) thyroid gland

13. A cretin is
(A) a child with hyperthyroidism
(B) an adult with an underperforming thyroid gland
(C) a young person with hypothyroidism
(D) an extremely irritable child

394
3 3 3 3 3 /PRACTICETEsT3/ 3 3 3 3 3

14. Which of the following is a probable result of myxedema?


(A) Sluggishness
(B) Hyperactivity
( C) Overproduction of thyroxine
(D) Perspiration

15. The word "forme;" in line 16 refers to


(A) hypothyrc 1dism (C) hyperthyroidism
(8) overactive thyroid (D) secretion

16. A goiter is
(A) a person with myxedema
(B) a swollen thyroid gland
(C) an underactive thyroid gland
(D) a chemical

17. Exocrine and endocrine glands are distinguished from each


other by whether they
(A) secrete through ducts or without ducts
(B) cause hyperthyroidism or hypothyroidism
(C) cause myxedema or cretinism
(D) result in an enlarged or shrunken goiter

18. In line l , the word "secrete" is closest in meaning to


(A) indiscernible (C) display
(B) emit (D) absorb

19. If a thyroid is not working enough, the illness is known as


(A) hyperthyroidism (C) excretion
( B) hyperactivity (D) hypothyroidism

20. The main idea of the passage is


(A) how glands work
( B) the function and illnesses of the thyroid gland
(C) secretion with and without glands
(D) the illnesses of an overactive thyroid gland

395
3 3 l_ 3 3 / PRACTICE TEST 3 / 3 3 3 3 3 J
21. The function of the thyroid gland is described in lines
(A) 2-4 ( B) 7-9 (C) 13- 15 (D) 22-25

Questions 22 through 30 are based on the follo wing passage.

A recent investigation by scientists at the U.S. Geological


Survey shows that strange animal behavior migh t help
predict earthquakes. Investigators found such occurre nces
within a ten-kilometer radius c f the epice;itcr of a fairly
(5) recent quake. Some birds screeched and flew about wildly;
dogs yelped and ran around uncontrollably.
Scientists believe tha t animals can perceive environ men-
tal changes several hours or even days before the mishap.
Animals were no ted as being restless fo r several weeks
(10) before a Tashkent, Uzbekistan, .::arthq uake. An hour before
the disaster, domestic animals refused to go indoors, and
dogs howled and barked furiously. In 1960, an earthq uake
struck Agadir in Morocco. Survivors recall that stray ani-
mals, including dogs. were seen streaming cut of town
(15 ) before the earthquake. In a safari zoo near San Francisco,
lla mas would not <>at the evening before a 1979 quake, and
they ran a ro und wildly a ll night.
U nusual animal behavior preceding ear:hquakes has
been no ted fo r centuries. British Admiral Robert Fitzroy
(20) reported huge flocks of screaming seabirds over Concep-
cion, C hile, in !835. An hour and a half later, dogs were
seen ftee:ng, and ten minutes later the town was destroyed.
Similar stories of chickens running around in apparent
states of panic, horses trembling, and dogs barking inces-
(25) santly were recorded th rougho ut the eighteenth and nine-
teenth centuries by survivors of e<>n hquake destruction in
Ind ia, Yugoslavia, Peru, Mexico, and the U nited S t:ites.
In 1976, after monitoring bizarre auimal behavior. the
Chinese predicted a devastating earthquake. Altho ug h
(30) hundreds of thousands of people were killed, the govern-
ment was able to evacuate millions of o ther people and thus
keep the death toll at a lower level.
396
3 3 3 3 3 / PRACTICETEsT3/ 3 3 3 3 3

22. Wha t prediction may be made by observing animal behavio r?


(A) An impending earthquake
( B) The number of people who will die
(C) The ten-kilometer radius from the epicenter
( D) The fact that ::in earthquake has occurred

23. The autho r implies that animals are aware of an impending


earthquake because
(A) of their superior intelligence
( B) they have certain instinctive abilities to perceive that
huma ns do not possess
( C) they are generally closer to the epicente r than the human
observers
(D) they react to o ther animal behavior

24. T he word "evacuate" in line 31 is closest in meaning to


(A) remove (B) exile (C) destroy (D) emaciate

25. All of the following statements are true EXCEPT


(A) some animals may be able to sense an approaching
earthquake
(B) by observi ng animal behavior scientists perhaps can pre-
dict earthquakes
(C) the Chinese have successfully predicted an earthquake and
saved many lives
(D) o nly dogs and horses seem to P'JSSess the special percep-
tion that allows them to predict earthquakes

26. In line 4, the word "epicenter" is nearest in meaning to


(A) stratosphere (C) periphery
(B) contour (D) core

397
3 3 3' 3 3 /PRACTICETusT3/ 3 3 3 3 3

27. The passage implies that if scientists can accurately predict


earthquakes, there will be
(A) fewer animals going crazy
(B) a lower death rate
(C) fewe r people evacuated
(D) fewer environmental changes

28. In line 29, " devastating" means most nearly the same as
(A) destructive (C) intense
(B) voracious (D) forthcoming

29. The main idea of thjs passage is that


(A) earthquakes can be prevented by observing animal behav-
ior
(B) scientists can interpret animal behavior
( C) observing animal behavior can help people prepare for
earthquakes
(D) people need to prepare animals for earthquakes

30. Where in the reading is it explained that the phenomena of


animals' reacting to earthquakes has been reported for hun-
dreds of years?
(A) Lines 3-5 (C) Lines 18-19
(B) Lines 7-8 (D) Lines 29-32

Questions 31 through 39 are based on the following passage.

As far back as 700 B.c., people have talked about children


being cared for by wolves. Romulus and Remus, the
legendary twin founders of Rome, were purported to have
been cared for by wolves. According to legend, Mars
(5) fathered the two boys. As a result, a relative of their mother
imprisoned her and ordered that the boys be drowned in the
Tiber River. However, a she-wolf saved them from this
horrible fate and took them back to her lair to care for them.
Legend has it that when a she-wolf loses her litter, she seeks

398
3 3 3 3 3 / PRACTICE TEST 3 / 3 3 3 3 3

(10) a human child to take its place.


This seemingly preposterous idea did not become cred-
ible until the late nineteenth century when a French doctor
act ua lly found a naked ten-year-old boy wa ndering in the
woods. He did no t walk erect, could no t speak intelligibly,
(15) nor could he relate to people. He o nly growled and stared at
them. Finally, the doctor won the boy's confidence and
began to work with him. After many long years of devoted
and patient instruction, the doctor was able to get the boy to
clothe and feed himself, recognize and utter a number of
(20) words, and write le tters and fo rm words.

31. The French doctor fo und the boy


(A) wandering in the woods
( B) at his doorstep
(C) growling at him
(D) speaking intelligibly

32. In line 9, the word " litter" means most nearly the same as
(A) garbage (B) master (C) offspring (D) hair

33. The doctor was able to work with the boy because
(A) the boy was highly intelligent
( B) the boy trusted him
(C) the boy liked to dress up
(D) the boy was dedicated and patient

34. The word "utter" in line 19 is nearest in meaning to


(A) absolute (B) speak (C) scream (D) read

399
3 3 3 ' 3 3 /PRACTICETusT3/ 3 3 3 3 3 ]

35. All of the following statements are true EXCEPT


(A) she-wolves have been said to substitute human children for
their lost litters
(B) ;:xamples of wolves' caring for human ; hildren can be
found only in the nineteenth century
(C) the French doctor succeeded in domesticating the boy
somewhat
(D) the ~1 .)ung boy never was able to speak perfectly

36. T?·:e word " preposterous" in line 11 is closest in meaning to


(A) dedicated (C) wonderful
(B) scientific (D) absurd

37. The main idea of this passage is that according to legend


(A) children who are raised by wolves can be rehabilitated
(B ) she-wolves replace their dead offspring with human chil-
dren
(C ) Romulus and Remus were cared for by a she-wolf
(D) a French doctor saved Romulus and Remus from drown-
ing

38. According to the legend, Romulus and Remus were


(A ) found abandoned in Rome
( B) the founders of Rome
(C) discovered by a French doctor
(D) drowned in the Tiber River in 700 B.C.

39. Where in the passage is it stated that, according to legend,


Romulus and Remus founded Rome?
(A) Lines 2-3 (C) Lines 9-10
(B) Lines5-7 (D) Lines 11-13

400
3 3 3 3 3 / PRAcnCETEsT3/ . 3 3 3 3 3]

Questions 40 through 50 are based on the following passage.

Vibrio parahaemolyticus is a bacterial organism that has


been isolated from sea water, shellfish, finfish, plankton,
and salt springs. It has been a major cause of food poisoning
in Japan, compelling the J apanese to do several studies on
(5) it. They have confirmed the presence of V. parahaemolyticus
in the north and central Pacific, with the highest abundance
in inshore waters, particularly in or near large harbors.
A man named Nishio studied the relationship between
the chloride content of sea water and the season<.! distribu-
(10) tion of V. parahaemolyticus and concluded that while the
isolation of the organism was independent of the sodium
chloride content, the distribution of the bacteria in sea
water was dependent on the water temperature. In fact, it
has been isolated in high frequencies during summer, from
(15) June to September, but was not isolated with th~ same
frequency in winter.
Within four or five days after eating contaminated foods,
a person will begin to experience diarrhea, the most
common symptom; this will very often be accompanied by
(20) stomach cramps, nausea, and vomiting. Headache and
fever, with or without chills, may also be experienced.

40. Which of the following locations would be most likely to have a


high concentration of Vibrio parahaemolyticus?
(A) A bay
(B) Asea
( C) The middle of the ocean
(D) Sediment

41. The word " inshore" in line 7 is closest in meaning to


(A) near the coast (C) active
(B) deep (D) cold

401
3 3 3 3 3 / PRACTICETEST3/ 3 3 3 3 3

42. The word " it" in line 13 refers to


(A) Vibrio parahaemolyticus
(B) sea water
(C) sodium chloride content
(D) water temperature

43. The safest time for eating seafood in the north Pacific is
probably
(A) August (C) July
( B) November (D) September

44. The most common symptom of V. parahaemofyticus poisoning is


(A) nausea (C) vomiting
(B ) diarrhea (D) headache and fever

45. The word " this" in line 19 refers to


(A) contaminated foods (C) a person
(B) symptoms (D ) diarrhea

46. The incubation period for this illness is


(A) 2 to 3 days (C) 4 to 5 days
( B) 3 to 4 hours (D) several mo nths

47. In li ne 17, "contaminated " is closest in meaning to


(A) ocean (B) tainted (C) salty (D) cooked

48. Nishic's stu dy showed that


(A) the presence of V. paralzaemolyticus was dependent on
neither the salt content nor the water tempera ture
(B) the presence of V. parahaemolyucus was dependent only o n
the salt content
(C) the presence of V. parahaem olyticus was independent of
both the water temperature and the salt content
(D) the presence of V. parahaem olyticus was dependent on the
water temperature

402
3 3 3 3 3 / PRACTICE TEST3/. 3 3 3 3 3

49. The word "cramps" in line 20 means most nearly t he same as


(A) noises (C) severe pain
( B) toxicity (D) high temperature

50. The word " isolation" in line 11 is closest in meaning to


(A) conjunction (C) separation
( B) impurity (D) discovery

STOP. THIS IS THE END OF THE EXAMINATJON. I F YOU FINISH BEFORE


TIME IS UP, CHECK YOUR WORK IN THIS SECTION ONLY. DO NOT WORK
ON ANY OTHER SECTION OF THE TEST.

403
1 1 f 1 1 / PRACTICE T EST 4 / 1 1 1 1 1

PRACTICE TEST 4

SECTION l
LISTENING COMPREHENSION
Time: 'Approximately 30 Minutes
50 Questions

Sectio n 1 has three parts. Each part has its own set of directions.
Do not take notes while listening or ma ke any marks on the test
pages. Notetaking. underlining, o r crossing o ut will be considered
cheating on the actual TOEFL exam. Answer the questions
followi ng the conversatio ns or talks based on what the speakers
have stated or implied.
Fo r Practice Test 4, insert your Listening Compre hensio n cas-
sette in your tape player. On the act ua l TOEFL, you will be give n
extra time to go o n to the next page when you finish ..1 page in the
Listening Comprehe nsion section. In the following test. however,
you will have o nly the 12 seconds given after each question . Turn
the page as soon as you have marked your answer. Start the cassette
now.

Part A

DIRECTIONS

In Part A, you will hear short conversations between two


speakers. At the e nd of e ach conve rsation, a third voice will ask a
question about what was said . The question will be SjX>ken just o ne
time. After yo u hear a conversation a nd the question ibo ut it. read
the fo ur possible answers and decide which one would be the best
answer 10 the question you have heard. Then. on your answer sheet,
find the number of the problem a nd mark your answer.

-W4
[ J 1 1 1 1 / PRACTICE TusT4/. 1 1 1 1 1

i . (A) Something happened to her car.


( B) She was broke and couldn't afford the bus.
IC) She got up too late to catch the bus.
(D) Her car got stuck in the driveway.

2. (A) She doesn' t like other people brushing her clothes.


(B) She doesn't like to drink.
(C) She doesn't like to knit.
(0) She doesn't like being snubbed at a party.

J. (A) He will buy the car as soon as I-.e gets the money.
(B) His friend is buying the car for him.
(C) He can't afford to buy a new car.
(D) He has already made the down payment on the car.

' (A) She had to fly out of town.


(B) She's sick.
(C) She said that s he'd come later.
(I:) She decided to stay home.

5 . (A) 15 (B) 50 (C) 85 (D) 100

o. (A) The man doesn't have to study a foreign language.


( B) The man just received an " A" on his test.
(C) T he man's adviser gave him some good advice.
(D) He doesn 't have to take the final exam.

7. (Aj Rusty will lose his car because he hasn't made the
payments.
(B) The finance company is returning Rusty's car.
(C) Rusty has a broken finger from falling on the pavement
behind his car.
(D) Rusty's car is being repaired.

405
1 1 1 1 1 / PRACTICE TEST 4 / 1 1 1 1 1

8. (A) Reviewed a previous lesson.


( 8 ) Presented new material.
(C) Tested the students.
(D) Made the students wri te in class.

9. (A) The woman is getting another job.


( B) The woman is disappointed a t not getting the job.
(C) The woman 's boss is letting her have a better job.
( 0) The woman 's job is much better than she had expected.

10. (A) They are pleased. ( C) They are undecided.


(B) They dread it. (0) They are frustrated .

11. (A) The bank closed before the woman could deposit her
money.
( B) If the woman hurries. she'll get to the bank before closing
time.
( C) The woman has to take some money o ut of the bank 5efore
it closes.
(0) The bank is closing the woman 's accoun t because she
hasn't deposited any money.

12. (A) They were total strangers.


(B) He knew them only slightly.
( C) He knew them very well.
(0) He wasn·t sure whether he knew them o r not.

13. (A) Cut the suga r cubes into smaller pieces.


(B) Put sugar in his coffee.
(C) Reduce the amount of sugar he ingests.
(0 ) Eat more sugar.

14. (A) They were pulled through the wreckage.


( B) They were pulling each other through the wreckage.
( C) None will survive.
~D) All will probably survive.

406
1 1 l 1 1 / PRACTICE TEsT4/ 1 1 1 1 1

15. (A) The re were so many tickets left that they had Lo sell them
again the next day.
(B) Not many showed up to purchase tickets on opening day.
(C) There were no tickets left by noon of the opening day.
(D) A fe w tickets we re left for the afternoon of opening day.

16. (A) Knowing th~ t he lacked experience, he still applied.


(B) Even tho ugh he was experie nced, he didn't apply fo r the
job.
(C) He was highly qualified for the job, so he applied.
(D) H e didn ·t have much experience working in the fields.

17. (A) It could not be solved by anyone.


(B) Everyone knew how to solve it.
(C) Gary was the only one who couldn ·t solve it.
(D) O nly Gary could solve it.

18. (A) The people thanked her for her response.


( B) T he people were grateful because she had requested the
information.
(C) She was happy with the response to her first request.
(D) She responded gratefully to their request.

19. (A) She can't attend the meeting because she has too much
homewo rk.
( B) She ccmpleted her homewo rk early so that she could
atte nd the meeting.
(C) Although she has homework due tomorrow, she plans to
go to the meeting.
(D) She refuses to attend this class because of the homework.

20. (A) Gil prefers that the woman wait fo r him.


( B) Gil is happy !Jccause the woman didn't wait fo r him.
(C) The 'Noman is angry because Gil left before she arrived.
{D ) Gil doesn't want the woman to wait fo r him.

407
1 1 l l l / PRACTICE TEsT 4/ ·1

21. (A) Jogging. (C) Taking a stroll.


(B) At the store. (D ) Getting a newspaper

22. (A) The woman will go home for dinner.


(B) The woman won't go to the concert.
(C) The man anct woman will eat together.
(D) Both of them will go home before going to the concert.

23. (A) Oscar pays his bills ahead of time.


(B) Oscar has decided to get a loan to pay his bills.
(C) Oscar has too many expenses and can' t save any money.
(D) Oscar's wife will have to go to work.

24. (A) His keys are lost forever.


(B) He expects to find his keys soon.
(C) His keys were lost, but now he has found them.
(D) Someone showed up with his keys soon after he had lost
them.

25. (A) They attended the concert even though the tickets were
expensive.
(B) They wanted to attend the concert, but the tickets were
sold out.
(C) The tickets were so inexpensive that they attended the
concert.
(D) They couldn't afford the tickets for the concert.

26. (A) Use a different type of viewing device.


(B) Sit down.
(C) Stand up.
(D) Stop raising the sign.

408
1 /PRACTICE TEST 4/ 1 1 1 1 1

27. (A) Melanie didn't wear the coat because she doesn't like red.
(B) Melanie is allergic to wool.
(C) Melanie wore the coat but broke out in a rash.
(D) Melanie couldn't wear the coat because she was in a rush.

28. {A) H e'd like to have a steak and salad now.


(B) He thinks he'll run out and buy a steak and salad right now.
( C) He just ate a steak and salad.
(D) He's eating a steak and salad at the moment.

29. {A) She usually goes to the football games.


( B) She hasn't seen a football game for a long time.
(C) She doesn't like football.
(D) She usually doesn't go to football games.

30. {A) The agent has sold no policies this week.


(B) The agent has sold o nly o ne policy this week.
(C) The agent hasn't sold too many policies this week.
(D) Last week, the agent sold more policies than anybody else.

Go ON TO PART B

Part B

DIRECTIONS

In Part B, you will hear longer conversations. After each


conversation, you will be asked some questions. The conversatio ns
and questions will be spoken just o ne time. They will not be written
out for you, so you will have to listen carefully in order to
understand and remember what the speaker says.
When you hear a question, read the four possible answers in your
test book and decide which one would be the best answer to the
question you have heard. Then, on your answer sheet, find the
number of the problem and fill in the space that corresponds to the
letter of the answer you have chosen.

409
1 1 i 1 1 / PRACTlCETEST4/ .1 1 1 1 1

31. (A) Department store. (C) Produ ce market.


( B ) Supermarket. ( D ) Variety store.

32. (A) Tuna fish. (B) Eggs. (C) Bleach. (D) Detergent.

33. (A) It is a no-frills sto re.


( B) The fresh food looked appetizing.
(C) The lines are sho rter.
(D) There is a wide selection.

34. (A) The man did not enjoy the sto re.
(B) Nothing was on sale.
(C) He spent mo re than $50.
(D) He found the food inexpensive.

35. (A) San Juan. (C) Miami.


(B) Venus. (D) Po rt-o-ca ll.

36. ( A) Morison. (C) Norrison.


(B) Mowrison. (D) Mo rrison.

37. (A) Cash. ( C) Personal check.


(B) Credit card. (D ) Traveler's checks.

38. (A) In three days. (C) March 15.


(B) March 27. (D) J une 20.

Go ON TO PART C

410
1 1 1 1 1 / PRACTICE TEST 4/ 1 1 1 1 1

PartC

DIRECTIONS

In Part C, you will hear several talks. After each talk, you will be
as ked some questions. The talks and questions will be spoken just
one time. They will not be written o ut for you, so yo u will have to
listen carefully in order to understand and remember what the
speaker says.
When you hear a question, read the fo ur possible answers in your
test book and decide which one would be the best answer to the
question you have heard. Then. on you r answer sheet, find the
number of the problem a nd fill in the space that corresponds to the
!ettcr of the answer you have chosen.

39. (A) The high cost of gasoline.


(B) Overcongestion of university areas.
(C) Dangerous driving conditions.
(D) Police roadblocks.

40. (A) State law only.


(B) City law only.
(C) Natural law.
(D) City and state law.

41. (A) Roller skating in the streets is o nly a local problem.


(B) Skaters are creating problems for motorists.
(C) Police will ticket violators.
(D) The problem is most common in college and university
areas.

42. (A) New political ways.


(B) New methods of fishing.
( C) New means of water travel.
(D) How to trap animals.

411
1 1 'J. 1 1 / PRACTICE TEsT4/ · J 1 1 1 1

43. (A) They were plentiful in England.


(B) They grew only in certain sections of the country.
(C) Tliey were preferred raw.
(D) They did not exist in England.

44. (A) By canoe.


(B) By blazing trails through the forest.
(C) By toboggan and snowshoes.
(D) On animals.

45. (A) Com.


(B) Domesticated animals.
(C) Building shelters.
(D) Trapping animals.

46. (A) The settlers were well prepared for the hardships that they
would encounter.
(B) The new settlers evidently found the winters severe.
(C) The Indians taught the settlers how to build canoes.
(D) The settlers brought tools and weapons to the New World.

47. (A) 6:45 in Baton Rouge.


(B) 1:45 in Atlanta.
(C) 1:45 in Dallas.
(D) 2:45 in Dallas.

48. (A) Smoking cigars.


tn) Drinking whiskey.
( C) Smoking a pipe.
(D) Smoking cigarettes.

49. (A) 3242 (B) 3224 (C) 2334 (D) 3442

50. (A) 1:45 P.M. (B) 12 midnight. (C) 1:45 A.M. (D) 6:45 P.M.

STOP. THIS IS THE END OF THE LISTENING COMPREHENSION SECTION.


Go ON TO SECTION 2.

412
2 2 2 2 2 / PRACTICE TusT 4/ ·2 2 2 2 2

SECTION 2
STRUCTURE AND WRITIEN EXPRESSION
Time: 25 1\-;:inutes
40 Ques tions

Part A

DIRECTIONS

Questions 1-15 are incomplete sente nces. Beneath each sentence


you will see four words o r phrases, marked (A), (B), (C), and ( 0 ).
Choose the one word or phrase that best completes the sente nce.
Then, on your answer sheet, find the number of the question and fill
in the space that corresponds to the letter of the a nswer you have
chosen. Fill in the space so that the letter inside the oval cannot be
seen.

I. The cyclist he crossed the main street.


(A) looked with caution after
( B) had looked cautiously before
(C) was looked cautious when
( 0) looks cautious when

2. Here _ _ _ _ _ notebook and report that I promised you


last week.
(A) is the
(B) are the
(C) was the
(0 ) has been a

41 3
2 2 '2 2 2 /PRAcnCE TEST 4 / · 2 2 2 2 2

3. Neither Jane nor her brothers _ _ _ __ a consent form for


tomorrow's field trip.
(A) need
(B) needs
( C) is needing
(D) has need

4. Cuba is _sugar-growing areas in the world.


(A) one of the larger
( B) one of largest
(C) one of the largest
(D) largest

5. The skiers would rather _ _ _ _ _ through the mountains


than go by bus.
(A) to travel on train
(B) traveled by train
(C) travel by train
(D) traveling by the train

6. That magnificent _ _ _ _ _ temple was constructed by the


Chinese.
(A) eight-centuries-old
(B) eight-century's-old
(C) old-eight-centuries
(D) eight-century-old

7. There were two small rooms in the beach house, - - - - -


served as a kitchen.
(A ) the smaller of which
(B ) the smallest of which
(C) the smaller of them
(D) smallest of that

414
2 2 2 2 2 / PRACTICE TEST 4/ .2 2 2 2 2

8. Pio neer men and women endured terrible hardships, and

(A) so do their child re n


( B) neither did the children
( C) also the childs
(D) so d id their childre n

9. Last year, Matt e a r n e d - - - - - his brother, who has a


better position.
(A) twice as much as
(B) twice mo re than
( C) twice as many as
(D) twice as mo re as

10. he wo uld have been able to pass the exam.


(A) If he studied mo re
( B) If he were studying to a greater degree
(C) Studying more
(D ) Had he studie d more

11. Mr. Duncan does not know _ _ _ _ _ the lawn mower after
they had finished using it.
(A) where did they put
(B) where they did put
(C) where they put
(D ) whe re to put

12. The facilities o f the o lder hospital _ _ _ __


(A) is as good o r better than the new hospital
( B) ar~ as good o r better that the new hospital
(C) are as good as or better than the new hospital
(D) are as good as o r better than those of the new hospital

415
~2 2 2 2 / PRAcnCETEsT4/ 2 2 2 2 2

13. Our fiighc from Amsterdam to London was del 1y1;d


_ _ __ _ the hec.vy fog.
(A) because of
(B) because
(C) on account
(D) as result

14. The teacher suggested chac her students _ _ _ _ _ experi-


ences with ESP.
(A) write a composition on cheir
( B) to write composition about the
(C) wrote some compositions of his or her
(D) had writte n any compositions for his

15. Of the two new teachers, one is experienced an d - - - - -


(A) the others a re not
( B) another is inexperienced
(C) che oth.!r is not
(D) other lacks experience

Go ON TO PART B

Part B

DIRECTIONS

In quescions 16-40, each sentence has fo ur underlined words or


phrases. The four underlined parts of the sentence are marked (A ),
(B), (C), and (0 ). Identify the one underlined word or phrase tha t
must be changed in order for the sencence to be correct. Then. on
your answer sheet, find the number c·f the question and fill in the
space that corresponds to the letter of the answer you have chosen.

16. While searching for the wreckage of~ unidentified aircraft,


A B C
the Coast Guard enco untered severe squalls at sea.
D

416
2 2 2 2 2 / PRACTICETEsT4/ 2 2 2 2 2

l 7. Although a number of police officers was guard ing the


A
price less treasures in the museum, the director worried that
B
someone would try to steal them.
c 0

18. Since it was so difficult fo r American Indians to negotiate a


A B
peace treaty or declare war in their native language, they used
c
a universal understood fo rm of sign language.
0

19. Louis Braille designed a form of communication

enabling people to convey and preserve their thoughts


A B
to incorpora te a series of dots which were read by the finger
C D
tips.

20. While ve rbalization is the most common form of language in


A
existence, humans make use of many others systems and
B C
techniques to express their thoughts a nd feelings.
0

21. The need for a well-rounded education was an idea


A B
espoused by the G reeks in time of Socrates.
c 0

41 7
2 2 2 2 2 / PRACTICE TEST4/ 2 2 2 2 2

22. Writers and media perso nne l sell the irselves best
A B
by the impression given in the ir verbal expression.
C D

23. In the spirit of the naturalist writers, that a utho r's work
A B C
portrays man's struggle for surviving.
D

24. Stephen Crane's story is ~clinical po rtrayal


A B
of man as an animal trapped by the fea r and hunge r.
C D

25. The ir silly, whiny conversation on a child level was meant


A
to create tension and heighten Nancy's fears and anxiety.
B C 0

26. For a long time, this officials have been known throughout the
A B
country as political bosses and law enfo rcers.
c 0

27. Nora hardly never misses an opportunity ~ in the tennis


A B C 0
tournaments.

28. Air pollution, together with littering, are causing


A B
many problems in our large, industrial cities today.
c 0

29. Because of the severe snow sto rm and the road blocks, the air
A B
force dropped food and medical supplies close the city.
c 0

418
2 2 2 2 2 /PRACTICE TEsT4/ 2 2 2 2 2

30. Hummingbirds are the only birds capable to fly backward


A B
as well as forward, up, and down.
C D

31. The news of the president's treaty negotiations with the


A
foreign government were received with mixed emotions by the
B C
citizens of both governments.
D

32. Angie's bilingual ability and previous experience were the


A
qua lities that which helped her get the job over all
B C
the other candidates.
D

33. Joel giving up smoking has caused him to gain weight and
A B C
become irritable with his acquaintances.
D

34. They asked me what did happen last night, but I was unable to
A B C
tell them.
D

35. The test administrator ordered we not to open our books until
A B C
he told us to do so.
D

36. Our new neighbors had been living in Arizona since ten years
A B C
before moving to their present house.
D

419
2 2 2 ' 2 2 / PRACTICETEsT 4 / 2 2 2 2 2

37. I would of attended the meeting of the planning commiccee


A B
last week, but I nad to deliver a speech at a convention.
C D

38. We are suppose to read all of chapter seven and answer the
A B C
questions for tomo rrow's class.
D

39. The explanation that our instructor gave us was different


A B
than the one yours gave you.
C D

40. In the sixteenth centu ry, Spain became involved in foreign


A · B
wars with several other Europea n countries a nd could not find
c
the means of fi nance the battles that e nsued.
D

STOP. TH IS IS THE E:-.:-0 OF TflF STRt, C:Tu R E ANO WRITIEN E.'<PRES-


S ION SCCTlvN. If YO U Fl. ISll BEFO R E TI ME IS U P, C HEC K YOUR WORK
ON PARTS A ANO i1 OF TH IS SECTION ONLY. 00 NOT WORK ON ANY
OTHER SEC TIO N OF THE TEST.
3 3 3 3 3 / PRACTICE TEST 4 / 3 3 3 3 3

SECTION 3
READING COMPREHENSION
Time: 55 Minutes
50 Questions

DI AECllONS

In this section, you will read a number of passages. Each one is


followed by approximately ten questions about it. Fo r questions
1-50, choose the one best answer, (A), (B), (C), or (D), to each
q uestion. Then, find the number of the question o n your answer
sheet, and fill in the space that corresponds to the le tter of the
answer you have chosen. Answer all questions following a passage
on the basis of what is stated o r implied in that passagt:.

Questions I through I 0 are based on the following passage.

Lichens, of which mo re than twenty thousand species


have been named. are complex associations between certain
fungi and certain algae. The lichen itself is not an organism;
rather it is the morphological and biochemical product of
(5) the association. Neither a fungus nor an alga alone can
produce a l!chen.
The intimate relations hip between these two living com-
po nents of a lichen was o nce erroneously thought to
represent mutualisn1. In mutualistic relationships, bo th
(10) participants benefit. With lichens, however, it appears the
fungus actually parasitizes the a lgae . This is one of the
conclusions drawn fro m experiments in which the two
components of lichens were separated and grown apart.
In nature. lichen fungi may encounter and grow around
(15) several kinds of algae. Some types of algae the fungi may
kill: other types it may reject. Lichen algae are autotrophic.
meaning they make their owri food through photosynthesis.
Lichen fungi a re heterotrophic. meaning they depend upon

421
3 3 3' 3 3 / PRACTICETEST4/ 3 3 3 3 3

the algae within the lichen to supply their food . Up to ninety


(20) percent of the food made by the green algal cells is
transferred to the fungus. What, if anything, the fungus
contributes to the association is not well understood.
Lichens are hardy. They grow in many habitats and are
often pioneers in hostile environments where few other
(25) organisms can flourish. They have been known to grow
endolithically, having been discovered thriving inside of
rocks in Antarctica. Lichens help reduce erosion by stabiliz-
ing soil. Several kinds of insects glue lichens to their
exoskeletons for camouflage. Many species of birds use
(30) lichens as building materials for nests. Humans have used
lichens for dyes and antibiotics.

L Which of the following best describes the lichen association?


(A) Simple plants made of two different au totrophic organisms
(B) A mutual istic association between a fungus and an a lga
(C) A parasitic association between two fungi, one a utotro-
phic, the second heterotrophic
(D) A union between a parasitic fungus a nd an autotrophic
alga

2. The word " hardy" in line 23 is closest in meaning to


(A) tender (C) armed
(B) durable (D) beneficial

3. In biology, mutualism occurs when two different organisms live


close together and
(A) o ne organism parasitizes the other
(B) both o rganisms benefit from the association
(C) both organisms arc harmed by the association
(D) o ne organism benefits while the other does not o r is
harmed by the association

4. In line 7, th<: word " itttimate" is nearest in meaning to


{A) living (C) biological
(!l) extraordinary (D) close

422
3 3 3 3 3 / PRACTICE TEST4/ 3 3 3 3 3

5. Lichens serve as camouflage fo r which of the following?


(A) Insects (C) Reptiles
(B) Birds (D) Mammals

6. The true nature of the relationship between the lichen compo-


nents was clarified by
(A) examining lichens with a microscope
( B) observing lichens placed in the dark
(C) observing the lichen components when grown apart
(D) decreasing the amount of nutrients available to the lichens

7. In line 24, the word " hostile" is closest in meaning to


(A) unusual (C) untraveled
(B) cool (D) inhospitable

8. An endolithic lichen is one that


(A) grows in the canopies of trees
( B) grows inside rocks
(C) grows at very high altitudes
(D) grows inside other organisms, including other. lichens

9. Many lichens contribute to the communities they inhabit by


(A) removing polluta nts from the air
(B) controlling wood-rotting fungi
(C) slowing the spread of viruses
(D) reducing soil erosion

10. In what part of the passage does the author indicate that
scientists have changed their previous position on the makeup
of lichens?
(A) Lines 3-5 (C) Lines 14-16
(B) Lines 7-9 (D) Lines 25-27

423
3 3 3\ 3 3 / PRACTICE TEST4/ 3 3 3 3 3

Questions 11 thro ugh 20 are based on the following passage.

When buying a house, you must be sure to have it checked


for termites. A termite is much like an ant in its communal
habits, although physically the two insects are distinct.
Like those of ants, termite colonies consist of different
(5) classes, each with its own particular job. The most perfectly
formed termites, both male and female, make up the
reproductive class. They have eyes, hard body walls, and
fully developed wings. A pair of reproductive termites
founds the colony. When new reproductive termites de-
(10) velop, they leave to fo rm another colony. They use their
wings only this one ti me and then break them off.
The worker te rmites are small, blind , and wingless, with
soft bodies. They make up the majori ty of the colony and do
all the work. Soldiers are eyeless and wingless but are larger
(15) than the workers and have hard heads and strong jaws and
legs. They defend the colony and are cared for by the
workers.
The male and female of the reproductive class remain
inside a closed-in cell where the female lays thousands of
(20) eggs. The workers place the eggs in cells and care for them.
Even if one colony is treated with poison, if a male and
female of the reproductive class escape, they can form a new
colony.
Pest control companies can inspect a house for infesta-
(25) tion of termites. Often, a lay person cannot spot the
evidence, so it is critica l to have the opinion of a profes-
sional. Treatments vary depending upon the type of termite.

11. How are te rmites like ants?


(A) They live in communities, and each class has a specific
duty.
( B) Their bodies are the same shape.
(C) The king and queen are imprisoned.
(D) The females' reproductive capacities are the same.

424
3 3 3 3 3 /PRACTICE TEST4:f 3 3 3 3 3

12. The word "communal" in line 2 is closest in m eaning to


(A) eating (C) organizational
( B) reproducing (D) social

13. Which of the following is NOT true?


(A) All termites have eyes.
(B) Some termites cannot fly.
(C) Workers are smaller than soldiers.
(D) Termites do not fly often.

14. In line 3, the word "distinct" is closest in meaning to


{A) similar (C) genetically related
( B) different (D) strong

15. In line 5, " classes" is closest in meaning to


(A) sexes (C) courses
(B) colonies (D) categories

16. Which of the following statements is probably true?


(A) Thousands of termites may move together to develop a
new colony.
(B) The male and female reproductives do not venture out-
doors except to form a new colony.
( C) There are more soldiers than workers.
(D) A worker could easily kill a soldier.

17. The word "founds" in line 9 is nearest in meaning to


(A) establishes (C) controls
(B) destroys (D) guards

18. The word "cells" in line 20 is closest in meaning to


(A) combs (C) compartments
(B) rows (D) piacenta

425
3 3 3'3 3 / PRACTICE TEST 4/ 3 3 3 3 3

19. Which of the followi ng would be the best title for this passage?
(A) Termites Destroy Houses
(8) Termites Wo rk Well Together
(C) The Hab its and Physical Characteristics of Tamitcs
( D) The Relationship of Soldier and Worker Termites

20. The wo rd '·each" in line 5 refers to


(A) ants (8) colonies (C) jobs (D) classes

Questions 21 through 31 are based on the following passage.

In recent yea rs, there has been an increasing awareness of


the inadequacies of the judicial syste m in the Un ited States.
Costs are staggering both fo r the taxpayers and the litigants-
and the litigants, or parties, have to wai t sometimes many
(5) years before having their day in court. Many suggestio ns
have been made concerning methods of ameliorating the
situation, but as in most branches of government, changes
come slowly.
One suggestion that has been made in order to maxi mize
(10) the efficiency of the system is to allow districts that have an
overabundance of pending cases to borrow judges from
o ther districts that do not have such a backlog. Anothe r
suggestion is to use pretrial conferences, in which judges
meet in their chambers with the litigants and their atto rneys
(15) in order to narrow the issues, limit the witnesses, a nd
provide for a more orderly trial. The theory behind pretrial
conferences is that judges will spend less time on each case
and parties will more readily settle before trial when they
realize the adequacy of their claims and their opponents·
(20) evidence. Unfortunately, at least o ne study has shown that
pretrial conferences use mo re judicial time than they save,
rare ly result in pretrial settlements, a nd actually result in
higher damage settlements.
Ma ny states have now established another method, small-
(25) claims courts, in which cases over sm all sums of money can

426
3 3 3 3 3 / PRACTICETEST4/ 3 3 3 3 3

be disposed of with co nsiderable dispatch. Such proceed-


ings cost the litigants almost nothing. In California, fo r
example, the parties must appear before the judge without
the assistance of counsel. The proceedings are quite infor-
(30) mat and there is no pleading-the litigants need to make
only a o ne-sentence statement of their claim. By going to
this type of court, the plaintiff waives any right to a jury trial
and the right to appea l the decision.
In coming years, we can expect to see more and more
(35) innovatio ns in the continuing effo rt to remedy a situation
which must be remedied if the citizens who have valid claims
are going to be able to have their day in court.

21. The pre trial conference, in theory, is supposed to do all of the


following EXCEPT
(A) narrow the issues
( B) cause early settlements
(C) save judicial time
(D) increase settlement costs

22. The wo rd "ameliorating" in line 6 is closest in meaning to


(A) improving (C) wo rsening
(B) increasing (D) distinguishing

23. In line 12, the word "backlog" is closest in meaning to


(A) laziness (C) overload
(B) ine fficie ncy (D) dearth

24. What is the main topic of the passage?


(A) All states sho uld follow California 's example in using
small-claims courts in order to free judges fo r other wo rlc.
( B) The legislature needs to formulate fewer laws so that the
judiciary can catch up on its older cases.
(C) Nobody seems to care enough to attempt to find me thods
for making the judicial system more efficient.
(D) While there are many problems with the court system,
the re are viable suggestions fo r improvement.

427
3 3 3, 3 3 / PRACTICETEST4/ 3 3 3 3 3

25. In line 3, " litigants" is closest in meaning to


(A) jury members (C) parties in a lawsuit
(B) commentators (D) taxpayers

26. Which of the following is true about small-claims courts?


(A) It is possible to have one's case heard by a jury if one is
dissatisfied wi th the court's decision .
(B) The litigants must plead accurately and according to a
strict form.
(C) The decision may not be appealed to a higher court.
(D) T he parties may not present their cases without an
attorney's help.

27. The word "staggering" in line 3 is nearest in meaning to


(A) up and down (C) charged
( B) decreasing (D) astounding

28. The word " dispatch" in line 26 means most nearly the same as
(A) transmi ttal (B) haste (C) clarity (D) conflict

29. It is implied in the passage that


(A) most people who feel they have been wronged have a ready
remedy in courts of law
(B) many people would like to bring a case to court but are
unable to because of the cost and time required
( C) the judicial system in the United States is highly acclaimed
for its efficiency
(D) someday pretrial conferences likely will replaC\ trials
completely

30. The word "remedy" in line 35 is closest in meaning to


(A) correct (B) review (C) expose (D) discover

31. The passage indicates that pretrial conferences may not actu-
a lly produce positive ref,ults in lines
(A) 3-5 (B) 16- 19 (C) 20-23 (D) 29-31

428
3 3 3 3 3 / PRACTlCE T EST4 / 3 3 3 3 3

Questions 3 2 through 39 are based on the following passage.

In an effort to produce the largest, fastest. and most


luxurious ship afloat, the British built the S.S. Titanic. It was
so supe rior to anything else o n t he seas that it was dubbed
''unsinkable." So sure of this were the owners that they
(5) provided only twenty lifeboats and rafts, less than one half
the number needed for the 2,227 passengers on board.
M<tny p:..-M: ne.ers were aboard th e ;iight it rammed a n
iceberg, only two days at sea and more than halfway
between England and its New York destination. Because
(10) the luxury liner was traveling so fast, it was impossible to
avoid th e ghostly looking iceberg. An unextinguished fire
also contributed to the ship's submersion. Panic increased
the numbe r o f casualties as people jum ped in to the icy wa ter
or fought t0 be a mong the few to board the lifeboats. Four
(15) hours after the mishap, a n oth er ship, the Ca1pathia, rescued
the 705 suivivors.
The infamous S.S. Titanic had e njoyed o nly two days of
sailing g lory o n its maiden voyage in 19 12 before plunging
into 12,000 feet of water near the coast of Newfo undland,
(20) where it lies today.

32. All of the followi ng are true EXCEPT


(A) o nly a third of those aboard perished
( B) the Carpathia rescued the survivors
(C) the S.S. Titanic sank nea r Newfoundl ond
(D) the S.S. Titanic was the fastest ship alloat in 1912

33. All of the following contrib uted to the !arge death toll EXCEPT
(A) panic (B) fire (C) speed (D) the Carpachia

34. H ow many days was t he S.S. Titanic at sea before sinking?


(A) 2 (B) 4 (C) 6 (D) 12

429
3 3 3 '3 3 / PRACTICETEST4/ 3 3 3 3 3

35. In line 11, the word "unextinguished" is closest in m ea ning to


(A) indcs 1ruc1ablc (C) undiscovered
(B) uncontrollable (D) unquenched

36. In line 18, " maiden voyage" is closest in meaning to


(A) inaugural (C) longest
(B) most elegant (D) final

37. The word " dubbed" in line 3 is closest in meaning to


(A) called (C) christe n e d
(B) initiated (D) listed

38. What is the main idea of this passage?


(A) The S.S. Titanic proved itself the most seaworthy vessel in
1912.
(B) Attempts to rescue the S.S. Titanic's survivors were not
successful.
(C) Overconfidence by builders and owners was greatly respon-
sible for the sin king of the vessel.
(D) A fire and panic were the only cau!'es for the sinking of the
ship.

39. In which lines does the author indicate that the S. S. Titanic's
owners were overly confident about its seawonhiness?
(A) Lines 1- 2 (C) Lines 7-9
(B) Lines 4--6 (D) Lines 14-16

Questions 40 through 50 are based on the following :;assage.

One of the seven wonders of the ancient world, the G reat


Pyramid of Giza was a monument of wisdom and prophecy
built as a tomb fo r Pharaoh Cheops in 2720 B.C. Despite its
antiquity, cen a in aspects of its construction m:ike it one of
(5) the truly great wonders of the world. The thirteen-acre
structure near the Nile River is a solid mass of stone blocks
covered with limestone. Inside are a number of hidden

430
3 3 3 3 3 / PRAcn-:ETt=~ i .+/ 3 3 3 3 3

passageways and the burial .:. h a ri 1o l;1· t0 r the pharaoh. It is


the largest single ~ L~ ·1 ct urc in ~he wor!J . The four sides of
(10) the pyramid are aligned .ilmosr exac rly o n rrne north. south,
east, and west-an im:redible engineering foa t. Thc 1ncicnr
Egyptians were -.un wor~hipers and great ast r 0 11 n:1w-_,. " '
computations [or the G reat Pyramid were b:i ~c '. .. n ..... ••·
nomical observati,rns.
l 15) Exploratio ns ;rnJ .!ctailed examinatinns ot t i1c:: j,,.,,<" ',f th.:
s tructure revc:al many ·::: ,_ :· '"'. ~ .;.·: =:urthc::r scicntiiic
s tudy indicates 1 hat th ~ . p . . -· . : t;. :--c: of ti.rneline of
eve nts-past, p1 t:seni . ar:_ .. ~re. Many of the events have
been interprcteJ :ind f\.lund to coincide with known facts of
(:ZU) the past. Others are prophesied for future gene rations a nd
arc currently under investigation. Many believe that pyra-
mids have supernatural powers, and this one is no excep-
tion. Some researchers even associa te it with extraterrestrial
beings of the ancient past.
es) Was this superstructure made by ordinary beings, o r one
built by a race far superior to any known today?

40. What has research of the base revealed?


(A) There are cracks in the foundation.
( B) Tomb robbe rs have stolen the pharaoh's body.
(C ) The lines represent important events.
(D) A superior race of people built it.

41. Extraterrestrial beings a re


(A) very strong workers
( B) astronomers in the ancient times
(C) re~earchers in Egyptology
(D) living beings from other planets

4J I
3 3 3 '3 3 / PRACTICE TEsT4/ 3 3 3 3 3

42. Wha t was the most probable reason for providing so many
hidden passages?
(A) To allow the weight of the pyramid to settle evenly
(B) To permit the high priests to pray at night
(C) To enable the pharaoh's family to bring food for his
journey to the afterlife
(0) To keep grave robbers from finding the tomb and the
treasure buried with the pharaoh
)
43. The word " intersecting" in line 15 is ne:1resr in m cnni"5 w
(A) crossing l C) observing
(B) aligning (0) cutting

44. What do the intersect ing lines in the hast.: symbolize ?


(A) ArchiteCL';· plans for the hidden passages
( B) Pathways of the grea t solar bodies
(C) Astrological compu ta tions
(0 ) Dates of important c:vents <aking place throughout time

45 . In line :20, the wo rd " prophesied" is dosest in meaning to


(A) affilia ted (C) terminated
(B) precipitawd (0) fo retold

46. What is the best title fo r the passage?


(A) Symbolism of the G reat Pyram id
(B) Problems with the Cmstruction o f the G reat Pyramid
(C) Wonders of the G reat Pyramid of Giza
(0) Exploratio n of the Burial C hamber of Cheops

47. On what did the ancient Egyp tians base their calculations?
(AJ Observation of the celt:scial bodies
(B ) Advanced techno logy
(C) Advanced too ls of measurement
(0) Knowledge of the car< h's surface
3 J J 3 3 /PRACTICE TEsT4/ J J 3 3 3

48. Why was the Great Pyramid constructed?


(A) As a solar observatory
( B ) As a re ligious temple
' ., As a tomh for the pha ra,_· ~
l D) AS an engineering feat

49. Why is the Great Pyramid of Giza considered o ne of the seven


wonders of the world?
(A) It is perfectly aligned with the four cardinal points of the
compass and contains many prophecies.
(B) It was selected as the tomb of Pharaoh Cheops.
(C) It was built by a super race.
(D) It is very old .

50. The word " feat " in line 11 is closest in meaning to


(A) accomplishment (C) festivity
( B) appendage (D) structure

STOP. THIS IS THE END OF T H E EXAMINATION. I F YOU FINISH BEFORE


T IME IS UP, CHECK YOUR WORK JN TH IS SECTION ONLY. Do NOT WORK
ON ANY OTHER SECTION OF THE TEST.

433
1 1 1 1 1 / PRACTICE TEST 5/ 1 1 1 1 1

PRACTICE TEST 5

SECTION 1
LISTENING COMPREHENSION
Time: Approximately 30 Minutes
SOQuestions

Section l has three parts. Each part has its own set of directions.
Do not take notes while listening or make any marks on the test
pages. Notetaking, underlining, o r crossing o ut will be considered
cheating on the actual TOEFL exam. Answer the questions
following the conversations or talks based on what the speakers
have stated or implied.
For Practice Test 5, restart your Listening Comprehension
cassette immediately following Practice Test 4. On the actual
TOEFL, you will be given extra time to go on to the next page when
you finish a page in the Listening Comprehension section. In the
following test, however. you will have only the 12 seconds given after
each question. Turn the page as soon as you have marked your
answer. Start the cassette now.

Part A

DIRECTIONS

In Part A, you will hear short conversations between two


speakers. At the end of each conversation, a third voice will ask a
question about what was said. The question will be spoken just one
time. After you hear a conversation and the question about it, read
the four possible answers and decide which one would be the best
answer to the question you have heard. Then, on your answer sheet,
find the number of the problem and mark your answer.

434
1 1 1 1 1 / PRACTICETEST5 / 1 1 1 1 1

1. (A) They were displeased.


(B) They found it sad.
(C) They thought it was shocking, but very funny.
(D) They became angry at the promiscuity.

2. (A) To the beach. (C) To a movie theater.


( B) To a play. (D) To a restaurant.

3. (A) He 's dying.


(B) He doesn't hear too well.
(C) He was at a party.
(D) He was reading something important.

4. (A) The class thought the demonstration was too complex.


( B ) Too many s tudents showed up.
(C) The professor didn't show up.
(D) The professor canceled it.

5. (A) It's more direct. (C) It's fas ter.


(B) There's a traffic jam. (D) It's less expensive.

6. (A) He got a one-way plane ticket.


(B) He went the wrong direction on a one-way street.
(C) He made an improper tum.
(D) He slowed down at the wrong time.

7. (A) Susan Flannigan is in a bell-ringing group.


(B) Her name sounds familiar.
(C) Susan Flannigan is ringing the bell now.
(D) Her name sounds melodic.

8. (A} Roy's standing in line for a gold medal.


( B) Roy was the best, so he got a gold medal.
(C) Nobody's better than Roy at getting gold medals.
(D) Roy probably won't win a gold medal.

435
1J 1 1 1 1 / PRACTICETEST5 / 1 l 1 1 1

9. (A) The cartridge does not need to be replaced.


(8) He does not intend to change the cartridge.
(C) He already changed the cartridge.
(D) He is uncomfortable because the woman is watching him.

10. (A) Every week, there are three direct flights from Atlanta to
Chicago.
(B) Next week, the three flights from Atlanta to Chicago will
be stopped.
(C) Three planes which travel from Atlanta to Chicago each
week make nine stops e nroute. ·
(D) The number of planes that travel from Atlanta to Chicago
will be reduced within the next three weeks.

11. (A) He is pleased because his fa mily is coming up to see him.


(B) He is considering several maps to decide where to go.
(C) He is rather excited because he has a vacation soon.
(D ) He is coming up to see us on his vacation.

12. (A) She is angry because there is too much chlorine in the pool.
(B) The chlorine in the swimming pool bothers her eyes.
(C) She believes the correct amount of chlorine is essential to a
clean swimming pool.
(D ) She doesn't believe that there is enough chlorine in the
pool.

13. (A) t5he is uncomfortable telling the man that he is losing his
job.
(B) She misplaced the man's papers.
(C) She is unsure when they will be moving.
(D) She hasn't decided where his office will be located.

14. (A) Returned them and got a pair of pants instead.


{B) Took the m back to the store a nd got some differ ent ones.
{C) Got her money back because they didn't fit properly.
{D) Received a refund because of a problem with the heel.

436
·I 1 1 1 1 1 / PRACTICETEST5 / 1 1 1 1 1

15. (A) He does not intend to go o n the field trip.


(B) Some people have not submitted a requ ired form.
(C) The trip has been canceled.
(D) Everybody is likely to go on the ~trip .

16. (A) She was no t able to read her assignment because she broke
her glasses.
(B) She could have read the assignment if she hadn't had to
wash dishes.
(C) She won 't go to class tomorrow because she must go to the
optometrist.
(D) She cut herself on some broken glass, so she didn't do her
homework.

17. (A) She has an easy schedule.


( B) She doesn't pay attention in class.
( C) Taking both courses together is a bad decision.
(D) She is brilliant.

18. (A) It will be canceled as a result of mismanagement of funds.


( B) They probably made an e rror in figuring the expenses.
(C) They must give a comple te re port on the estimated costs.
(D ) They have to charge t he calculations to the company o ffice.

" 19. (A) She was able to go because her employer paid her
expenses.
(B) She couldn't go because her boss wo uldn 't pay her while
she was away.
(C) Altho ugh he r employer had offered to pay her expenses,
she didn 't go.
(D) Her boss refused to give her money, but she went anyway.

20. (A) Although June doesn't like television . her husband watches
it t:very night.
(B) June refuses to let her husband watch television.
(C) June always asks her husband to watch television with her.
(D) June's husband refuses to let he r watch televisio n.

437
l l l 1 1 / PRACTICETEST5/ l l l 1 1

21. (A) If it were a hot day, the trip would be difficult.


(B) It is a very hot day for the long trip.
(C) It's a magnificent day.
(D) It's not as far as the woman think:; to their destination.

22. (A) The Kehoes got a bargain.


(B) Chuck bought a new house.
(C) The Kehoes bought a house out of the country.
(D) Mr. Kehoe is a real estate agent.

23. (A) The food spoiled.


(B) The group was shameful.
(C) The weather was bad.
(D) The program director wanted to have it on another day.

24. (A) Sebring High School.


(B) Clark High School.
(C) Melrose Community College.
(D) Enrold College.

25. {A) A bicycle. (C) A shirt.


(B) Agame. (D) Baseball shoes.

26. (A) Please give me your hand.


(B) Would you help me carry these packages?
( C) Please remov.! your hands from those packages.
(D) My hand is stuck under the packages.

27. (A) She is trying to find a new typing job.


(B) She is looking for somebody to type her research paper.
(C) She is trying to find somebody to move her typewriter to
another table.
(D) She has accepted employment as a typist.

438
1 1 1 1 1 / PRACTICE TEsT 5 I 1 1 1 1 1

28. {A) Harvey didn't go to class because he didn't know there was
going to be a test.
(B) Harvey didn't want to take the test, so he skippe d class.
(C) Harvey went to class altho ugh he didn't want to take the
test.
(D) Harvey was happy that yesterday's test was postponed.

29. (A) Joe uses some strange me thods when he studies.


( B) Joe receives very good grades although he doesn't study.
(C) Joe is very fond of studying dange rous situations.
(D) It's too bad that Joe dislikes studying.

30. (A) He has entered the university hospital for treatme nt.
( B) He met his wife while s he was working as a nurse a t the
university hospital.
(C) He wants to find a place close to the university to keep his
children during the day.
(D) He likes the un iversity because it has a good nursing
program.

Go ON TO PART B

Part B

DIRECTIONS

In P art B, you will hear longer convPrsations. After each


conversatio n, you will be asked some questions. The conversations
and questions will be spoken just o ne time. They will not be writte n
out for you, so you will have to listen carefully in order to
understand and remembe r what the speaker says.
When you hear a questio n, read the four possible answt:rs in your
test book and decide which one would be the best answer to the
question you have heard. Then, o n your answer sheet, find the
number of the problem and fill in the space that correspo nds to the
letter of the answer you have chosen.

439
1 1 1 1 1 / PRACTICE TEsT 5 I 1 1 1 1 1

31. (A) More than $195. (C) $150.


(B) Less than $195. (D ) Less than $150.

32. (A) O ne day. (C) Several hours.


(B) Four days. (D) Te n hours.

33. (A) Mechanic. (C) TV repairman.


(B) Policeman. (D) Car salesman.

34. (A) Broken fuel pump. (C) Dirty oil.


(B) Dirty carburetor. (D) Leaky radiator.

35. (A) All her expenses will be paid.


(B) She'll earn a great deal of money.
(C) She can practice her Spanish.
(D) She can spend her free time at the beach.

36. (A) One week. (C) Six weeks.


(B) Immediately. (D) A few ho urs.

37. (A) Swimsuit. ( C) Passpo rt.


(B) A Spanish dictionary. (D) Money.

38. (A) Interview local artists.


( B) Photograph the craftsmen.
(C) Write her story.
(D) Listen to mariachi music.

Go ON TO PART C

440
1 1 1 1 1 I PRACTICE TEST 5I 1 1 t 1 1

PartC

DIRECTIONS

In Part C, you will h'!ar several talks. After each talk, you will be
asked some questions. The talks and questions will be spoken just
one time. They will not be written out for you, so you will have to
listen carefully in order to understand and remember what the
speaker says.
When you hear a question, read the fo ur possible answers in your
test book and decide which one would be the best answer to the
question you have heard. Then, on your answer sheet, find the
number of the problem and fill in the space that corresponds to the
letter of the answer you have chosen.

39. (A) He was struck by lightning.


( B) He was very old.
( C) He was in a car accident.
(D) He fell down in his yard.

40. (A) His wife. (B) A tree. (C) A clock. (D) Lightning.

41. (A) Edwards had been blind for nine years.


( B) Edwards was unconscious fo r twenty minutes after the
lightning had struck him.
( C) Doctors believe that Edwards was never rea lly blind or
deaf.
(D) Edwards awoke with his face in a puddle of water.

42. (A) Hiding from the storm under a tree.


(B) Climbing a tree.
(C) Driving a car.
(D) Lying on the ground.

441
1 1 1 1 1 / PRACTICETEsT 5 / 1 1 1 1 1

43. {A ) H e regained his sight from a head injury when he fell from
a tree.
( B) He was happy after his wife entered his room for the first
time in nine years.
(C) The lightning took the feeling from his legs and gave
feeling in his eyes.
(D ) Because the blow that blinded him was very severe, it took
another very severe blow to restore his sight.

44. (A) Co tton. (B) Nylon. (C) Grains. ( D ) Rayon.

45. (A) It is the smallest state in size.


( B ) It was the first to discover lightweight fiber.
(C) It was the fi rst to ratify the Constitution.
(D) It was the "bread baske t" in colonial days.

46. (A) Irish. (B) Swerlish. (C) English. (D) Dutch.

47. (A) It was at the heart of the country.


(B) It was extremely small.
(C) Its inhabitants sold baskets wh ich they made by hand.
(D ) Its inhabita nts produced corn, wheat, and o the r grains,
which were sold throughout the country.

48. (A) A duck. (C) A chameleon.


(B ) A skunk. (D) An Arctic fox.

49. (A ) Their bite. ( C) Their odor.


(B ) Their pigmenta tion. (D) T heir quills.

50. (A) Claws. ( C) Bite.


( B ) Sting. (D) Pigment<•tio n.

STOP. TIII S IS T llE E ND OF T ll E LISTEN ING CO MPREH E N S ION S ECTIO N.


Go o To "Er r10N 2.

442
2 2 2 2 2 I PRAcnCE TEST 5/ 2 2 2 2 2

SECTION 2
STRUCTURE AND WRITTEN EXPRESSION
T ime: 25 Minutes
40 Questions

Part A

DIRECT IONS

Questions l 15 are inco mplete sentences. I3cneath each sente nce


;·:::.u wiil ..,ee tou; ·;.ords or phrases. mr:rked (A), (B), (C), and (D).
Choose the one word or phrase that best completes the o;encencc.
Then. o n yo ur an~we r sheet, find the number of the question and fill
in the space that corresponds to the letter of the answer you hav1..:
chosen. Fill in the space so that the lem.: r inside the oval cannot be
seen.

1. I understand that the governo r is considering a new proposal

(A) what would eliminate unnecessary writing in government


( B) who wan ts to cut down on the amount of writing in
5overnmen•
(C) that wv uld eliminate unnecessary paperwork iTJ govern-
ment
(D ) to cause that the amo unt of papers written in govern ment
offices will be reduced

2. The docto r told his receptionist that he woul d 1eturn

(A) as early as it would be possible


( B) at the earliest that it could be possible
(C) as soon as possible
(D ) at the nearest early possibility

443
2 2 2 2 2 / PRACTICETEST5 / 2 2 2 2 2

3. George belongs to the - - - - -


(A) class of the upper middle
( B) upper middle class
(C) class from the center up
(D) high mediu·m class

4. A good student must know---- -


(A) to study ha rd
(B) to be a good student
( C) how to study effectively
(D) the way of efficiency in study

5. Jane changed her major from French to business,


(A) with hopes to be able easier to locate employment
( B) hoping she can easier get a job
(C) with the hope for being able to find better a job
(D) hoping to find a job more easily

6. He has received several scholarships _ _ _ __


(A) not o nly because of his artistic but his academic :ibility
{B) for both his academic ability as well :is his artistic
{C) because of his academic and artistic ability
{D) as resulting of his ability in the :irt and the academy

7. Harvey will wash the clothes. _ _ _ __


(A) iron the shirts, prepare the meal, dusting the furn iture
(B) ironing the shirts, preparing the meal, and d usting the
furniture
(C) iron the shirts, prepare the meal. and dust the furniture
(D) to iron the shirts, prepare tht: meal, and dust the furniture
8. _ _ _ __ that new information to anyone else but the
sergi!ant.
{A) T!1ey asked him not to give
( B) They asked him to don 't give
(C) They asked him no give
(D) They asked him to nb give

441
2 2 2 2 2 / PRACTICETEST5 / 2 2 2 2 2

9. , he would have signed his name in the corner.


(A) If he painted that picture
( B) If he paints that picture
(C) If he had painted that picture
(D) If he would have painted that picture

10. The doctor insisted that his patient-----


(A) that he not work too hard for three months
(B) take it easy for three months
(C) taki ng it easy inside of three months
(D) to take some vacations for three months

11. The manager was angry because somebody _ _ _ __


(A) had allowed the photographers to enter the building
(B) had let the photographers to enter into the building
(C) permitting the photographers enter the building
(D) the photographers let into the building without the proper
documentations

12. Richard was asked to withdraw from graduate school because

(A) they believed he was not really able to complete research


(B) he was deemed incapable of completing his research
(C) it was decided that he was not capable to complete the
research
(D) his ability to finish the research was not believed o r trusted

13. The committee members resented - - - - -


(A) the president that he did not tell them about the meeting
(B) the president not to inform them of the meeting
(C) the president's not informing them of the meeting
(D) that the president had failed informing themselves that
there was going to be a meeting

445
2 2 2 2 2 / PRACTICE T EST 5 / 2 2 2 2 2

14. did Arthur realize tha t there wa:, danger.


(A) Upon entering the store
(B) When he entered the sto re
(C) After he had entered the store
(D) O nly after e ntering the store

15. The rabbit scurrit!d away in frigh _ _ __ __


(A) when it heard the movement in the bushes
( B) the movement among the bushes having been heard
( C) after it was hearing moving inside of the bushes
(D) when he has heard that something moved in the bushes

GO ON TO PART B

Part B

DIRECTIONS

In questions 16-40, each sentence has four underlined words or


phrases. The four underlined parts of the sentence are marked (A),
(B), (C), and (D). Identify the one underlined word o r phrase that
must be changed in order for the sentence to be correct. Then, o n
your answer sheet, find the number of the question a nd fill in the
space that corresponds to the letter of the answer you have chosen.

16. Neither of the girls have turned in the term papers


A B
to the instructor ~-
c D

17. After studying all the new materials, the student was able to
A B C
rise his test score by twenty-five points.
D

446
[ 2 2 2 2 2 / P RACTlCETEST5 /. 2 2 2 2 2

18. The book that you see~ o n the table belo ngs to the
A B C D
teac her.

19. I suggest that he goes to the doctor as soon as he


A - B- c
returns from caking the exam.
D

20. She is looking forwa rd tog£ to Europe after she finishes her
A B C
studies at the university.
D

21. T hey sa id th at the man jumped off of the bndgt: and


A B C
plunged into the freezing wate r.
D

22. Mr. Anderson used to~ in the crisp mo rning air d uring
A B C
the winter mo nths, but now he has stopped.
D

23. The volume four of our encyclopedia set has been missing
A B C
fo r two mo nths.
D

24. I do no t know where could he have gone so e arly


A B C
m the mo rning.
D

447
2 2 2 2 2 /PRACT1CE T EST5/ 2 2 2 2 2

25. The people tried of defending their village, but they were
A B
finally forced to retreat.
C D

26. The professor was considering postponing the examination


A B
until the following week because the students' confusion.
c 0

27. Having lost the election, the presidential candidate intends


A
supporting the opposition despite the objections of his staff.
B C 0

28. The congressman, accompanied~ secret service agents and


A
aides, ~ preparing to enter the convention hall
B C
within the next few minutes.
0

29. Because the torrential rains that had devastated the area, the
A B C
governor sent the National Guard to assist in the clean-up
0
operation.

30. Lack of sanitation in restaurants are a major cause of disease


A B C
in some areas of the country.
D

448
2 2 2 2 2 / PRACTICETEST5/. 2 2 2 2 2

31. Had the committee members considered the alternatives


A
more carefullv, they would have realized that the
B
second was better as the first.
C D

32. Mal nutritio n is a major cause of death


A
in those countries where the cultivation of rice have been
B C
impeded by recurrent drought.
D

33. The decision to withdraw all support from the activities of the
A B
athletes are causing an uproar among the athletes' fans.
C D

34. Underutilized species of fish has been proposed as a solution


A B C
to the famine in many underdeveloped countries.
D

35. Because the residents had worked so diligent to renovate the


A B C D
old building, the manager had a party.

36. Jo hn·s wisdom teeth were troubling him. so he went to a


A
dental surgeon to see about having them ~·
B C D

37. Hardly he had entered the office when he realized that he had
A B C

449
2 2 2 2 2 / PRACTICE TEST 5 / 2 2 2 2 2

38. Su_zy had better to change her study habits if she


A B
hopes to be admitted to a good unive rsity.
C D

39. The teacher told the students to don ' t discuss the take-home
A B C
exam with each other.
D

40. Some bacteria are extremely harmfu l, but ano thers are
A B
regularly used in pro ducing cheeses, crackers. and
c
many other foods.
D

STOP. THIS IS THE END OF THE STRUCTURE AND WRllTE EXPRES-


SION SECTION. I F YOU FINISH BEFO RE TlME IS UP, CHECK YOU R WORK
ON PARTS A AND B OF nns SECTfON ONLY. Do NOT WORK ON ANY
OTHER SECTION OF THE TEST.

450
[ 3 3 3 3 3 /PRACTICE TEsT 5I 3 3 3 3 3

SECTION 3
READING COMPREHENSION
Time: SS Minutes
SO Questions

DIRECTIONS

In this section, you will read a number of passages. Each one is


followed by approximarely ten questions about it. For questions
i-:JU. c11oose the one best answer, (A), ( B), (C), or (D), to each
q uestion. Then, find the number of the question on your answer
sheet. and fill in the space that corresponds to the letter of the
answer you have chosen. Answer a ll questions following a passage
on the basis of what is stated or implied in that passage.

Questions I through 11 are based on the following passage.

T he food we eat seems to have profound effects on o ur


health. Altho ugh science has made eno rmous steps in
making food more fit to eat. it has, at the same time, made
ma ny foods unfit to eat. Some research has shown that
(5) per haps eighty percent of all human ill nesses are related to
diet and forty percent of cancer is related to the diet as we ll,
especially cancer of the colon. People of differe nt cultures
are more prone to contract certain illnesses because of the
cha racteristic foods they consume.
(10) That food is related to illness is not a new discovery. In
1945, government researchers realized that nitrates and
nitrites (commonly used to preserve color in meats) as well
as other food additives caused cancer. Yet, these carcino-
genic additives re main in our food, and it becomes more
(15) difficult all the time to know which ingredients on the
packaging labels of processed food are helpful or harmful.
The additives tha t we eat are not all so direcc. Farmers
often give penicillin to cattle and po ultry, and because of

451
3 3 3 3 3 / PRACTICE TEST 5 / 3 3 3 3 3

thi~. penicillin has been fo und in the milk of treated cows.


(20) Sometimes similar drugs are administered to animals not
for medicinal purposes, but fo r financial reasons. The
farmers are simply trying to fatte n the animals in order to
obtain a higher price on the market. Although the Food and
Drug Administration (FDA) has tried repeatedly to control
(25) the ·e procedures, the practices continue.
A healthy die t is directly related to good health. Often we
an: unaware of detrimental substances we ingest. Some-
times well-meaning farmers or othe rs who do not realize the
consequences add these substances to food without our
(30) knowledge.

l. How has science done a disservice to people?


(A ) Because of science, disease caused by contaminated food
has been virtually eradicated.
(B ) It has caused a lack of info rmation concerning the value of
food.
(C) As a result of scientific intervention, some potentially
hannful substances have been added to our food.
(D ) The scientists have preserved the color of meats, but not of
vegetables.

2. The word "prone" in line 8 is nea rest in meaning to


(A ) supine (C) healthy
( B) unlikely (D) predisposed

3. What are nitrates used for?


(A) They preserve flavor in packaged foods.
( B) They preserve the color of meats.
(C) They are the objects of research.
(D) They cause the animals to become fatter.
3 3 3 3 3 / PRACTICE T EST 5 / 3 3 3 3 3

4. FDA means
(A) Food Direct Additives
(B) Final Difficult AnalyW'
( C) Food and Drug Administration
(0 ) Federal Dairy Additives

5. The word " these" in line 13 refers to


(A) meats ( C) researchers
(B) colors (D) nitrates and nitrites

6. In line 13, the word "carcinogenic" is closest in mean ing to


(A) trouble-making (C) money-making
( B) color-re taining (0) cancer-causing

7. All of the following stateme nts are true EXCEPT


(A) drugs are always given to an ima ls fo r medical reasons
(B) some of the additives in o ur food are added to the food
itself and some are given to the living animals
(C) researchers have knowo about the potential hazards of
food add itives for more than forty-five years
(D) food may cause forty percent of the cancer in the w0rld

8. The word "additives" in line 13 is closest in meaning to


(A) added substances (C) natural substances
(B) dangerous substances (D) benign substances

9. What is the best title for this passage?


(A) Harmful and Harm less Substances in Food
(B) Improving Health Through a Natural Diet
(C) The Food You Eat Can Affect Your Health
(D) Avoiding Injurious Substances in Food

10. In line 3, the word "fit" is closest in meaning to


(A) athletic (B) suitable (C) tasty (D) adaptable

453
3 3 3 3 3 / P RACTICETEST5/ 3 3 -3 3 3

11. The fact l; ..!~ the topic has been kno wn for some time is
discussed in lines
(A) 2-4 (B) 10-11 (C) 17-19 (D) 26-27

Questions 12 through 21 .<re based on the following passage.

T he a ncient Egyptians firmly bel ieved in the afterlife and


spent their ti me on earth preparing for it. Elaborate burial
rituals included preparing the burial site, providing fo r all of
the deceascd's material needs (food. clothing, jewels, and
(5) tools of rheir trade). and preserving the corpse so that it
would not decay. Thi!-. preservatio n was accomplished
through a procc:-.s of mummification. The ancients left no
written acco unts as to the executio n of this process, so
scientists have had to examine mummies and establish their
(10) own theories. The embalming process might have taken up
to seventy days fo r the pharJ.ohs and nobili ty and only a few
days for the pear.
The embalmers spread a variety of compounds of salt,
spices. and resins in antl over the corpse to preserve it. They
( 15) fo llowed this with d prescribed wrapping, a procedure in
which they wo un d strips of fine linen aro und, over, and
under the body while placing various amulets withi n the
wrapp ings to protect the deceased from harm o n the long
journey to the afterl ife. T hey ..1Iso painted resins over the
(7.0) wrapped linen. Finally, a pharaoh or noble would have: been
encased in a wooden box before being placed i11 a sarcopha-
gus.

454
3 3 3 3 3 / PRACI1CETEST5/ 3 3 3 3 3

12. How have we been able to learn about the mummification


process?
(A) Accurate reco rds have been handed down to us.
( B) Interviews with embalmers who still use the process have
revealed the secret.
(C) After studyi ng mummies, scientists have developed their
own theories. ·
(D) Chemical analysis of the compounds has led us to an
explanation of the method used.

13. The word "they" in line 19 refers to


(A) embalmers (C) pharaoh:.
(B) spices (D) the poor

14. T he embalming process can best be described as


(A) lengthy and complicated
( B) short and simple
(C) strict and unfaltering
(D) wild and terrifying

15. The word " decay" in line 6 is closest in meaning to


(A) die (C) embalm
( B) dete riorate (D) rej uvenate

16. All of the following statements are true EXCEPT


(A) bodies were preserved as a matter of religious belief
(B) all mummification too k seventy days to complete
(C) special compound~ wer:; used to embalm the bodies
(D) it has been difficult to determine the process used

17. Why did the ancient Egyptians mummify the deceased?_


(A) T o preserve the body fro m destruction
( B) To scare tomb robbers
( C) To encase the body in a sarcophagus
(D) To protect the body from harm on the journey to the
afterli fe

455
3 3 3 3 3 / PRACTICE TEsT 5I 3 3 3 3 3

18. It can be inferred that the Egyptians buried food, clothing,


jewels, and tools with the deceased because
(A) the family did not want anyone else to share them
( B) that was the wish of the deceased
( C) they were afraid
(D) the deceased would need them while enroute to the
afterlife

19. The word "amulets" in line 17 is closest in meaning to


(A) weapons (B) coins (C) charms (D) curses

20. In line 6, ·'accomplished" is closest in meaning to


(A) performed (C) reproduced
(B) forsake n (D) dwindled

21. The distinction between mummification of bodies from differ-


ent classes is explained in lines
(A) 2--0 (S) 10-12 (C) 13-14 (D) 15-17

Questions 22 through 30 are based on the following passage.

A tapeworm is a parasite that lives in the intestines of


humans and animals. Some tapeworms attach themselves to
the intestinal wall by means of suckers in their heads.
O thers float freely in the intestines and absorb food through
(5) the walls of their bodies.
A tapeworm consists of numerous segments. When a new
segment fonns, the older ones move to the back of the
animal. E:ich segment contains hermaphroditic sexual or-
gans (that is, male and female organs). The uterus of each
( 10) segment fills with eggs, which develop into embryos. Gener-
ally, when the eggs are ready to hatch, the segment breaks
off and is eliminated through the host's excretory system.
These embryos hatch, develop into larvae, and grow to
adults only if ingested by an intermediate host.
( 15) One may be infected by tapeworms by eating under-

456
I. 3 3 3 3 3 /PRACTICETEST5/ 3 3 3 3 3

cooked beef, pork, or fish. Symptoms include irregular


appetite, abdominal discomfort, anemia, weakness, and
nervousness.

22. The passage implies that all of the following are true EXCEPT
(A) an embryo will cease to develop if not ingested by a host
( B) a tapeworm will continue to live even when segments break
off
(C) the segment farthest back on the tail is the oldest
(D) tapeworms always float freely in the digestive system

23. The word "eliminated" in line 12 is closest in meaning to


(A) ingested (B) expelled (C) eaten (D) grown

24. A hermaphrodite is
(A) a tapeworm
(B) a segment containing an embryo
(C) a being that contains male and female sexual organs
(D) an animal made of segments

25. The word " others" in line 4 refers to


(A) segments (C) eggs
(B) embryos (D) tapeworms

26. Which of the following is probably NOT a symptom of tape-


worm infestation?
(A) Unusual eating habits
(B) Excitability
(C) Deficiency of red blood cells
(D) Euphoria

27. Which of the following statements is true?


(A) A tapeworm uterus contains one egg.
( B) Overcooked beef is a cause of tapeworms.
( C) A male tapeworm must always be ingested before reproduc-
tion will occur.
(D) Tapeworms vary in their methods of ingesting food.

457
3 3 3 3 3 /PRACTICETEST5/ 3 3 3 3 3

28. What would be the bt!sL title for this reading passage?
(A) Parasites
(B) Reproduction of the Tapeworm
(C) The Tapewo rm, a Harmful Parasite
(0) Segmented Parasites

?.9. A tapeworm attaches itself to the intestinal wall by


(A) suction (B) liquid (C) food (D) teeth

30. In line 6, the word "segments" is closest in meaning to


(A) types (B) sections (C) organs (0) worms

Questions 31 through 40 are based on the following passage.

After inventing dyna mite, Swedish-born Alfred Nobel


became a very rich man. However, he foresaw its universally
destructive powers too late. Nobel preferred not to be
remembered as the inventor of dynamite, so in 1895, j ust
(5) two weeks before his death. he created a fund to be used for
awarding prizes to people who had made worthwhile
contributions to humanity. Originally there were five awards:
literature, physics, chemistry, medicine, and peace. Econom-
ics was added in 1968, just sixty-seven years after tht: fi rst
(10) awards ceremony.
Nobel's original legacy of nine million dollars was in-
vested, and the interest on this sum is used for the awards
which vary from $30,000 to $125,000.
Every yea r on December 10, the anniversary o f Nobel's
(15) death. tht:: awards (gold medal, illumin ated diploma, and
money) are prese nted to the winners. Sometimes politi<.:s
plays an impo rtant roie in the judges' decisions. Ame ricans
have won numerous science awards. but relatively few
literature prizes.
(2'J) No awards were presented fro m 1940 to 1942 at the
beginning of World V/ ar IL Some pl'Ople have wo n twv
prizes, but this is rare; o thers have shar~d their prizes.

453
I3 3 3 3 . 3 / PRACTicETusTS/ 3 3 3 3 Jl
3 L The word " foresaw" in line 2 is nearest in meaning to
(A) prevailed (C) prevented
(B) postponed (D) predicted

32. The Nobel Prize was established in order to


(A) recognize worthwhile contributions to humanity
(B) resolve political differences
(C) honor the inventor of dynamite
(D) spend money

33. In which area have Americans received the most awards?


(A) I .itP.rattire (C) Economics
(B) Peace (D) Science

34. All of the following statements are true EXCEPT


(A) awards vary in monetary value
(B ) ceremonies are held on December 10 to commemorate
Nobel's invention
(C) politics plays an important role in selecting the winners
(D) a few individuals have won two awards

35. In how many fields are prizes bestowed?


{A) 2 (B) 5 (C) 6 (D) 10

36. It is implied that Nobel's profession was in


(A) economics (C) literature
(B) medicine (D) science

37. In line 6, "worthwhile" is closest in meaning to


(A) economic (C) trivial
(B) prestigious (D) valuable

38. How much money did Nobel leave for the prizes?
(A) $30,000 (C) $155,000
(B) $125,000 (D) $9,000,000

459
3 3 3 . 3 3 / PRACTICETEST5 / 3 3 3 3 3

39. What is the main idea of this passage?


(A) Alfred Nobel became very rich when he invented dyna-
mite.
(B) Alfred Nobel created awards in six categories fo r contribu-
tions to humani ty.
(C) Alfred Nobel left a ll of his money to science.
(D) Alfred Nobel made a lasting contribution to humanity.

40. T he wo rd " legacy" in line 11 means most nearly the same as


(A) legend (B) bequest (C) prize (D) debt

Questions 41 through 50 are based on the following passage.

Ever since humans have inhabited the earth. they have


made use of various forms of communicatio n. Generally,
this expression of thoughts and feelings has been in the fo rm
of o ral speech. When there is a language barrier, comm uni-
(5) cation is accomplished thro ugh sign language in which
motions stand fo r letters, words, and ideas. Tourists, the
deaf. and the mute have had to resort to this for m of
expression. Many of these symbo ls of who le words are very
picturesque and exact and can be used internationally;
(10) spe lling, however, cannot.
Body language transmits ideas or tho ughts by certain
actions, either intentionally or unintentionally. A wink can
be a way of flirting or indicating that the party is on ly joking.
A nod signifies approval, while shaking the head indica tes a
( 15) negative reaction.
O ther forms of nonlinguistic language can be found in
Braille (a system of raised dots read with the fingertips),
signal flags, Mo rse code. and smoke signals. Road maps and
picture signs also guide, warn, and instruct people.
(20) While verbalization is the most common fo rm of lan-
guage, other systems and techniques also express human
thoughts and feelings.

460
3 3 3 3 3. / PRACTICE TEST 5/ 3 3 3 3 3

41 . Whic h of the follow ing bes t s ummarizes th is passage?


tA) When language is a barrier, people will find o ther forms of
communication.
( B) Everybody uses only one fo rm of communication.
(C) Nonlinguistic language is invaluable to foreigners.
(D ) Although other forms of communicatio n exist. verbaliza-
tion is the fastest.

42. The word " these" in line 8 refers to


(A) tourists
( B) the deaf and the mute
(C) thoughts and feelings
(D ) sign language mo tions

43. All of the following statements are true EXCEPT


(A ) there are many fo rms of communication in existence today
( B) verbaliza tion is the most common fo rm of communication
(C ) the deaf and mute use an oral form of communicatio n
(D) ideas and tho ughts can be transmitted by body language

44. Which fo rm o ther than oral speech would be most commonly


used among blind people?
(A) Picture signs (C) Body language
( B) Braille ( D) Signal flags

45. How many different forms of communication are mentioned


here?
(A) 5 (B) 7 (C) 9 (D) 11

46. The word "wink" in line 12 means most nearly the same as
(A) close one eye briefly
(B) close two eyes briefly
( C) bob the head up a nd down
. (D) shake the head from side to side

461
3 3 3 3 3 / PRACTICE TEST 5/ 3 3 3 3 3

47. Sign language is said to be very picturesque and exact and can
be used internationally EXCEPT for
(A) spdling (C) whole words
( B) ideas (D) expressions

48. People need to communicate in order to


(A) create language ba rriers
(B) keep from reading with their fingertips
(C) be picturesque and exact
(D) express thoughts and feelings

49. What is the best title for the passage?


(A) The Importance of Sign Language
( B) The Many Forms of Communication
( C) Ways of Expressing Feelings
(D) Picturesque Symbols of Communication

50. Who would be MOST likely to use Morse code?


(A) A scientist (C) An airli ne pilot
(B) A spy (D) Atelegrapher

STOP. THIS IS THE END OF THE EXAMINATION. IF YOU FINISH BEFORE


TIME IS UP, CHECK YOUR WORK IN T HIS SECTION ONLY. Do NOT WORK
ON ANY OTHER SECTION OF THE TEST.

462
1 1 1 1 1 / PRACTICE TEST 6/ 1 1 1 1 1 J
PRACTICE TEST 6

SECTION 1
LISTENING COMPREHENSION
Time: Approximately JO Minutes
50 Questions

Section 1 has three parts. Each part has its own set of directions.
Do nOL take notes while listening o r make any marks o n the test
pages. Notetaking, underlining, or crossing o ut will be considered
cheating on the actual TOEFL exam. Answer the questions
following the conversations or talks based on wha t the speakers
have stated or implied.
Fo r Practice Test 6, restart your Listening Comprehension
cassctle immediately fo llowing Practice Test 5. On the actual
TOEFL. you will be give n extra time to go on to the next page when
you finish a page in tht: Listening Comprehension section. In the
following test, howeve r. you will have only the 12 seconds give n after
each question. Turn the page as soon as you have marked your
answer. Start the cassette now.

Part A

DIRECTIONS

In Part A. you will hear short conversations between two


speakers. At the e nd of each conversation, a third voice will ask a
question about w hat was said. The q uestion will be spoken just one
ti me. Aft~r you hear a conversation and the question about it, read
the fo ur possible a nswers and decide which one would be the best
answer to th e q Jestion you have heard. Then, o n your answer sheet,
find the num r.er of the problem and mark your answer.

463
1 1 l 1 1 / PRACTICE TEsT 6/ 1 1 1- 1 1

1. (A) Mark is fo nd of ra re meat.


(B) Mark is angry at the chef.
(C) Mark disl ikes rare mear.
(D) Mark doesn 't want his meat cooked medium rare.

2. (A) The man doesn't like skim milk.


(B) The milk has turned bad.
(C) The man's check-cashing card has expired.
(D) The milk may tum sour if they don 't drink it withi n the
next five days.

3. (A) He has o ther plans.


(B ) He has a bad personality.
(C) He thinks it will be frightening.
(D) He doesn't have eno ugh time.

4. (A) It has been in her fa mily a long time.


( B) It is a fa mily disgrace.
(C) Her mother doesn't like it.
(D) Her boyfriend gave it to her.

5. (A) It was cleaned.


(B) There was a large sale.
(C) The employees had to work very late.
(D) There was a robbery.

6. (A) She has good taste in clothes.


(B) Her choice is not suitable for the occasion.
(C) The skjn is pretty, but not the blouse.
(D) It is too elegant.

7. (A) Jack didn't visit them.


(B) Jack will not visit them because it's not on his way.
(C) They hope Jack will visit them.
(D) They are sure that Jack will visit them if he doesn't run out
of time. -

464
1 1 1 1 1 /PRACI1CE TEST6/ · 1 1 1 1 1

8. (A) She' ll count the votes on the proposal.


(B) She ·11 support the man's proposal.
(C) She'll make the proposal herself.
(D) She'll back out of the proposal.

9. (A) At the jewelry store. (C) From a machine.


(B) From the purchaser. (D) Down the hal l.

10. (A) It will :-ain later.


(B) It's pr·)bably not going to rain.
(C) It's r aining now, but will probably stop.
(D) The rain has already stopped.
11. (A) The man should ask fo r advice.
(B) The man nl!eds to make his own decision.
( C) The man should not buy the house.
(D) The man should make an offe r.

12. (A) Monday, Wednesday, and Friday.


(B) Saturday and Sunday.
(C) Tuesday, Thursday, and Sunday.
(D) Monday, Friday, and Saturday.

13. (A) It was hard for her to learn so much material.


(B) She learned the difficult extremes.
(C) She wasn't able to materialize the difficulties.
lD) She found the material after a difficult search.

14. (A) He studied because it was a nice day.


(B) H e didn't study because it was a very nice day.
(C) He studied in spite of the beautiful weather.
(D) H e likes to study when the weather is nice.

465
1 1 1 1 1 / PRACTICE TEST 6/ 1 1 1 1 1

15. (A) She found a hard seat because the theater was dark.
(B) She couldn't find a seat in the dark.
(C) She couldn't seat her friends.
(D) She had some difficulty finding a seat.

16. (A) After the class had begun, some of the brazen studenls
entered the room.
(B) The!"e were three dozen students in the class after it had
begun.
(C) There were 24 students in the class after it had begu n.
(D ) The dozen people iri the room wet"e doubling as studc:::.-.ts.

17. (A) He expected the professor to contradict himself.


( B) He had cxpcctl.!d the professor to cancel the class.
( C) He was contrary with the professor.
(D ) He hadn"t expected the professor to cancel class, but he
did.

18. (A) In a butcher shop. (C) In a pharmacy.


(B) Inabakery. (:U) In an ice-cream store.

19. (A) He won the trophy.


(B)- Ht!'s a minister.
(C) He sprained his ankle.
(D) He broke his a rm.

20. (A) In the winter. (C) In September.


(B) In July. (D) In April.

21. (A) The speakers may cause Katie to be late.


(B) She will be here shortly.
(C) The spt!akers will probably be late if she doesn 't arrive
soon.
(D) She:! probably forgot the appointmen t.

466
1 1 1 1 1 / PRACTICETEST6/· 1 1 1 1 l

22. (A) Go to a party eith er Friday or Saturday night.


(B) Go to a party if the weather is good.
(C) Go to a party on both Friday and Saturday nights.
(D) Not go to a party because of the weather.

23. (A) Helen hates to eat fis h.


(B) Helen often fishes with her husband, but she doesn't like it.
(C) H elen hates her husband after he has been fishing.
(D ) Helen likes to eat fish, but her husband likes to fish too
much.

24. (A) Karl enjoys painting puzzles.


( B) Karl's expression puzzled the woman.
(C) Karl was confused.
(D) Karl expressed the woman's face in a puzzle.

25. (A) H e must exha u st t he runner.


(B) He was probably very tired after running.
(C) The rum made him sleepy.
(D) He must run after the thief.

26. (A) He can probably see her tomorrow afternoon.


(B) T omorrow at noon she will see him.
(C) She may see him now, but she'll be too busy tomorrow.
(D) He must pay his last bill if he wants to see her.

27. (A) Sally's friends have very bad habits.


( B) Sally doesn't have many friends because she..is spiteful.
(C) Sally has many friends although she has bad habits.
(D) Bad people are avoided by Sally.

28. (A) The houses are too simple to cost so much.


(B) It is easy to pay for a nice house.
(C) They don ' t have enough money for a new house because of
the high prices.
(D ) They can afford a new house now, but not next year.

467
1 1 1 1 1 /PRACTICETEST6/ 1 1 1 1 1

29. (A) It should have closed yesterday, but it will close in two
days.
( B) It closes two days from now.
( C) It closed two days ago.
(D) Registration for the class is late.

30. (A) She didn't know that they had changed plans.
( B) Their change in plans didn 't affect her plans.
(C) She didn't tell them when she changed plans.
(D) She didn 't know that they had changed the schedule of the
planes.

Go ON TO PART s

Part B

DIRECTIONS

In Part B, you wi ll hear longer conversations. After each


conversation. you will be asked some questions. The conversations
and q uestions will be spoken just one time. They will not be written
out for you, so you will have to listen carefully in order to
understand and remember what the speaker says.
When you ht:ar a question, read the fou r possible answers in your
test book and decide which one would be the best answer to the
question you have heard. Then, o n your answer sheet, find the
num ber of the problem and fill in the space that corresponds to the
letter of the answer you have chosen.

3 1. (A) He"s lost his job as a chef.


(B ) He c:m"t keep up with new trends.
( C) He hurt himself exercising.
(D) He is overweight.

468
1 1 1 1 1 / PRACTICE TEsT 6/ 1 1 1 1 1

32. (A) Limit fats a nd keep working o ut in the gym .


(B) Eat carbohydrates.
( C) Stop eating dessert.
(D) Do a breathing program.

33. (A) Eat yogurt.


( B) Read a good book.
(C) Snack on low-fat vegetables.
(D) Take long walks.

34. (A) Take ho me leftovers from a salad bar.


(B) Read books on dieting.
(C) Bring lunch from home.
(D) Skip lunch.

35. (A ) Co-authors of a book.


( B ) Librarian-researcher.
(C) Biographt:r-book buyer.
(D) Professor-student.

36. (A) France. (B) Chile. (C) Mexico. ( D ) Spain.


37. (A) Painter. ( C) Sociology professor.
(B) Wo rld traveler. (D ) Writer.

38. (A) Chik. (C) Germany.


(8) France. ( D ) C.rnada.

Go o !'. TO P \ RTc
1 1 1 1 1 /PRACTICE 'fEs.T 6/ 1 1 1 1 1

PartC

DIRECTIONS

In Part C, you will hear several talks. After each talk, you will be
asked some questions. The talks and questions will be spoken just
one time. They will not be written o ut fo r you, so you will have to
listen carefully in o rder to understand and re membe r what the
speaker says.
When you hea r a question, read the fo ur possible answers in your
test book and decide which one would be the best answer to the
q uestion you have heard. Then, on your answe r sheet, fi nd the
number of the problem and fill in the space that corresponds to the
letter of the a nswer you have chosen.

39. (A) There was no t much wind.


( B) There was no way of controlling them.
(C) It was hard to ge t off the ground.
(D) They were too heavy.

40. (A) Germany. (C) United States.


(B) France. (D ) England.

41. (A) A French clockmaker. (C) A French count.


(B ) Yo n Zeppelin. (D) Blimp.

42. (A) Germany. (C) U nited States.


( B) E ngland. (D) France.

43 . (A) The airships were used for wartime purposes.


(B) They were afraid because of the tragedy of the Hindenberg.
(C) The newer models were too sma ll.
(D) They were d ifficult to control.

44. (A) Elephants. (C) Bears.


(B) Rabbits. (D) Tigers.

470
1 1 1 1 1 / P RACTICE TEST 6/ 1 1 1 1 1

45. (A) Butterfl ies. ( C) Spide rs.


( B) Mice . (D) Turtles.

46. {A) Disease. (B) Speed . (C) Size. ( D ) Fur.

47. (A ) Beasts. (C) Carnivorous.


( B) Herbivoro us. (D) Dinosaurs.

48. (A) 180 millio n years ago.


(B) 60 million years ago.
(C) 16 million years ago.
(D) 150 million years ago.

49. (A) By excavating sites.


(B) By reconstructing skeletons.
(C) By observing t h e m closely.
( D) By living with them.

50. (A ) Scientists have stud ied them fo r centuries.


( 8 ) They were meat eating as well as plant eating.
(C ) They wande red the earth for mill ions of years.
(D ) They lived o n land, in the se a , and in the sky.

STOP. THIS IS THE END Of rHE LISTENING COMPREHENSION SECTION.


G O ON TO SECTION 2.

471
2 2 2 2 2 / PRACTICE TEST 6/ 2 2 2 2 2

SECTION 2
STRUCTURE AND WRITTEN EXPRESSION
Time: 25 Minutes
40 Q ues tio ns

Part A

DIRECTIONS

Questions l- 15 are incomplete sentences. Beneath each sentence


you will sec fo ur words or phrases. marked (A), (B). (C), and (D).
Choose the one word o r phrase that best completes the sentence.
Then, o n your answe r sheet, find the number of the question and fill
in the space that corresponds to the letter of the answer you have
chosen. Fill in the space so that the letter inside the oval cannot be
seen.

l. George did not do well in the class because _ __


(A) he studied bad
(B) he was not good studywise
(C) he was a badly studc nr
(0) he failed to study properly

2. This university's programs _ _ _ _ _ those of Harva rd.


(A) come second afte r
(B) are second o nly to
( C) are first except fo r
(D) are in second place from

3. The more she wo rked, _ __ __


(A) the less she achieved
( B) she achieved not e nough
(C) she did not achieve enough
(D) she was ach ieving less

472
2 2 2 2 2 /PRACTICE TEsT6/ .2 2 2 2 2

4. the best car to buy is a Mercedes Benz.


(A) Because of its durability and economy,
(B) Because it lasts a long time, and it is very economical,
(C) Because of its durability and it is economical,
( D) Because durability and economywise it is better than all
the others,

5. When Henry arrived home after a hard day at work,

(A) his wife was sleeping


( B) his wife slept
(C) his wife has slept
(D) his wife has been s leeping

6. He gave _ _ _ __
(A) to the class a tough assignment
( B) the class a tough assignment
(C) a tough assignment for the class
(D ) a n assignment very tough to the class

7. People all over the world are starving _ _ _ __


(A) gr eater in numbers
(B) in more numbers
(C) more numerously
(D) in greater numbers

8. It was not until she arrived in class _ __ _ _ realized she


had fo rgotten her book.
(A) and she
(B) when she
(C) she
(D) that she

473
2 2 2 2 2 /PRACIICETEST6/ 2 2 2 2 2

9. John has not been able to recall where _ _ _ __


(A) does she live
(B) she lives
( C) did she live
(D) lived the girl

10. Ben would have studied medicine if he - - - - - to a


medical sch ool.
(A) could be able to enter
(8) had been admitted
( C) wa~ admitted
(D) were admitted

11. He entered a univ~rsity _ _ _ __


(A) when he had sixteen years
(B) when sixteen years were his age
(C) at the age of sixteen
(D) at age sixteen years old

12. The jurors were told to - -- - -


(A) talk all they wanted
(B) make lots of expressions
( C) speak freely
(D) talk with their minds open

13. Those students do not like to read novels, - - - - - t-:-xt


books.
(A) in any case
(8) forgetting about
( C) leaving out of the question
(D) much less

14. He looked forward to the new venture.


(A) eagerly
(B) with great eagerness
( C) eagernessly
(D) in a state of increasing eagerness

474
15. The families were told to evacuate their houses immediately

(A) at the time when the water began to go up


(B) when the water began to rise
(C) when up was going the water
(D) in the time when the water raised

GO ON TO PART B

Part B

DIRECTIONS

In questions 16-40, each sentence has four underlined words or


phrases. The four underlined parts of the sentence are marked (A),
(B), (C), and (D). Identify the one underlined word or phrase that
must be changed in order for the sentence to be correct. Then, on
your answer sheet, find the number of the question and fill in the
space that corresponds to the letter of the answer you have chosen.

16. Most Americans would not be happy without~ color


A B C
television, two cars, and working at an extra job.
D

17. The lion has long been ~ symbol of strer.gth, power, and
A B C
it is very cruel.
D

18. All the scouts got theirselves ready for the Jong camping trip
A B C
by spending their weekends living in the open.
D

475
2 2 2 2 2 /PRACTICETEsT6/ 2 2 2 2 2

19. Nobody had known before the presentation that Sue and her
A B
si5ter will receive the awards for outstanding scholarship.
C D

20. In 1927, Charles Lindbergh was the first to fly solo nonstop
A B
fro m New Yo rk to Paris in such sho rt time.
C D

21. Until his last class at the university in 1978, Bob always
A
turns in all of his assignments on time.
B C D

22. When I last saw Janet, she hurried to her next class on
A B
the other side of the campus and did no t have time to talk.
C D

23. Before we returned from swimming in the river near the camp,
A
someone had stole our clothes, and we had to walk back with
B C
o ur towels around us.
D

24. Patrick was very late getting home last night, and
A
unfortunately for him, the dog barking woke everyone ~·
B C D

25. He has been hoped for a raise for the last fo ur months, but his
A B
boss is reluctant to give him one.
C D

476
2 2 2 ·2 2 /~enraTEsr6/ · 2 .2 2 2 2

26. After driving for twenty miles, he suddenly realized that he


A B
has been driving in the wrong direction.
C D

27. The Department of Foreign Languages are not located in the


T 6 c
new building opposite the old one.
D

28. The Nobel Prize winner, accompanied ~ her husband and


A
children, are staying in Sweden until after the presentation.
6 -CO-
29. Neither of the scout leaders know how to trap wild animals
A B
or how to prepare them for mounting.
C D

30. Those of you who signed up for Dr. Daniel's anthropology


A B
class should get their books as soon as possible.
C D

31. I put my new book of zoology here on the desk a few minutes
A B
~· but I cannot seem to find it.
C D

32. Marta being chosen as the most outstanding student on her


A B . C
campus made her parents very happy.
D

477
2 2 2 2 2 /PRACTICE TEsT6/ 2 2 2 ~~ 2

33. Jane said she would borrow me her new movie camera if I
- -A- - B-
wanted to use it on my trip to Europe.
C D

34. When Cliff was sick with the flu, his r•. :>ther made him
A B
to eat chicken soup and rest in bed.
C D

35. My cousin composes not only the music, but also sings
A B
the songs for the major Broadway musicals.
C D

36. The geology professor showed us a sample about vc!canic rock


A B C
which dated back seven hundred years.
D

37. The giti whom my cousin married was used to be a chorus girl
A B C
for the Rockettes in Radio City Music Hall in New York.
D

38. Ralph has called his lawyer last night to tell him about his
A B
problems, but was told that the lawyer had gone to a lecture..
C D

39. Some bumper stickers are very funny and make us laugh, yet
A B
another can make us angry because of tlieir ridiculousness.
C D

478
2 2 2 2 2 /PRACTICE TEST6/ -2 2 2 2 2

40. The results of the test proved to Fred and me that we neede d
A B C
to study harder and watch less movies on television if we
D
wanted to receive scholarships.

STOP. THIS IS THE END OF THE STRUcruRE AND WRITIEN EXPRES-


SION SECTION. I F YOU FINISH BEFORE TIME IS UP, CHECK YOUR WORK
ON PARTS A AND B OF THIS SECTION ONLY. DO NOT WORK ON ANY
OTHER SECTION OF THF. TEST.

479
3 3 3 3 3 / PRACTICE TEST 6/ 3 3 3 3 3

SECTION 3
READING COMPREHENSION
Time: 55 Minutes
SO Questions

DIRECTIONS

In this section, you will read a number of passages. Each o ne is


followed by approximately ten questions about it. For q1,1estions
1- 50, choose the one best answer, (A), (B), (C), or (D ), to each
question. Then, find the number of the question on your answe r
sheet, and fi ll in the space tha t correspo nds to the letter of the
answe r you have chosen. Answer all questio ns following a passage
on the basis of what is stated or implied in tha t passage.

Questions 1 through JO are based on the following reading passage.

The First Amendment to the American Constitution


declares freedom of the press to all people. Although this
right was not officially adopted until 1791, the fa mous
Zenger trial of 1735 laid the groundwo rk for insuring this
(5) precious freedom .
John Peter Zenger emigrated as a teenager from Ger-
ma ny. Jn 1733, he began publishing the New York Weekly
Journal. The following year, he was arrested for writing a
story about the crown-appointed governor of New York.
(10) While Zenger was imprisoned for nine mo nths, his wife
du tifully published the newspaper every day, bravely telling
the truth about the corrupt government officials sent by the
king to govern the colonies.
Finally Zenger 's long-awai ted trial took place. T he hos-
(15) tile judge dismissed Ze nger 's local lawyers, making it
necessary fo r his wife to seek o ut Andrew Hamilton, a
prominent Philadelphia lawyer. Persuaded by Hamilton,

480
3 3 3 3 3 / PRACTICE TEST6/ 3 3 3 3 3

the j u ry brave ly returned a not-guilty verdict, defying t he


judge's orders fo r a conviction.
(20) As a result of determination and bravery on the part of
the colonists, a lasting victory for freedom of the press was
gained by a young immigrant.

1. John Peter Zenger was a


(A) corrupt governor of New York
( B) famous lawyer
(C) brave newspaper publisher
(D) hostile judge

2. What political problem existed in the colonies at that time?


(A) Government officials were corrupt.
( B) Newspapers exaggerated the truth about the political
officials.
(C) Lawyers were hostile to witnesses.
(D) AH newspaper publishers were imprisoned.

3. How long did it take after the Zenger trial before the concept of
freedom of the press was officially adopted?
(A) 9 months (C) 56 years
(B) l year (D) 58 years

4. All of the following are true EXCEPT


(A) despite Zenger's imprisonme nt, his newspaper continued
to be published
( B) Andrew H amilton encouraged the jury to fight for freedom
( C) the jury obeyed the judge's orders and convicted Zenger
(D) the king col'!trolled the colonies through his own appo inted
rulers

5. Why was Peter Zenger arrested?


(A) He emigrated from Germany.
( B) His wife published his newspaper for him.
( C) He wrote a story about the governor of New York.
(D) He persuaded a jury to defy the judge's orde rs.

48 1
3 3 3 3 3 /PRACllCE TEST 6/ 3 3 3 3 3

6. It can be inferred that the judge nas hostile toward Peter


Zenger because the judge
(A} represented the ideas of the king
(B) hated newspaper publishers
(C) didn't like interference with the Constitution
(D) had appointed the gove rnor abo ut whom Zenger wrote

7. The word " def1ing" in line 18 is closest in meaning to


(A) altering (C) disregarding
( B) defecting (D) defending

8. In line 11, the wo rd " dutifully" is closest in meaning to


(A) faithfully (C) unfortunately
( B) care lessly ( D) vigorously

9. The main idea of this passage is


(A) Andrew Hamilton gave Americans freedom of the press
(B) Peter Zenger's persistent fight paved the way for freedom
of the press
(C) judges don' t always get juries to agree with them
(D) Peter Zenger's trial prepared the way for jurors to defy
judges' orders

10. The passage indicates that the governor was appointed by the
monarch of another country in lines
(A) 3-5 (B) 6-8 (C) 8-9 (D) 14-16

Questions 11 through 20 are based on the following passage.

The period commonly known as the R enaissance ( 1400-


1600) began in Florence, Italy. It represented a renewed
interest in Greek and Roman art and literature . The
grea test achievements m art during this period were the
(5) perfection of depth perspective, use of colors, and effects of
light and shadow. Artists across Europe improved o n the
ancient artists' techniques as no o ther period had done. The

482
3 ·3 3 3 3 / PRACTICE TusT 6-/ 3 3 3 3 3

learned were studying G reek a nd R oman to read t h e


an cient literary classics. There were many advances in
(10) science and technology, discoveries in the New World, and
changes in religion. The growth of universities throughout
Europe helped create a more educated middle class that
was to take over running the government within the follow-
ing centuries. Europe had come out of the Dark Ages. This
(15) idea of rebirth in learning characterized other epochs in
history in different parts of the world.
In A.D. 800, Charlemagne became king of the Franks and
initiated the Carolingian renaissance, which lasted until the
end of the ninth century. This period saw beautiful and
(20) more modern cities patterned on Roman architecture.
Charlemagne stimulated learning and the development of
the arts, sponsored a palace academy, established a curricu-
lum in schools for the nobility, created libraries (a carryover
from Alexandrian Egypt of 323 B.C. ), and changed writing to
(25) an improved style of script.
Kievan Russia also enjoyed a century of rebirth some two
hundred years later under the able rule of Yaroslav the
Wise. Like Charlemagne, he founded schools, established
libraries, and brought about many architectural achieve-
(30) ments.

11. Which was the earliest period of rebirth mentioned?


(A) Russian (C) Carolingian
(B) Italian (D) Roman

12. Which city did Charlemagne look upon as a model for his
architectural improvements?
(A) Kiev (B) Rome (C) Carolingian (D ) Frank

483
3 3 3 3 3 / PRACTICETEST6/. 3 3 3 3 3

13. All of the following are mentioned as characteristics of periods


of renaissance EXCEPT
(A) maintaining the status quo
( B ) improved education
(C) architectural advances
(D) the creation of libraries

14. What can we assume about Yaroslav?


(A) He was demented.
(B ) He was a competent leader.
(C) He was ine pt.
(D) He was cruel.

15. The word "carryover" in line 23 means most nearly the same as
(A) remnant (C) innovation
( B ) residue (D) barbarism

16. According to the passage, what do all three periods have in


common?
(A) A concern for education and learning
(B) A desire for advanced science a nd technology
(C) An aversion to new libraries
(D) An inte rest in traveling throughout the world

17. What does the word " renaissance" mean in the context of these
three civilizations?
(A) Improving on the basic principles of past cultures
(B) Trying to do everything as the ancients had done
(C) Helping to maintain Roman culture at any cost
(D) Tearing down existing buildings and using Roman architec-
tural techniques for new ones

18. Which of the renaissance periods lasted the longest?


(A) Italian (C) Alexandrian
(B) Carolingian (D) Kievan
3 3 3 3 3 /PRACTICE TEST6 / 3 3 3 3 3

19. Which renaissance had the most widespread and lasting effect
on future generations?
(A) Italian (C) Alexandrian
(B) Kievan (D) Carolingian

20. The main idea of this passage is that


(A) throughout history there has been a rebirth of ideas and an
effort to copy without creating anything new
(B) o nly western Europe was interested in rebirth
( C) the periods of r ebirth saw greater advances for each
culture
(D) rebirth of a culture does not depend on outside influences

Questions 21 through 29 are based on the following passage.

Gelatin is a protein substance that comes from the skins


and bones of animals. Most people know it as the substance
used to make a jellylike salad or dessert. Not only is it useful
in making these foods, but it is also beneficial to the
(5) consumer because of its high protein content. Gelatin is also
commonly used in the photographic industry and in making
medicinal capsules.
The process for producing gelatin is a long and complex
one. In the processing of gelatin made from bones (which
(10) varies slightly from that of gelatin made from skin), the
grease first must be eliminated. Then the bones are soaked
in a solution of hydrochloric acid in order to rid them of
minerals and washed several times in water. Next, the bones
are placed in distilled water, heated to over 90°F for a few
(15) hours, placed in fresh distilled water, and then heated again
at a little over 100°F. A fluid forms from this heating, and it
is concentrated, chilled, and sliced. Finally, it is dried and
ground. In its final form, gelatin is white, tasteless, and
odorless.

485
3 3 3 3 3 /PRACTICE1'EsT6/ 3 3 3 3 £]
21. It can be infe rred from this reading passage that
(A) one could easily make gelatin at home
(B) it is necessary to add minerals to the gela tin
(C) fat aids in ma king good gelatin
(D ) gelatin is useful for e lderly and ill people because it is easy
to chew and high in protein

22. T he word " fluid" in line 16 is closest in meaning to


(A) liquid (C) hard mate rial
(B) distilled wate r (D) substance

23. Which of the following is true?


(A ) Gelatin made from skin is produced in the same way as
that made fro m bones.
(B) Grease probably does not aid in producing gelatin.
(C) The chemical used in making gelatin comes off the surface
of the bones by rinsing with water.
(D) When the gelatin is dried, it is in powder fo rm.

24. Which of the following would be the best title for this passage ?
(A) The Process of Making G elatin
(B) Proteiu Foods
( C) Uses fo r Bones
(D) A Great D essert

25. All of the following industries are mentioned as using gela tin
EXCEPT
(A) the lawn care industry
(B) the photographic industry
(C) the pharmaceutical industry
(D)"' the food industry ·

26. According to the passage, why is eating gelatin healthy?


(A) It does not damage the teeth.
(B) It is low fat.
( C) It is protein rich.
(D) It has no animal byproducts.

486
3 3 3 3 3 / PRACTICE TusT.6/ 3 3 3 3 3

27. The word "ground" in line 18 is closest in meaning to


(A) refrigerated ( C) putrified
(B) pulverized (D) dirtied

28. Why would gelatin be useful for medicine capsules?


(A) It tastes good.
(B) It is a natural substance that is easy to digest.
(C) It is easy to make and thus inexpensive.
(D) It won't melt at high temperatures.

29. The word " that" in line 10 refers to


(A) gelatin (B) processing (C) skin (D) bones

Questions 30 through 41 are based on the following passage.

In recent years, scientific a nd technological developments


have drastically changed human life on our planet, as well as
our views both of ourselves as individuals in society and of
the universe as a whole. Perhaps one of the most profound
(5) developments of the 1970s was the discovery of recombinant
DNA technology, which allows scientists to introduce ge-
netic material (or genes) from one organism into another.
In its simplest form, the technology requires the isolation of
a piece of DNA, either directly from the DNA of the
(10) organism under study or artificially synthesized from an
RNA template by using a viral enzyme called reverse
transcriptase. This piece of DNA is then ligated to a
fragment of bacterial DNA which has the capacity to
replicate itself independently. The recombinant molecule
(15) thus produced can be introduced into the common intesti-
nal bacterium Escherichia coli, which can be grown in very
large amounts in synthetic media. Under proper conditions,
the foreign gene will not only replicate in the bacteria, but
also express itself, through the process of transcription and
(20) translation, to give rise to large amounts of the specific
protein coded by the foreign gene.

487
3 3 3 3 3 /PRACTICETEST6/ 3 3 3 3 3

The technology has already been successfully applieri to


the production of several therapeutically important biomol-
ecules, such as insulin, interferon, and growth hormones.
(25) Many other important applications are under detailed
investigation in laboratories throughout the world.

30. Recombinant DNA technology consists primarily of


(A) producing several therapeutically important biomolecules
(B) giving rise to large 2&:.Junts of protein
(C) introducing genetic material from one organism into an-
other
(D) 1.tsing a viral enzyme called reverse transcriptase

31. The word " profound" in line 4 is closest ;u meaning to


(A) significant (C) dangerous
(B) boring (D) secret

32. In line 8, the word "isolation" is closest in meaning to


(A) destruction (C) segregation
(B) duplication (D) study

33. Recombinant DNA technology has been used in the production


of all of the following biomolecules EXCEPT
(A) growth hormones (C) interferon
(B) Escherichia coli (D ) insulin

34. In line 10, " artificially" is closest in meaning to


{A) correctly (C) artistically
(B) synthetically (D) carefully

35. The word " ligated" in line 12 is closest in meaning to


(A) intersected (B) cut (C) elevated (D) bound

488
3 3 3 3 3 /PRAcncE TEsT 6/ 3 . 3 3 3 3

36. Which of tile following is NOT true?


(A) The foreign gene will replicate in the bacteria, but it will
not express itself through tran~cription and translation.
(B) The bacterium Escherichia coli can be grown in large
amounts in synthetic media.
(C) Research continues in an effort to find other uses for this
technology.
(D ) Recombinant DNA technology is a recent development.

37. In line 13, the word " fragment" is nearest in meaning to


( A ) particle (C) opposite
(B) reproduction (D) large piece

38. T he word "capacity" in line 13 is nearest in meaning to


(A) hormones (C) ability
( B} technology ( D) space

39. Expression of a gene in Escherichia coli requires


{A) the viral enzyme reverse transcriptase
( B) the processes of transcription and translation
(C) productio n of insulin and other biomolecules
(D) that the bacteria be grown in a synthetic media

40. The term " recombinant" is used because


(A) by ligation, a recombinant molecule is produced, which has
the capacity of replication
( B) the technique requires the combination of several types of
technology
( C) by .ligation, a recombinant protein is produced; part of
whole amino acids come from each different organism
(D) Escherichia coli is a recombinant organism

41. The word " replicate" in line 14 is closest in meaning to


(A) reproduce (B) join (C) reside (D) coexist

489
3 3 3 3 3 /PRACTICE TEST 6/ 3 3 3 3 3

Questions 42 through 50 are based on the following passage.

Of the six outer plane ts, Mars, commonly called the Red
Planet, is the closest to Earth . Mars, 4,200 miles in diameter
and 55 percent of the size of Earth, is 34,600,000 miles from
Earth, and 141,000,000 miles from the Sun. It takes this
(5) planet, along with its two moons, Phobos and Deimos, 1.88
years to circle the Sun, compared to 365 days for the Earth.
For many years, Mars had been thought of as the planet
with the man-made canals, supposedly discovered by an
Italian astronomer, Schiaparelli, in 1877. With the United
(10) States spacecraft Viking I's landing on Mars in 1976, the
man-made canal theory was proven to be only a myth .
Viking I, after landing o n the soil of Mars, performed
many scientific experiments and took numerous pictures.
The pictures showed th at the red color of the planet is due
(15) to the reddish, rocky Martian soil. No biological life was
found, though it had been speculated by many scientists.
The Viking also monitored many weather changes including
violent <lust storms. Some water vapor, polar ice, and
permafrost (frost below the surface) were found, indicating
(20) that at one time there were significant quantities of water on
this distant planet. Evidence collected by the spacecraft
shows some present volcanic action, though the volcanoes
are believed to be dormant, if not extinct.

42. All of the following are true EXCEPT


(A) Mars has two moons
(B) it takes longer for Mars to circle the sun than it takes Earth
(C) Martian soil is rocky
(D) Mars is larger than Earth

43. Man-made canals were supposedly discovered by


(A) Viking I (C) Phobos
(B) Schiaparelli (D) Martian

490
3 3 3 3 3 /PRA.cnCETEST6/ 3 3 3 3 3

44. The word "supposed ly" in li ne 8 is closest in meaning to


{A) actually (C) formerly
(B ) presumably (0) unquestio nably

45. Mars has been nicknamed


(A) Viking I (C) Deimos
(B) the Red Planet (0 ) Martian

46. In line 11, the word " myth" is closest in meaning to


(A) fact (B) event (C) legend (0) enigma

47. The Viking I exploration accomplished all of the following


EXCEPT
(A) performing scientific experiments
(B) collecting information showing volcanic action
(C) monito ring weather conditions
(0) discovering large quantities of po lar ice and permafrost

48. What is the main idea of this passage?


(A) Fairly recent studies of this planet reveal data that contra-
dict previously held theories.
( B ) Very little of the Martian landsca pe has changed over the
years.
(C) Scientists a re only speculating about the Red Planet.
(D) Scientists are no longer interested in the planet because
there is no life on it.

49. The word " monitored" in line 17 is nearest in meaning to


(A) programmed {C) censored
(B) televised (D) observed

50. The word " dormant" in line 23 is closest in mea ning to


(A) dangerous (C) erupting
. (B) inactive (D) significant
STOP. Tms IS THE END O F THE EXAMINATION. I F YOU FINISH BEFORE
TIME IS UP, CHECK YOUR WORK IN THIS SECTION ONLY. Do NOT WORK
ON ANY OTHER SECTION OF THE TEST.

491
PART V: Listening Comprehension Scripts,
Answers, and Explanations
for Practice Tests 1 through 6
HOW TO USE PART V
Part V contains answers and explanations for the six practice
tests, scripts for the listening comprehension sections, answer keys
t hat are cross-referenced to grammar review pages in Part III, a nd
scoring charts to help you see your strengths and weaknesses. In
order to improve your score, you MUST analyze your mistakes and
strive to avoid making the same errors again. MAKE FULL USE OF 11-IE
FOLLOWING PAGES TO IMPROVE YOUR PERFORMANCE. Follow this
step-by-step procedure.
• First turn to the answer keys to check your results. Then turn to
the A nalysis-Scoring Sheet for the test you have taken and fill in the
number of questions that you got CORRECT in each section. Follow
the d irections to figure your total converted score. Which section
did you do best in? Which section did you do worst in? The section
i.n which you received your lowest score is the section which you
must work hardest on improving.
• If you did poorly on the listening comprehension, study the script
r.arefully, comparing the questions that you heard to what you read
on paper. Listen ro the wpe again to see if you can now hear more
clearly. If there are vocabulary items. idiomatic expressions. or
grammatical constructions that a re causing you to make mistakes in
listening, look them up and study them again. Use your dictionary
for expressions that were not covered in this guide.
• If you did poorly in the grammar section, you must look back at the
rule and study it again until you can recognize such a problem
immediately. Most grarr.mar explanations contain a shortened
version of the rule, arid the answer keys are cross-referenced by
page number. The page numbers refer to rules and examples in Part
III that you should study again. Sometimes there are several page
numbers because the sentence contains several different problems.
• For the reading comprd.ension questions. look at the explanations
and refer back to the reading itself to understand why you missed the
question. The explanation will often tell you in which sentence the
material necessary to answer the question can be fo und. Remember
that generally the words in the questions and the words in the text
495
I
496 CONVtKIEU sc01u: SH EET

are not txactly the same::. Be sure that you understand the:: rnea11i11g
of eac h reading selection and question and that you learn any nt:w
I
vocabul<i 1 y w o rds 1hat you r un across.
• A livay:.· look ba<:k ut q11es1iu11s 1ha1 you missed co sec whctha you
I
co ulJ auswer t hem wrrcctly nuw that you '.iave reswdied.

CONVERTED SCORE SHEET

To use this chart, find the number in the raw score column th at
I
I
corresponds to your to tal CORHEC.T answers o n each section. The
converted score in euch section is listed to the righ t of the raw score.
Transfer each of the three converteJ scores to the..: Practice Test
Analysis-Scoring Shet!t that precedes the explana tion section for
each prnctice test. Follow the directions given there to de termine
your total conven ed score. The highest possible score on th e tests in I
this guide is 673; the lowest is 223. On the actual TOEFL the scores
may ra11ge from 700 to 200.
Converted Scores Converted Scores
I
Raw Sec tion Seclion Scclion
Scan:
50
I
68
2
67
3
Raw Sec1ion Seciion Section
Score
40
I
56
2
67 55
3
I
--·- - - -
49
48
66
64
66
65
39
38
56
55
66
64
54
53
I
47 63 63 37 54 63 52
I
46 62 61
·- 36 53 61 51

-
45
44
61
- --
60
60

59
35
- - --·-
34
52
52
59
58
50
49 I
43

42
59
58
58

57
33
32
51

50
57

55
49
48 I
I
41 57 56 31 50 54 48

I
CONVE RTE O SCOJU : SI 11.:1:: l' 497

ConvcncJ Scor i::s Convc n t:J Sco res


Raw Sect io n Section Section R a w Sect ion Sccl io n S.:ct io n
Score 2 3 Score 2 3
30 !
~-:~37
49 53 47 15 40 35
--1 - ---
~~ 52 47 14 39 34
28 48 51 .:5 13 38 36 33
27 48 50 45 12 37 35 32
----
26 47 ~-49 - i~- II 36 34 31
25 46 48 I 44 10 34 33 JU
- -- - - - - _j
24 46 47 I 43 9 33 :n 29
23 45 4() 42 8 32 30 28
-
22 4 '1 45 41 7 31 29 28
-- ---- -
21 4-t 44 41 6 30 28 27
-- - --
20 ·13 43 40 5 29 26 26
19 43 42 39 4 28 25 25
- -- - - -
18 42 41 38 3 27 24 24
17 41 40 37 2 26 22 23
16 41 39 36 l 25 20 22
PRACTICE TEST 1
ANSWER KEY FOR PRACTICE TEST 1

After some answe rs in this answer key, you will fi nd numbers in


italic ty pe. These are page numbers in Part Ill where you wi ll find
review material fo r these questions. Although any o ne questio n may
invo lve several d ifferent rules and concepts, these page numbers
refer to importanc areas you sho uld review if you have missed a
q uestion o r are not sure of the mate ria l involved. Make full use of
these page n umber refe rences and of the index to di rect your
personal review.

Section l: Listening Comprehension


1. ( B) 11. (C) 21. (C) 31. ( B) 41. (0 )
2. (A) 12. (B) 22. (B) 32. ( B) 42. (C)
3. ( B) 13. (D) 23. (D ) 33. (B) 43. (B)
4. (C) 14. (D ) 24. (A) 34. ( B) -M. (B)
5. (C) 15. (C) 25. ( B) 35. (C) 45. (C)
6. (B) 16. (A) 26. (B) 36. (A ) 46. (A )
7. (A) 17. (A) 27. (C) 37. ( B) 47. (B)
8. (C) 18. (C) 28. (C) 38. (D) 48. (A )
9. (B) 19. (D) 29. (C) 39. (A ) 49. (C)
10. (D) 20. (C) 30. ( B) 40. (D ) 50. (A)

498
SCORING PRACTICE TEST I 499

Section 2: Structure and Written Expression


l. (C)39 21. ( D) 62
2. ( D ) 52 22. ( B) 44-49
3. ( D) 74-75 23. ( B) 65--07
4. (C) 102-103 24. ( 0) 204-206, 212-215
5. (C) 132 25. (D ) 100-101
6. ( B) 112. 153- 154 26. (A ) 167-170
7. (Il) 212-215 27. (B) 224-226
8. (D) 52-53, 100-101 28. (C) 104
9. l D) 183-184 29. (C) 54
IO. ( 0) 98 30. (D ) 100-101
l 1. (C) 204-206 31. (D ) 123- 124
l2. ( B) 44-49, 74-75 32. (A) 47
ij. (C) 94 33. (0) 44-49
14. (C) 44--19 34. (A) 76
l5. (D) 96-98 35. (A) 44-49
16. (C) 280-281 36. (A) 152
l7. ( A) 44-49 37. (B ) 44-49
l8. l O ) 229 38. (B) 277-278
19. (DJ 62 39. (A) 280-281
20. (A) 102-103 40. (A) 39

Section 3: Reading Comprehens ion


l. tB) 11. ( 0) 21. (D) 31. (A ) 41. (B)
2. (A) 12. (0 ) 22. (0) 32. (A ) 42. (C)
3. (B ) 13. (A) 23. (B) 33. (A ) 43. (A)
4. (A) 14. (B ) 24 . ( B) 34. (B ) 44. (B )
5. (D ) 15. (A) 25. ( D)· 35. (D ) 45. (B)
6. ( B) 16. (B) 26. (C) 36. (B) 46. (A)
7. (B) l 7. (C) 27. (A) 37. (A ) 47. (A)
8. (C) 18. (C) 28. ( B) 38. (A) 48. (C)
9. (A) 19. (B) 29. ( D) 39. (C) 49. (A )
1.0. (B) 20. (B ) 30. (B) 40. (C) 50 . (C)
500 SCORING PRACTIC'F. fEST I

PRACTICE TEST l: L\NALYSIS-SCORING SH EET

Use the chart below to spot your strengths and weak nesses in
each test section and tu arrive a t yo ur total converted score. Fill in
your number of correct :i.nswers for each "Sectio n in the space
provided. Refer to the Converted Score Sheet o n page 496 to find
yo ur converted score for each sectio n and enter those numbers on
the chart. Find the ~ um oi your converted scores, multiply that sum
by 10. and divide by 3.
Ex.ample: If raw scores :.ire then converted scores are
Section 1: 33 51
Section 2: 26 49
Section 3: 38 53
.:ium of Converted Scores 153
Times 10 = l.530
Divided by 3 = SIO = Total Converted
Score
This wi ll give you the approxima te -;core that yo u would obtain if
this were a n actual TO EFL. Remember that your score here may
possibly be higher than the score that yo u might receive o n an actual
TOEFL simply because you are studying the clemt:n ts of the test
shortly before taking each test. The score is intended o nly to give
you a general idea of approxima tely what your actual score will he.

Total Total Converted


Possible Correct Score
Section I: I

Listening Comprehension 50
!
i
---1
I
I
Section 2:
~o
Structure and Written Expression I -1I
'

Section 3:

-· - ~
50
Reading Comprehension

T OTALS 140
I _J
Sum of Converted Scores
T imes 10 =
----
Divided l•y 3 =
-- --
= Total Converted Score
SECTION I: LISTENING COMPREHENSION SCRIPT 501

SECTION 1: LISTENING COMPREHENSION SCRIPT

Part A

L Man: I hear Jan isn' t teaching here this term.


Woman: T hat's right. She was fired.
f hird Voice: What does the woman say about Ja.n ?

Nancy, I heard you were late for class this


mo rning.
Woman: r overslept and missed the bus.
Third Voice: Why does the woman say she was late?

~ Woman: I heard o n the radio that the eas tbound Janes of


Interstate 4 are closed.
\fan: Yes, a tractor-trailer jackknifed and caused a
huge pileup.
Third Voice: What are the speakers discussing?

-+. \Jlan: What do you think of Professor Conrad's class?


Wom an: Well, his lectures are interesting enough, but
think he could choose more appro priate ques-
tions fo r the tests.
Dlird Voice: What does :he woman say about Professor Con-
rad's class?

5 Woman: Are you going to watch the movie on TV tonight?


vtan: No, I th ink I'll watch the soccer game and then
the docume ntary o n volcanoes.
Third Voice: What does the man say is the fi rst program he is
planning to watch?
502 ANSWERS AND EXPLANATIONS FOR PRACTICE TEST I

6. Man: Where did Suzanne come from?


Woman: She wa~ born m Switzerland and grew up in
Sweden. but she's a citizen of England.
Third Voice: Which country does the woman say is Suzanne's
present home?

7. Woman: Karen is entering Stetson University th is fall.


Man: So she did apply.
Third Voice: What had the man assu med about Karen ~

8. Man: Why are you weari ng that cream all ove ,· your
arms?
Woman: I ate wild berries at the picnic last week, and
broke out in a rash.
Third Voice: What does the woman say happened to ne:?

9. Woman: Would you please spell your namt! fo r me. sir?


Man: Sure. W-1 -Jouble T-N-E-R.
Third Voice: How J oes the man say he spells his last name?

10. Woman: I have to go o ut of town for a :nc::eting to;1 ·orrow,


and I need somebody to work for me.
Man: Sure. I could use the extra hours!
Third Voice: What is the man probably going co do?

l l. Woman: Louie, how did your football team do la;;: sea ...:.m?
Man: We won three, Jost five, and tied twic•;.
Third Voice: How many games does the man say his team tier'~

12. Woman: Do you know wh~·t happened to Sally?


Man: She couldn't find the classroom until after the
class had begun.
'Third Voice: What does the man say happened to Sally?
503
SECTIO~ I : LIST"'tHl-10 COMPREHENSI01~ SCRIP'T

Did April visit you in the hospital when you were


13 . Woman:
ill?
No, but it was certainly kind of her to send me
Man:
flowers.
·1 hird Voice: What does the n1:!n say about the flowers?

14. Man: William looked very tired this morning.


Woman: He drove George's car from Georgia to New
York without stopping to sleep.
Tnird Voice: What does the woman mean?

15. W0rr.a.:: How was the turnout at the meeting last night?
Ma11: Fc::wer people came than I had expected.
Third Voice: What does the man say abo ut attendance at the
meeting?

16. Woman: Wa~ Harry rnt;cessful at his new venture?

Man: He spent five hours knocking o n doors, but he


didn't sell a single magazine.
Third Voice: What does the man say about Harry?

17. Man. Did Frank have his house repaired?


Woman: The contractor said the repairs would be very
expensive, but he decided to have the work done.
Third Voice: What does the woman say about the repairs to
Frank's house?

18. Wopian: What did you do last night?


Man: I should have studied. but I was too tired.
Third Voice: What does tl1e man say he did last night?

19. Man : Do you think Gloria will come with us?


Woman: I understand she hasn't gone to a movie in years.
Third Voice: Whai J oes the woman say about Gloria?
504 ANSWERS AND EXPLANATIONS FOR PRACTICE TEST l

20. Man: What happened to Harvey today?


Woman: His face turned bright red when the teacher
asked him a question.
Third Voice: What does the woman say about Harvey?

21. Woman: Good afternoon, I'm Roseanne, your flight atten-


dant. Welcome a board.
Man: Hello. I've got seat A8. I hope it's by a window so
that I can see the view.
Third Voice: Where did this conversation most probably take
place?

22. Man: I heard ~a;1e isn't going to be working this


summe r.
Woman: That's right. She's taking sick leave.
Third Voice: What does the woman say about Jane ·?

23. Man: A change has sure come over you.


Woman: I fina lly had my annual review meeting with my
boss. It couldn 't have gone better.
Third Voice: What does the woman mean?

24. Man: Doesn't Professor Jones realize there are o nly


two days before the test?
Woman: He apologized for not announcing the test ear-
lier.
T hird Voice: What does the woman say about the professor?

25 . Wo man: Stacey had a disagreement with her bosc; yester-


day, didn' t she?
Man: She says she is leaving her job for good.
Third Voice: What does the man say about Stacey?
SECTION I: LISTENING COMPREHENSION SCRIPT 505

26. Man: Did John stop at the store?


Woman: No, he had some money, but not enough to buy
groceries.
Third Voice: What does the woman say about John?

27. Man: I can't believe there were no emp}' seats at the


rally.
Woman: They expected eighty people., but twice that many
showed up.
Third Voice: How many people does the woman say attended
t he rally?

28. Woman: Are we doing an ything today?


Man: We were supposed to meet Fred and Mary at the
movies, but we' re broke.
Third Voice: What does the man mean ?

29. Man: I unde rstand Ana is mad at T ed.


Woman: Yes, he refused to go to t he banquet even th~rngh
he was going to receive an award.
Third Voice: Why does the woman say Ana is angry at Ted?

30. Woman: How does Mike like his coffee?


Man: He likes sugar in it. but nothing else.
Third Voice: What does the man say about Mike?

Part B

Questions 31 through 34 are based on the following conversation.

Man: How lo ng have you been out of the country, miss? Where
did you go ?
Wo man: l spent three weeks in Switzerland and one week in
Gn.· !~'.:e .
506 ANSWERS AND EXPLANATIONS FOR PRACTICE TEST I

Man: Did you spend any time in agricultural areas there?


Woman: No, I stayed mostly in the cities and spe nt one day at the
beach in Corinth.
Man: . Do you have any plants, meat, or alcoholic beverages to
declare?
Woman: I have only two bottles of wine.
Man: What else did you buy?
Woman: A couple of fes tive costumes, books, and native arts and
crafts.
Man: How much did you spend on your purchases while you
were away?
Woman: About $300.
Man: Please open this small suitcase for me .... O K. give this
card to the official at the red desk.

31. Where did 1his conversation most likely take place?


32. How many countries did the woman visit?
33. What does the man ask the woman to do?
34. What did the woman have to declare?

Questions 35 through 38 are based on the following conversation.

Woman: I hear that Paul Schmidt has written a new novel.


Man: Yes, it's a science fiction piece called Martian Renaissance.
Woma n:Sounds intriguing. What's the plot like?
Man: It deals with a five-man, one-woman crew on a three-year
mission to Mars.
Woman: Is their mission successful?
Man: Well, in some respects it is. They have a series of
incredible adventures once they land.
Woman: Do they meet any real Martians?
Man: Yes, they are even held captive by them.
SECllON 1: LISTENING COMPREHENSION SCRIPT 507

Woman: What do the Martians look like? Aie they little green
men?
Man: You'll have to read the book to find out.

35. What is the name of Paul Schmidt's new book?


36. What type of book is it?
37. How long did the mission to Mars take?
38. Which of the following is NOT mentioned?

PartC

Questions 39 through 42 are based on the following lecture about the


game ofjai alai.

Although played quite well in Florida and Latin America, jai alai
is not an American game. This handball type gamt originated in the
Basque region of Spain. Jai alai is one of the fas test-moving ball
games. In Florida it is legal to place bets on the players, somewhat
similar to betting in horse racing. Bets are placed on a win, place,
show basis-that is, first, second, and third.
Sports experts agree that jai alai requires more skill, speed,
endurance, and nerve than any other ball game.

39. Where did jai alai originate?


40. Betting on jai alai players is compared to what other sport?
41. Which of the following is NOT true?
42. To what game is jai alai compared in the reading?

Questions 43 through 47 are based on the foll.owing lecture about


Alexander Graham Bell.

Alexander Graham Bell was born in Edinburgh, Scotland, in the


nineteenth century and later came to the United States. Several
members of his family did a great deal to encourage him in the field
of science. His father was most instrumental in supervisine his work
with th•! deaf. While h~ dealt with the deaf and investigated the
science of acoustics, his studies eventually led to the invention of the
508 ANSWERS AND EXPLANATIONS FOR PRACTICE TEST I

multiple telegraph and his greatest inventio n- the telepho ne. The
last q uarter of a century of his life was dedicated to advances in
aviat ion .

43. What was considered to be Alexander G raham Bell's grea test


achieveme nt?
44. To what did Bell dedicate the last years of his life'?
45. What can we co nclude about Alexande r G raham Bell?
46. Which of the following stacements is NOT true'?
47. How many years did Bell dedicace to aviation?

Questions 48 through 50 are based on the following explanation of life


insurance products to a customer.

Now chat you know you want to purchase life insurance, you must
choose fro m cwo types. The amount of money paid periodically for
an insurance policy is a premium. T he type of life insurance you
choose will affect the amount oi the premium you pay.
Te rm life insura nce is purchased for a given period of time, or
term. At the end of the term, the insurance expires. It insures your
life based on a fo rmula that considers how long you are expected to
live. This product provides the greatest coverage for the lease
amount of money. You do not pay any money as a n investment in
addition to the insurance cost. If yo u choose to purchase insurance
after the expiratio n, the premium will be higher because it is
calculated o n yo ur attained age, and a t that point, yo u will be older
than you are now. T he longer the premium is guara nteed to remain
consta nt, the gre ater the premium will be because it reflects the
average cost of insurance for all years being cove red.
Cash value life insurance, on the o ther hand, has a component
fund in which the life insurance company deposits part of the
premium and pays interest earned on its investments in mortgages,
bonds, stocks, and other investments. T he balance of the premium
purchases term i ns~rance, which is calculated in the sam e way as if
you purchase term life insurance. Consequently, the premium is
significantly higher than that of term life insurance. However, most
cash value life insurance products have a fixed premium schedule
a nd remain in effect througho ut your life. In many cases, the
SECTION 2: STRUCTURE AND WRITTEN EXPRESSION 509

interest earned on the investment portion of the premium will


ultimately pay the premiums so that at some point you can
discontinue making payments out of pocket.
Once the fund has started to accumulate, you may borrow some
of the funds at a low interest rate, receive retirement income, or
eve:, stop paying premiums. This type of policy also builds up a cash
value so that if you want to cancel the policy, some money is actually
returned, unlike the term policy, which has no value other than the
insurance.

48. Which of the following is NOT a reason to buy cash value


insurance?
49. Why is cash value life insurance more expensive than term life
in~mance?
50. Which of the following is a benefit of term life insurance?

EXPLANATIONS FOR PRACI1CE TEST 1

SECTION 2 : STRUCTURE AND WRITTEN EXPRESSION

Part A

1. (C) The word order should be: subject + verb + complement +


modifier of manner + modifier of time.

2. (D) Choice (A) is incorrect because another is singular and


pants is plural. (B) and (C) are incorrect because other cannot
be used in the plural form when it is functioning as an adjective.
(D) is correct; pair is preceded by the singular article (an +other
pair).

3. (D) Committee is singular, so the pronoun that follows it must


be it and the verb must be has. Choice (B) is verbose and uses a
plural reflexive pronoun, themselves, incorrectly. (C) is passive
and thus not parallel. It also contains an unnecessary preposi-
tion, at.
I
510 ANSWERS AND EXPLANATIONS FOR PRACTICE TEST I

4. (C) Choice (A) is incorrect because /onelynessly is not a word,


I
and the expression in times previous should read in previous tim es
(or better, p reviously). Choice (B ) creates a double negative, not
never, and sole means only. It does not mean alone. (D) is
I
verbose.
I
I
5. (C) Must + perfective indicates a logical conclusion. (He made
the highest score, so we assume that he studied.)

6. (B) Modal + [verb in simple form). (Will be teaching = future


progressive.) Choice (A) is incorrect because must should not
be followed by the infinitive. (C) is incorrect because because of
I
cannot be followed by a complete sentence. (D) is incorrect
because have cannot be followed by a [verb + ing]. I
7. (B) Choices (A), {C), and (D) all have dangling participles
(suggesting that the problem, it, o r the discussion may have been
served lunch). The subject of the participial phrase must be the
I
committee members.
I
8. (D ) Follow the negative agreement rule: neither + auxiliary +
subject. Choice (A) is incorrect because states is plura l and
hasn 't is singular. (B) is incorrect fo r the same reason, and it has
others before a noun. (C) is incorrect because also is redundant
when used with either. I
9. (D) This sentence requires the subjunctive form: requested that
+ [verb in simple form). In choice (A) the verb is in the past
tense (studied) rather than in the simple form (study), and the
I
modifier (m ore carefully) is incorrectly placed before the comple-
ment (the problem). (B) also contains the ve::b in past tense and
carefu/nessly, which is not a word.
I
10. (D) Use the affirmative agreement rule: so + auxiliary +
\
subject. Choices (A) and (B) do not have an auxiliary, and (C)
has an incorrect auxiliary (is instead of does).
I
I
5ECTION 2: STRUCI"URE AND WRITI'EN EXPRESSIOI' 511

11. (C) The sequence of tenses should be said . . . could l Past ...
past). In choice {A) can is present tense, and the usually car.not
precede a day of the week. Choice (B) is passive. The passive
construction is not necessary here and makes the sentence
verbose. Choice (0) c1Jntains incorrect wo~d order.

12. (B) Organization is singular and requires a singular verb, has.


Less is used with non-count nouns and fewer with count nouns.
Choice {A) contains a plural pronoun and verb, and volunteers
should be preceded by fewer. (C) uses incorrect word order. (D)
is incorrect because you should use than, not that, in a
comparative.

13. (C) This is an embedded question: q ue!:~!'.l!l word + subject +


verb. Choices (A), (B), and (D) do not follow this order. (D)
also has the expression the tim e when, which is redundant.

14. (C) Use much + non-count nouns. Choice ( A) uses many


instead of much. (B) and ( D ) are verbose and use the incorrectly
(sugar here is general, not spedfic).

15. (D) This is a tag question. Has is the auxiliary m the main
clause; therefore, has must be used in the tag. The main clause
is negative, so the tag should be affirmative. When there is used
as the subject of the main clause, it must also be the subject of
the tag.

PartB

NOTE: Gt = nothing, indicating that t his word or phrase should be


deleted.

16. (C) should be on. On + the name of a street.

17. (A) should be fewer. Members is a counc noun and must be


preceded by fewer.
512 ANSWERS AND EXPU\NATJONS FOR PRAcnCE TEST l

l8. (D) should be very. Really is slang and not appropriate in formal
written English.

19. (D) should be since. Use since + beginning time (the action
began in the 1960s and continues up to the present). During or
in would also be correct if the sentence were taken to mean that
the progress took place only in the 1960s.

20. (A) should be rarely. Rarely is negative and cannot be used with
another negativ~ . Not rarely is a double negative.

21. (D ) should be yet. Use yet in negative sentences. A/ready is used


only in positive sentences.

22. (B) should be 0 or some. News is a non-<:ount noun and a means


one.

23. (B) should be remembered. He gave the assignment first (past


perfect), and then he remembered (simple past) that Monday
was a holiday.

24. (D) should be cried out. The correct sequence of tenses requires
past tense. (Having+ [verb in past participle] means past time.)

25. (D) should be either. Correct negative agree.~nent is: subject +


auxiliary +not +either.

26. (A) should be was hit. Passive voice is necessary here. Be +


[verb in past principlej. (The ship hit the bridge.)

27. (B) should be the manager. Subject + verb + indirect object +


direct object. There should be no preposition.

28. (C) should be not to allow. This is a negative indirect command:


verb + (not) + infinitive.

29. (C) should be those. It is incorrect to say these ones or those ones
although it is possible to say this one or that one.
SE'.'.:TION 3: READING COMPREHENSION 513

30. (D) should be doesn 't either. Seldom is negative and must be
followed by negative agreement, not positive agreement.

31. (D) should be didn't it. Used to indicates a past habit and uses
did when an auxiliary is needed.

32. (A) should be a. Use this indefinite article before words


beginning with a consonant sound.

33. (D) should be much homewo.·!:. Homework can never be plural,


and it is non-count, so it must be preceded by much.

34. (A) si1nuld be are. Scissors is plural and must be used with a
plural verb.

35. (A) should be infonnation. This noun can never be plural. It is


non-count.

36. (A) should be intelligent enough. Adjective + enough.

37. (B) should be many. People is a plural count noun.

38. (B) sh~uld befrom. From a time to a time.

39. (A) should be on. Always use this preposition with the floor of a
building because a floor is a surface.

40. (A) should be a new sports car last week. The complement (a new
spons car) should precede the modifier (Last wee.':).

SECTION 3: READING COMPREHENSION

1. (B) The three periods are the Paleolithic, Mesolithic, and


Neolithic.

2. (A) 'rite sentence indicates that the name " Stone Age" was
" derived from," or " came from," the tools and weapons that
we.;e used.
514 ANSWERS AND EXPLANATIONS FOR PRACTICE TEST I

3. (B) The hatchet was developed between 2 million B.C. and 8000
s.c., during the first period.

4. (A) Farming was never mentioned.

5. (D) The reading implies that the items made were not sophisti-
cated, so the reader can infer that the word "crude" is related to
that concept.

6. (B) Sentence 2 says, "Its name was derived from the stone tools
and weapons that modem scientists found."

7. (8 ) The sentence indicates that the people began domesticating


animals and establishing permanent govern;.1cnts. indicating
that those actions were less ·'nomadic" than in previous times,
which should lead you to understand that " nomadic" means
"wandering."

8. (C) The subject described in the introductory sentence of the


passage is the Stone Age. The Neolithic (A) and Paleolithic (B)
ages are discussed as subdivisions of the Stone Age. The Ice
Age (D) is mentioned in just o ne sentence as being an
important influence on ancient societies.

9. (A) The passage describes the numerous developments, or


inventions, of the Mesolithic Age (pottery, fish hooks, hunting
dogs, and the bow and arrow). Hunting and fishing would not
keep people indoors all the time (B), and no mention was made
of people being warriors (C) or crude (D ).

HJ. (B) The sentence indicates that the age being discussed is
different from previous .. eras," so that should provide you with
a hint that "era" means "period" of time.

11. (D) The sentence indicates that the need to preserve energy has
become more evident, or " highlighted."
SECTION 3: READING COMPREHENSION 515

12. (D) Choices (A), (B), and (C) were drawba~ks (disadvantages)
of the " C1Jnventional method" of boning. Only choice (D ),
toughness of meat, was given as a drawback of hot boning.

13. (A) The sentence indicates that there is some "urgency" to


preserve energy.

14. (B) The first paragraph concerns the fact that hot boning is an
energy-saving technique, and the last paragraph says that
refrigeration space and costs are minimized by hot boning.

15. ( A) A "carcass" is :i "body," and "to chill" r.1eans " to cool."

16. (B) " Early excision" means " hot boning." Paragraph 3 says
" early excision, or hot boning," which indicates that they mean
the same thing.

17. (C) The last paragraph states, " . . . hot boning following
electrical stimulation has been used to reduce the necessary
time of rigor mortis."

18. (C) There is no contextual clue to assist you if you do not know
what " bovine" means, other than that it is an adjective
modifying " carcass."

19. (B) The sentence preceding this sentence is discussing the


advantage of hot bo ning following electrical stimulation, rather
than hot boning alone. So ·'this" refers to that concept. Notice
that ·'rigor mortis" (C) is not a " method" at all.

20. (B) The entire reading is about treatment of the body of the
dead animal, which is what a "carcass" is.

21. (D) The word "considerable" modifies " amount of refrigerated


space." What can modify an amount? It must mean a large
amount or a small amount. Answers (A) and (B) make no sense
in that context, and " lesser" is not followed by words indicating
it is a comparison.
516 ANSWERS AND EXPLANATIONS FOR PRACTlcE TEST I

22. (D) The reading states that hot boning actually results in
tougher meat, so answer (A) is not correct. The reading does
not say anything about what the bones can be used for, so (B) is
incorrect. (C) is incorrect because increasing chilling time
certainly would not be a benefit and would not be recom-
mend<:.d.

23. (B) This word is related to the verb "trim," which means to cut
or remove.

24. (B) The last sentence says that the United States currently
controls the canal.

25. (D) Locks are enclosures of th1; canal govcu11:;d by gates that
allow the interior water level to be raised or lowered so ships
can pass from one elevation of the e<>.nal to another.

26. (C) Sentence 2 says that it costs fifteen thousand dollars to


travel through the canal and ten times that amount ($150,000)
to go around Cape Horn.

27. (A) Sentence 1 suggests that 1920 was thirty-nine years after the
canal construction was begun (1920 - 39 = 1881).

28. (B) Because of lower costs and shorter traveling time, W"' can
assume that the project has been beneficial.

29. (D ) The fact that the sentence indicates the Atlantic and Pacific
Oceans are " linked" indicates that they have been "joined."

30. (B) "Initiated" means "commenced," or " la-u:!c.hed."

31. (A ) The passage states that more than fifteen thousand ships
pass through the canal each year, so (A) cannot be true.
Choices (B), (C). and (D) are all facts from the passage.

32. (A ) They did not place the olive wreaths on their "own" heads.
SECTION 3: READING COMPREHENSION 517

33. (A) The sentence ~tates that the games were open to all free
males with no criminal record, but that previously that was not
t!-.e case, which indicates that "elite" means "aristocracy."

34. (B) The first sentence says that " the first Olympic Games were
held .. . to honor the Greeks' chief god, Zeus."

35. (D) Add a s.c. date to an A.O. date to get the total length of
time. The question asks for the approximate number of years,
and the answer choices are in round numbers, so round off 776
B.C. to 800 s.c. and add it to the approximate modern date of
2000. 800 + 2000 = 2,800.

36. (B) This is an inference question. Ct;.,;~ (A) is incorrect


because the passage does not indicate the Greeks were paci-
fists; in fact, it states that they were sometimes involved in wars.
Choice (C) is incorrect because nothing in the reading passage
suggests that they were "simple." Because they "calculated
time in four-year cycles," it could not be inferred that they
couldn't count (choice D). Because the whole passage concerns
athletics, choice (B) is the logical answer.

37. (A) The passage specifically states that " the Greeks empha-
sized physical fitness and strength in their education of youth."
Choice (B) is not mentioned in the passage. Choice (C) is
incorrect because only the elite and military could participate at
first, and then only free Greek males. Pleasing the gods was the
goal of competing during religious festivities, not returning to
war (D).

38. (A) The sentence indicates that winners were honored for their
"deeds," so it can be inferred that "deeds" are " accomplish-
ments."

39. (C) The third sentence states that the "games were open to all
free Greek males who had no criminal record."

40. (C) The sentence indicites that the games were very important,
so it is implied that a \\ar would be stopped, or "curtailed."
518 ANSWERS AND EXPIANATIONS FOR PRACTICE lCST I

41. (B) The passage states that the winners oflocal athletic contests
competed every four years at Mount Olympus, so an "Olym-
piad" spans the time between the Olympic Games.

42. (C) Sentence 3 states that Ybor moved his business from south
Florida to west Florida.

43. (A) The sentence states that he was " forced to flee" as a result
of a revolution, so " flee" means "escape."

44. (B) The sentence indicates that the problems where he was
living caused him to "seek," " look for," or " pmsue" another
location.

45. (B) It can be inferred from the reading that Ybor will be
remembered because Ybor City was named in his honor.

46. {A ) The sentence states that people " flocked" because of the
demand for workers, which implies that many people traveled
to the location.

47. {A) One fifth of the city's twenty thousand residents means that
four thousand people worked at the cigar factory.

48. (C) The passage is about Ybor, so it would be logical to use his
name in the title. The passage is not about the Spanish-
American War (A), nor is it a technical article about cigar
making (0). Choice (B) is feasible: however, the passage
focuses on Ybor's contribution to the Tampa area and not the
broad contributions of many Cubans.

49. (A ) A "site" is a " location," and this is implied in the sentence


that states Ybor bought a tract of land on which he built his
factory.

50. (C) The passage states that Marti was a Cuban poet and
freedom fighter who organized a revolution. None of the other
choices is mentioned in the passage.
PRACJ'ICE TEST 2
ANSWER KEY FOR PRACTICE TEST 2

After some answers in this answer key, you will find numbers in
italic type. These are page numbers in Part III where you will find
review material for these questions. Although any one question may
involve several different rules and concepts, these page numbers
refer to important areas you should review if you have missed a
q uestio n or are not sure of the material involved. Make full use of
these page number references and of the index to direct your
personal review.

Section 1: Listening Comprehension


1. (D ) 11. (8 ) 21. (D ) 31. (C) 41. (C)
2. (C) 12. (C) 22. (8) 32. tD) 42. (8 )
3. (C) 13. (C) 23. (8 ) 33. (A) 43. (8 )
4. ( B) l4. (C) 24. (B) 34. (B) 44. (D )
5. (0 ) LS. (A) 25. (C) 35. (A) 45. (C)
6. (A) 16. (8 ) 26. (8) 36. (B ) 46. (C)
7. (A) 17. (8) 27. (C) 37. (C) 47. (C)
8. (D ) 18. (A) 28. (C) 38. (B) 48. (B)
9. (A) 19. (A ) 29. (0) 39. (C) 49. (0 )
10. (A) 20. (0 ) 30. (C) 40. (A) 50. (B)

519
520 SCORING PRACTICE TEST 2

Sect.ion 2: Structure and Written Expression


l. (C) 229 21. (A) 45
2. (D) 46. 148-149, 229 22. (C) 71
3. (B) 212-215 23. (C) 69- 70
4. (C) 80, 229 24. (0 ) 98
5. (B) 44-49 25. (C) 86
6. (B) 209-210 26. (D ) 135
7. (A ) 134-135, 186-187 27. (D) 150-151
8. (C) 221-223 28. (C)68~9
9. (A) 29. (B)
lO. (B) 67. 134-135 30. (B) 82
l l. (A ) 84-85. 134-135, 209 31. (A)
12. ( DJ 138- 139 32. (D) 147-148
13. ( B) 135 33. (B) 85-86
14. ( D) 212-215 34. (C) 269
15. (B) 135, 229 35. (B) 150- 151
16. (B) 89-90 36. (D) 91
17. (A ) 123-124 37. (D) 97
18. (A ) 188 38. (A) 205-206
19. (A) 84-85 39. (A) 69-70
20. (A) 112 40. (B)

Section 3: Reading Comprehension


1. (A ) 11. (C) 21. (B) 31. (B) 41. (D )
2. ( B) 12. (A) 22. (C) 32. (C) 42. (B)
3. (A) 13. (B) 23. (A) 33. (C) 43. (D )
4. (B) 14. (D) 24. ( D) 34. (A) 44. (D )
S. (C) 15. (D) 25. (A) 35. (B) 45. (A)
6. (A) 16. (B) 26. (B) 36. (B) 46. (C)
7. (C) 17. (A) 27. (C) 37. ( B) 47. (B)
8. (A) 18. (A) 28. (C) 38. (C) 48. (B)
9. (B) 19. (A) 29. (C) 39. (A) 49. (A)
lO. ( B) 20. (A) 30. (D) 40. (A) 50. (B )
SCORING PRACTICE TEST 2 521

PRACTICE TEST 2: ANALYSIS-SCORING SHEET


Use che chaI1 below to spot your strengths and weaknesses in
each test section and to arrive at your total converted score. Fill in
your number of correct answers for each section in the space
provided. Refer to the Converted Score Sheet on page 496 to find
your converted score for each section and enter those numbers on
the chart. Find the sum of your converted scores, multiply that sum
by 10, and divide by 3.
Example: If raw cores are then converted scores are
Section l : 33 51
Section 2: 26 49
~cct;on ~· '.lQ 53
urn d Converted Scores 153
Times 10 = 1,530
Divided by 3 = 510 =Total Converted
Score
This will give you the approximate score that you would obtain if
this were an actual TOEFL. Remember chat your score here may
possibly be higher than the score that you might receive on an actual
TOEFL simply because you are studying the elements of the test
shortly before- taking each test. The score is intended only to give
you a general idea of approximately what your actual score will be.

Total Total Converted


Possible Correct Score
Section 1:
Listening Comprehension 50

Section 2:
Structure and Written Expression 40

Section 3:
50
Reading Comprehension

TarALS 140

Sum of Converted Scores


Times 10 = - - -
Divided by 3 = = Total Converted Score
522 ANSWERS AND EXPLANATIONS FOR PRACTICE TEST 2

SECTION 1: LISTENING COMPREHENSION SCRIPT

Part A

1. Man: We have diet soda, but no regular cola.


Woman: I'll pass, thanks.
Third Voice: What is the woman probably going to do?

2. Man: How did your parents like the play they attended
last week?
Woman: My mother thought the language was terrible, but
my father liked it.
Third Voice: What does the woman say about her parents'
reaction to the play?

3. Woman: I need to have this prescription filled, please.


Man: AJI right, but you'll have a fifteen-minute wait.
Third Voice: Where did this conversation most probably take
place?

4. Woman: I thought we were going to have a science test this


afternoon.
Man: It was postponed because the teacher had to
attend a conference.
Third Voice: What does the man mean?

5. Woman: The president's State of the Union message last


night was q-.ite inspiring.
Man: I couldn't watch it because my political science
lecture let out too late.
Third Voice: What does the man mean?
SECTION I: LISTENING COMPREHENSION SCRIPT 523

6. Man: Do you think I have a chance of proving my case?


Woman: Definitely, and we' re going to s11e for punitive
damages as well.
Third Voice: What is the probable relationship between the
man and the woman?

7. Woman: I can't even think with all that racket.


Man: It won' t get any better. Why don't you work inside
the meeting room.
Third Voice: What does the man imply?

8. Man: Marie's not eating her supper tonight. What's the


matter with her?
Woman: She went to the orthodontist and had braces put
o n her teeth. She says it hurts too much to chew.
Third Voice: What does the woman say about Marie?

9. Man: Have you been to that new supermarket that just


o pened?
Woman: Yes. The prices are quite reasonable. They have a
great variety of meats and vegetables, but you
have to bag your own groceries.
Third Voice: What does the woman say she dislikes about the
store?

10. Man: l hope you'll be ready to return to work soon.


Woman: I'm not quite up to it yet.
Third Voice: What does the woman mean?

11. Man: I heard Marilyn's entered college.


Woman: Yes, she's taking courses in statistics, 1~conomics,
and accounting.
Third Voice: What career does Marilyn probably plan to fol-
low?
524 ANSWERS AND EXPLANATIONS FOR PRACTICE TEST 2

12. Woman: Would you like Swiss cheese or American?


Man: There's no better cheese than Swiss cheese.
Third Voice: What does the man say about Swiss cheese?

13. Woman: I hope the game's not canceled.


Man: It will be held rain or shine.
Third Voice: What does the rnan say about the game?

14. Woman: I hear Martha was criticized in class.


Man: Had she read the material, she would have been
prepared.
Third Voice: What does the man say about Martha?

15. Man: Did everyone return the evaluation fo rms?


Woman: Sixty people received them, but only half re-
turned them.
Third Voice: What does the woman say about the evaluation
forms?

16. Wo man: Charlie has the potential to be a professional


musician, but he is too lazy to practice.
Man: Wh?.t a shame!
Third Voice: What does the woman say about Charlie?

17. Man: How long are you going to be away?


Woman: I'm planning on spending the weekend at the
beach as long as the weather stays nice.
Third Voice: What is the woman probably going to do?

18. Woman: Are there any witnesses to the terrible accident?


Man: No one but the seven-year-old boy saw it.
Third Voice: What do the speakers say about the boy?
SECTION I: LISTENING COMPREHENSION SCRIPT 525

19. Woman: Louise writes in Spanish very well.


Man: She writes it as well as she speaks it.
Third Voice: What does the man say about Louise?

20. Woman: How are Peter and Lucy doing lately?


Man: They had a quarrel, but they soon made up.
Third Voice: What does the man say about Peter and Lucy?

21. Woman: Do you make connections with the Maple Ave-


nue line?
Man: Yes, ma'am. Pay your fare and I'll give you a free
transfer and call you befc ::-e we get to Maple
Avenue.
Third Voice: Where did this conversation most probably take
place?

22. Woman: · I need some help with this math calculation.


Man: I've already looked at it: It's over my head too.
Third Voice: What does the man mean?

23. Man: What does Mary do with her children while she
works?
Woman: She takes them to a nursery on her way to work.
Third Voice: What does the woman mean?

24. Woman: Do you know where Dan is moving?


Man: He will move to Florida if his job confirmation
comes through.
Third Voice: What does the man say about Dan?

25 . Man: Does your father fish even in this heat?


Woman: Sure. He likes nothing better than fishing on a
hot, summer day.
Third Voice: What does the woman say about her father?
526 ANSWERS AND EXPLANATIONS FOR PRACTICE TE'iT 2

26. Man: Have the actors finished with rehearsals?


Woman: They went over their lines one more time before
the production began.
Third Voice: What does the woman say about the actors?

27. Man: We are supposed to turn in our assignments


tomorrow, right?
Woman: No. :vts. D aly asked us to hand them in today.
Third Voice: What does the woman say about Ms. Daly?

28. Woman: What are we doing with the dog while we are on
vacation?
Man: Stacey will look after him.
Third Voice: What does the man say?

29. Man: A lot of people play golf here in the United


States.
Woman: Yes. Although the game originated in Scotland,
it's more popular here than anywhere else.
Third Voice: What do the speakers say about golf?

30. Woman: When was the last time you saw them?
Man: It's been thirty years.
Third Voice: What does the man mean?

Part B

Questions 31 through 34 are based on the f ollowing conversation.

Man: You don't look too happy. What seems to be the prob-
lem?
Woman: ['ve got to write a Jong composition for my English class,
and I just can't come up with any ideas; it's due tomorrow.
SECTION I: LISTENING COMPREHENSION SCRIPT 527

Man: That shouldn't be too difficult. Remember those pictures


you were showing me last week, the ones from your cruise
last winter?
Woman: Sure. I've got them someplace.
Man: Why don't you write about your impressions of the
pyramids in Egypt and the camel ride you took.
Woman: That sounds like a good idea. I can also tell about our visit
to North Africa, the Holy Land, and all of the historical,
biblical places we visited.
Man: Well, now that you' re feeling better about this, I think I'll
be on my way. I've got to finish my composition too.
Woman: Thanks for your help. Once I get organized, it won't be so
difficult.

31. What was the woman's problem?


32. What does the man suggest?
33. Which of the following places did the woman NOT visit?
34. Why does the man have to leave?

Questions 35 through 38 are based on the following conversation.

Woman: Alan, you've been so busy lately that we don't see you
anymore.
Man: I've been trying to finish this research project so that I can
present my findings at the annual conference in July.
Woman: But that's two months away. You've still got lots of time.
Man: Not really. You see, I've finished all the research, and I've
j ust about organized all my notes, but it will take me
almost two months to type them.
Woman: I can type up your paper in less than twc weeks.

35. What does the woman offer to do for the man?


36. Why does the man need to finish the paper?
37. According to the conversation, what month is it now?
38. What stage of completing the project has the man reached?
528 ANSWERS AND EXPl.ANATIONS FOR PRACTICE TEST 2

PartC

Questions 39 through 41 are based on the following talk in a history


class.

Nathaniel Bacon was a man determined to protect his property


against Indian raids. He encouraged other Piedmont farmers to do
likewise. After Governor William Berkeley of Virginia had refused
to help them, Bacon and his friends banded together and destroyed
a group of attackers in April of 1676. Governor Berkeley declared
them traitors, and they assembled a group of some five hundred
people and marched on Jamestown, the capital, to insist or> the
governor's assistance. Berkeley later ordered them an arrested.
Because of this, the farmers burned Jamestown and took control of
the government. The governor fled.
The fight, which was known as Bacon's Rebellion, lasted almost a
year. Bacon contracted malaria and died in October of 1676, leaving
the farmers at the mercy of Governor Berkeley. Twenty-three men
were hanged at his request. King Charles II was very much upset by
the governor's treatment of the farmers and forced him out of
office.

39. Which of the following is NOT true?


40. Approximately how long were Bacon and the farmers able to
fight off the governor?
41. How did Nathaniel Bacon die?

Questions 42 through 46 are based on the following talk about Mount


Rushmore.

Towering over the Black Hills of South Dakota at six thousand


feet above sea level can be seen the majestic and lifelike figures of
four of America's greatest presidents. Gutzon Borglum spent
fourteen years carving these gargantuan busts in Mount Rushmore
as a lasting tribute to American leadership. In 1927, Borglum began
this monumental task when he was sixty years old, a time when most
men are preparing for their retirement, and not fo r a lengthy
project. Upon Borglum's death, his son continued the project until
the funding ran out.
SECTION I: LISTENING COMPREHENSION $CRIPT 529

Of the four presidents, George Washington's bust is the most


prominent, looking as serious as we tend to think of him. Behind
him is Thomas Jefferson, who bears a friendlier vi~age. Teddy
Roosevelt is tucked off into the comer next to the last of the four,
Abraham Lincoln, whose bust is the lc:ast complete.
It is unbelievable that such a monumental masterpiece should sit
in a now quiet area, once the scene of deadly battles between the
Sioux Indians and the white man.

42. Why was work on Mount Rushmore finally discontinued?


43 . Which of the following presidents is NOT represented in this
magnificent sculpture?
44. How old was Gutzon Borglum when he died?
45. How can the figures of Mount Rushmore best be desc:-ibed?
46. Which of the following is NOT true?

Questions 47 through 50 are based on the following talk about crude oil

What you see below you is crude oil. Crude oil is the source of all
petroleum products: gasoline, fuel oil. jet fuel. asphalt, lubricants,
and chemicals. The modem world cannot exist without the energy
derived from petroleum products, for industrial operations 2S well
as transportation. These products are burned to produce energy or
used as lubricants to reduce friction.
Many years ago, crude oil could be found close to the surface, and
it would ooze from the ground without a recovery operation. That
supply was very limited. Today oil wells must be drilled to depths of
hundreds and even thousands of feet. Generally, crude oil must be
pumped to the surface.
Crude oils are classified into two types determined by the
percentage of sulfur they contain. "Sweet" crude oil has less than
one percent sulfur, while " sour" crude has greater than one percent
sulfur content. Crude oil is a mixture of many complex organic
compounds. Though the chemical elements involved are mostly
carbon and hydrogen, with traces of sulfur, nitrogen, and oxygen,
the complicated structures of these organic chemicals create a
multitude of different compounds.
530 ANSWERS AND EXPLANATIONS FOR PRACTICE TEST 2

47. Where did this talk most probably take place?


48. What is crude oil?
49. How are crude oils classified?
50. What does " sour" crude oil refer to?

EXPLANATIONS FOR PRACTICE TEST 2

SECT!ON 2: STRUCTURE AND WRITTEN EXPRESSION

Part A

1. (C) In choice (A), the verb being is incorrect because it suggests


that Captain Henry is now remote fro m the enemy. If this were
true, he wo uld not need to creep through the underbrush. (B) is
incorrect because the infinitive 10 encounter is split by the
particle not. (D) uses incorrect vocabulary. O ne can " involve
oneself in something," but one cannot ·'involve oneself in a
person or people."

2. (D ) Choice (A) incorrectly uses childs. Children is the correct


plural of child. (B) is incorrect because a plural noun is required
after one of the. Choice ( C) is ver bose.

3. (B) Choice (A) is in error because it contains a dangling


participle, suggesting that it entered school. (C) conta ins an
improper use of the past perfect progressive (had been entering).
(D ) incorrectly uses enter into; use enter + r.oun. (Exceptio n: It
is correct to enter into an agreem ent o r contract.)

4. (C) In choice (A) carelessly is misused. Carelessly is the opposite


of carefully and can be used only with people o r animals. In (B)
it's is an error. It's means it is. It is not the same as the possessive
pronoun its. (D) is incorrect because only a person can care
about something. COle indicates the presence of feelings, which
plants (marijuana) do not have.
SECTION 2: STRUcnJRE AND WRITTEN EXPRESSION 531

5. (B) Choice (A) is incorrect because evidence is a non-count


noun, so a singular article cannot be used with it. Also, you
should use a possessive form before a gerund (our wanting). (C)
is incorrect because it is an incomplete sentence. After the feet,
which is the subject of the sentence, a verb is necessary (the
verb has in this case i~ part of a relative clau se). Choice (D ) is
verbose.

6. (B) Choice (A) is a sentence fragment. It has no main verb. (C)


uses themselves incorrectly. It has no antecedent. Choice (D )
has no sensible meaning.

7. (A) Choice (B) contains an incorrect inclusive (too ). A /s(> would


be correcl here. C hoices (C) and ( D) s hould say tt:acht:s well. A
verb is modified by an adverb.

8. (C) Choice (A) uses improper word choice and is not parallel.
T he verb receive refers only to money. Another verb would be
necessary for different types ofshows (such as, broadcast different
types of shows). (B) uses improper word choice and order. (D)
is verbose, and differently from should be different from.

9. (A) Choice (B) is incorrect because the proper form is:


preposition + [verb + ing]. Choice (C) has improper word
choice. We do not speak of quantities of m oney. Choice (D) is
incorrect because amount ofprofit is redundant, and bigger is too
in[.;:-mal for written English. It would be correct if it said earn a
Larger profit.

10. (B) The subject of this sentence is singular (production). Choice


(A) incorrectly uses a plural verh (ha ve). Choice (C) is incorrect
because be is a linking verb and cannot be modified by the
adverb erratically. (D ) also uses a plural verb with a singular
subject. Also, going up and down is too informal for written
English.
532 ANSWERS AND EXPlANATIONS FOR PRACTICE TEST 2

11. (A) In choice (B), the pronoun they has no antecedent. Choice
(C) is incorrect because verbs are modified by adverbs. In this
case, the word should be well, not good. (D) is verbose, has no
antecedent for the pronoun they, and should read capable of
producing. Capable + of+ [verb + ing].

12. (D ) In an equal comparison use as . . . as.

13. (B) Improved is a verb and must be modified by an adverb.

14. (D ) Choices (A), (B), ancl (C) all contain dangling participles.
suggesting that the bicycle, it, or the stonn is attempting to reach
home.

15. (B) Choices (A) and (D) are verbose. Choice (C) uses fast~y,
which is not a word.

Part B

16. (B) should be their. Use a possessive adjective before a gerund.

17. (A) should be cooking. Be used to + [verb + ing].

18. (A) should be kno ws how. Know how + [verb in infinitive].

19. (A) should be writing. Stop + [verb + ing].

20. (A) should be retype. Modal + [verb in simple fo rm].

21. (A) should be many. Times is a plural count noun and thus
cannot be modified by much.

22. (C) should be his. Each is singular and must be followed by a


singular verb and pronoun.

23. (C) should be is. Mrs. Stevens is a singular subject and requires a
singular verb. The phrase beginning with along with has no
effect on the number of the verb.
SECTION 2: STRUCTURE AND WRITIEN EXPRESSION 5J3

24. (D) should be so are. The auxiliary in the main sentence is are.
The positive agreement must contain the same auxiliary.

25. (C) should be forward to spending. Look forward to + (verb + ing].

26. (D) should be greatly. Adverb+ adjective+ noun.

27. (D) should be winter. When a noun functions as an adjective. it


cannot be plural. (Winter is the adjective and coats is the noun. )

28. (C) should be does. The singular subject lunch requires the
singular verb does.

29. (B) should be our. Fui agreement of pronouns use us . . . our.

30. (B) should be himself Hisse/f is not a word.

31. (A) should be on. One advises someone on something.

32. (D) should be higher. Use the comparative, not the superlative,
when only two entities are mentioned.

33. (B) should be heating. Use a gerund [verb + ing] after a


preposition (for).

34. (C) should be among. Use among for three or more entities and
between for two entities.

35. (B) should be dollar. When a noun functions as an adjective, it


cannot be plural.

36. (D) should be repair. In need of+ noun.

37. (D) should be isn 't it. A liter is singular, so the tag must also be
singular.

38. (A) should be was. The correct sequence of tenses is thought


(past) . . . was (past).
534 ANSWERS AND EXPLANATIONS FOR PRAcnCE TEST 2

39. (A) should be is. T he subject is singular (supply) and must take a
singular verb (is).

40. (B) should be any. Either is used for o nly two items, any for
three or more.

SECTION 3: READING COMPREHENSION

1. (A) Three languages, hieroglyphics. demotic, and Greek, are


mentioned in paragraph 1, sentence 4. The only choice not
!T'e!'.tioned is (A), French.

2. (B) Paragraph 2, sentence 5. states t hat they wo rked "indepen-


dently of each other." " Independently" means t he opposite of
" together."

3. (A) In the passage, " deciphering" is done to ancient Egyptian


hieroglyphics. so choices (8) and (C) can be elim inated immedi-
ately. Later in the passage, mention is made of " unraveling" the
langu age mysteries. which should lead you to choose " decod-
ing" rather than " probing."

4. (B) Paragraph 2, sen tence 1, says that the word was " Ptolemy."

5. (C) The words " t hey were fo rced to remain t here for three
years" indicate that they were waiting to continue t heir cam-
paign. This is an inference question. Note that choice (A ) is
contradicted by paragraph 1, sentence 2, which tells us they lost
a naval battle. Choices (B) and (D ) are contradicted by the fact
that t he stone was d iscovered by accident during the construc-
tion of a fo rt.

6. (A) Paragraph 2, sentence l, statC3 that Champollion deci-


phered the first word.

7. (C) Sentence 3 states the stele was " known as the Rosetta
stone, in commemoration of the town near the fort."

-------..--- --- -
SECTION 3: READING COMPREHENSION 535

8. (A) The sentence states that he had an " ambition to control all
the area around the Mediterranean Sea," which should lead
you ro unders tand that "ambitio n " is a desire, o r " aspiration."

9. (B) Choice (A) is not inaccurate; however, one might expect a


passage so titled to concentrate on the details of Napoleon and
how he discovered the stone, which it doesn't. Choice (C)
covers a detail of the passage , as does choice ( D). Determining
what was written o n the stone was the chief subject of the
passage.

10. (B) Lines 14-18 state that "the direction in which the~, were
,_,._. . ., -. ·- P-·· . .·~--- .,..~ ........_ . ,. ~ ~,. . -......
The <" " a....... '"~~e. a~an<ied .
tollowing sentence continues the description.

11. (C) The entire reading, especially the last paragraph, indicates
that Sequoyah will be remembered because he created a new
alphabet. Although he will also be remembered because the
redwoods were named after him, that simply manifests the
resul t of his accomplishment.

12. (A) The first sentence states that he was the "son of a white
trader and an Indian squaw." This is a description of his
parents. If you are unfamiliar with the word, answers (A)
through (C) could describe a person, but (A) is the correct
answer. Choice (D) would not be a good guess since his other
parent has been identified as a trader. It would be unlikely that
"squaw" would also mean "trader."

13. (B) Paragraph 2, sentence 1, says, "His family and friends


thought him mad."

14. (D) Paragraph 3, sentence 1, says that he desired "to preserve


words and events for later generations." Those words and
events would be the history of his people.

15. (D) " Illiterate" means " not literate," or not able to read or
write.
536 ANSWERS AND EXPLANATIONS FOR PRACTICE TEST 2

16. (B) No mention of music was made in the passage (A ), nor was
any made of Sequoyah actually going mad and listening to
leaves (C). T he passage states that some thoughl written
mate rial was from the Great Spirit, but no mention was made o f
avenues of supernatural communication (0). Sequoyah was
fascinated with reading, writing, and the idea of recording and
storing information for future generations.

17. (A) The fact that he spent twelve years developing this written
alphabet. despite obstacles, demonstrates his de 1 ermination .

18. (A) The passage is about Sequoyah, so it would be logical fo r


the title to contain his name. This would make choice (A)
preferable to choice (B). Choice (C) is inaccurate, a nd choice
(D), although mentioned, is not what the passage is about.

19. (A) The reading implies that he was very interested in the
" leaf," so you should be able to guess that " fascinated" means
that. A synonym for " fascinated" in this context is ·' absorbed."

20. (A) Paragraph 2, sentence 1, says that he worked " indepen-


dently,'" which means that he had the help of no one. Choice
(A) is the only choice that is not true. Choices (C) and (D ) a re
specifically stated in the passage. Choice (B) should be inferred
from the facts given. such as Sequoyah's basing his alphabet on
sound patterns he heard, and his early obsexvation of the
" talking leaf."

21. (B) Sentence 2 indicates that the Aztecs believed that they must
offer human sacrifices to appease, or pacify, the gods.

22. (C ) Paragraph 2, sentences 2 and 3, tell us that the victims were


indoctrinated (brainwashed) and heavily sedated (drugged).

23. (A ) The last sentence says that " they accepted their fate
passively," or "submissively."
SECTION 3: READING COMPREHE NSION 537

24. (D) From the context you can gather that the gods needed to b~
pleased. This eliminates choices (B) and (C). " Glorified" (A)
appe.~rs to be a possibility, but the definition of "appease" is to
soothe and satisfy (D).

25. (A) Making sacrifices to the gods is the subject of the reading
passage. Choices (B) and (D) are details in the passage, and
choice (C) is too broad in scope.

26. (B) The priests offered thei r collected blood to the gods,
warriors killed in battle gained favor with the gods, and victims'
palpitating hearts were removed for the gods, so ciK>i;e (B) is
correct.

27. (C) The list of reasons human sacrifices were necessary is


contained in sentence 2. The need for sacrifices as a require-
mem 0f fame is not mentioned in the passage.

::?8. (C) The second paragraph explains that the priests made the
victinis believe that they would occupy a place of honor in the
afterlife if they would allow themselves to be sacrificed.

29. (C) Sentence 3 states, "The priests practiced forms of self-


mutilation, such as piercing their tongues . . . "

30. (D ) Aztecs were both " mighty" and " warlike." That should
provide a clue that " mighty" means "powerful. "

31. (B) Paragraph 1, sentence 1, says that crude oil is found "from a
few hundred feet beneath the surface to as deep as 25,000 feet. "
Thus (B), " always found a few hundred feet," is not true.

32. (C) The sentence in which " minute" appears indicates that
" minute" means a small amount because the trace elements are
" disregarded." This should give you the hint that " minute"
means " insignificant" in this sentence.
538 ANSWERS AND EXPLANATIONS FOR PRACTICE TEST 2

33. (C) Paragraph 3, last sentence, specifically states this as the


reason fo r the many thousands of compounds. Complicated
refining processes (B) are me ntio ned as producing other
products, but not tho usands of co mpo unds.

34. (A ) The sentence states that o ther products are produced,


"some of which are used to upgrade and increase the octane
rating. " This implies that to " upgrade" means to ·'improve."

35. (B) Paragraph 4, sentence 1, gives heating and co ndensing as


the methods of producing products.

36. (B) T he sentence implies that " opaque" is a physical appear-


ance. just like ·'thin," " thick." and " transparent. " Furthermore.
" thin" and " thick" are opposites, so you should assume that
" transparent" and " opaque., are o pposites as well. Answer
choice (A) is the opposite of " opaque." Choice (C) is incorrect
because ·'tight" is not a characteristic of " o paque .., "Crude"
(D ) is raw oil. not a description of its appearance.

37. (B) The last sentence of paragraph l says, " Either pressure at
the so urce or pumping forces crude oil to the surface."
Although choices (A), (C), and ( D), might conceivably prod uce
pressure, they are not as complete a~ (B) because they do not
include pumping and are not specifically mentioned in the
reading.

38. (C) Paragraph 4, sentence 2, lists examples of light o ils as


gasoline, kerosene, and distillate oil. L ubricating oil is me n-
tioned in the first sentence of the reading, but we are not told
whether it is classified as a light o r heavy oil.

39. (A) fhe third paragraph explains tha t the chemical composi-
tion of petroleum products is carbon a nd hydrogen.

40. (A ) A reduction in temperature (cooling) changes vapors to


liquids, thus creating gasoline. kerosene. and distillate oil.
Expanding (B) or d iluting (C) the vapors, or gases, would not
create one of the liquid oil products mentioned. To refute is to
prove false, so choice (D) is incorrect.

41. (D) The sentence states, "Trace elements are also found, but in
such minute quantities that they are disregarded." The subject
of the sentence is clearly the reference fo r the pronoun.

42. (B) Paragraph 1, sentence 1, says that an election year is one


that is evenly divisible by four. Of the choices given, only (B),
1930, is not evenly divisible by four, leaving a remainder of two.

-B. (D) The last sentence of paragraph 1 tells us that William H.


Harriso n serve d the shortest term.

44. (D) Paragraph 2 gives the names of the fo ur American


presidents assassinated. Choice (A) is not true because the
reading. gives information only about presidents since 1840.
Ronald Reagan also did not die in office . Choice (C) is not true
because, although Roosevelt was elected fo ur times, he died
during the fourth term.

45. (A ) The entire reading answers this question. The presidents


mentioned are Harrison, Lincoln, Garfield, McKinley, Kennedy,
Harding, and Roosevelt.

46. (C) " Inauguration" means " swearing-in ceremony," a ceremo-


nial induction into office.

47. (B) Paragraph 3 tells us that Roosevelt died of polio; he was not
assassinated.

_.8. (B) T his fo rm of the relative pronoun is the objective or


co mplement form, meaning it must replace the complement of
the sentence rather than the subject. Analyze the sentence.
"James A. Garfield ... was shot .. . by a man . . ." (He would not
give a job to the man. ) So the refe rence is to the man to whom
Garfield would not give a job, and that is the same man who
assassinated him.
540 ANSWERS AND EXPLANATIONS FOR PRACTICE TEST'.!

49. (A) The entire passage is about presidents who died or were
murdered in office. There are clues throughout the passage that
" assassinated" means ··murdered. "

50. ( B) The sentence states " He had contracted polio . . . and


eventually died .. ." The fact that polio is a disease should lead
you to presume that "contracted" means ·'caught" or ·'devel-
oped."

\
PRACTICE TEST 3
ANSWER KEY FOR PRACTICE TEST 3

After some answers in this answer key, you will find numbers in
italic type. These are page numbers in Part III where you will find
review material for these questions. Although any o ne question may
involve several differ ent rules and concepts, these page numbers
refe r to impo rtant a reas you sho uld review if you have missed a
question o r are not sure of the material involved. Make full use of
these page number references and of the index to direct your
personal review.

Section 1: Listening Comprehension


1. (D) 11. (C) 21. (B) 31. (D) 41. (B)
2. (A) 12. (C) 22. (C) 32. (B) 42. (B)
3. (D) 13. (A) 23. (B) 33. (B) 43. (D)
4. (A) 14. (D) 24. (B) 34. (B) 44. (B)
5. (B) 15. (B) 25. (C) 35. (B) 45. (B )
6. (B) 16. (C) 26. (C) 36. (B) 46. (B)
7. (C) 17. (B) 27. (A) 37. (A) 47. (B )
8. (C) 18. (C) 28. (D) 38. (D) 48. (B)
9. (C) 19. (A) 29. (B) 39. (C) 49. (D )
10. (C) 20. (B ) 30. (D) 40. (A) 50. (C)

541
542 SCORING PRACTICE TEST 3

Section 2: Structure a nd Written Expression


1. (A) 21. (A) 45
2. (D) 22. (B) 65--07, 205
3. (D ) 191-193 23. (D) 62
4. (D) 48-50 24. (B) 71
S. (B) 25. (D) 69-70
6. (C) 118 26. (B) 187
7. (B) 227-228 27. (C) 120-123
8. (D ) 45, 153-154 28. (B) 89-90
9. (B) 118 29. (B) 157
10. (D) 118 30. (C) 149
11. (C) 52-53 31. (A ) 45
12. ( B) 219- 220 32. (B) 150-151
13. (A) 33. (C) 70
14. (C) 62--03 34. (C) 269
15. (C) 88, 221-223 35. (A ) 45
16. (A) 121-122 36. (B) JOO
17. ( D ) 98-99 37. (C) 148
18. (A ) 157 38. (B)
19. (D ) 83-84 39. (B) 47
20. (A ) 83-84 40. (B) 146

Section 3: Reading Comprehension


1. (C) 11. (C) 21. (B) 31. (A) 41. (A)
2. (A) 12. (D ) 22. (A) 32. (C) 42. (A)
3. (A) 13. (C) 23. (B) 33. (B) 43. (B)
4. (B) 14. (A) 24. (A ) 34. (B) 44. (B)
5. (C) 15. (A) 25. (D ) 35. (B) 45. (D )
6. (B) 16. (B) 26. (D) 36. (D) 46. (C)
7. (A ) 17. (A ) 27. (B) 37. ( B) 47. (B)
8. (C) 18. (B) 28. (A) 38. (B) 48. (D )
9. (C) 19. (D ) 29. (C) 39. (A ) 49. (C)
10. (C) 20. (B) 30. (C) 40. (A) 50. (C)
SCORING PRACTICE TEST 3 543

PRACTICE TEST 3: ANALYSIS-SCORING SHEET


Use the chart below to spot your strengths and weaknesses in
each test section and to arrive at your total converted score. Fill in
your number of correct answers for each section in the space
provided. Refer to the Converted Score Sheet on page 496 to find
your converted score for each section and enter those numbers on
the chart. Find the sum of yo ur converted scores, multiply that sum
by 10, and divide by 3.
Example: lf raw scores are then converted scores are
Section 1: 33 51
Section 2: 26 49
Section 3: 38 53
Sum of Converted Scores 153
Times 10 = 1.530
Divided by 3 = 510 = Total Converted
Score
This will give you the' approximate score that you would obtain if
this were an actual TOEFL. Remember that your score here may
possibly be higher than the score that you might receive on an actual
TOEFL simply because you are studying the elements of the test
shortly before taking each test. The score is intended only to give
you a general idea of approximately what your actual score will be.

Total Total Converted


Possible Correct Score
Section 1:
Listening Comprehension 50

Section 2:
Structure and Written Expression 40
Section 3:
50
Reading Comprehension

TOTALS 140

Sum of Converted Scores


Times 10 = - - -
Divided by 3 = _ _ _ = Total Convened Score
544 ANSWERS AND EXPlANATlONS FOR PRACTICE TEST 3

SECTION 1: LISTENING COMPREHENSION SCRIPT

Part A

l. Man: I thought Mike and Francie we re getting married


in J une.
Woman: No, that's when his cousin's wedding is. They're
getting married the following month.
Third Voice: When does the woman say Mike and Francie are
getting married?

2. Man: Did you hear that the Chapmans sold thei r house
and are mo·, :ng to Arizona?
Woman: Yes. and the man who bought the house is a
doctor from Philadelphia.
Third Voice: According to the speakers, from where will the
new owner come?

3. Wo man: My computer won't boot up this morning.


Man: Good thing you backed up your data last night.
Third Voice: What does the man mean?

4. Woman: This doesn't look at all familiar. We must be lost.


We'd better get some directions.
Man: Let's pull in he re. While I'm filling the tank. you
can ask fo r directions.
Third Voice: Where will the man and woman go fo r assis-
tance?

5. Man: May 1 speak to Jason Daniels, please?


Woman: Nobody by that name works here.
Third Voice: What does the woman mean?
SECTIOl'I o: LISTENING COMPREHENSION SCRIPT 545

6. Man: Kelly, yo u look tired.


Wo m an : I am . I've been working on the budget report for
the finance committee fo r three days and nights.
Third Voice: Why does the woman say she's tired?

7. Woman: John, how are you? I heard you were sick.


Man : They must have confused me with somebody else.
rve never felt better.
Third Voice: How does the man say he feels?

8. Man: I'd likt! to buy this table. but I'm $20 short.
Woman: I'll lend you the money if you can pay me back by
Friday.
Third Voice: Can the man buy the table?

9. Man: Ha!> Dave returned fro m Europe yet?


Woman: Yes, but he was here fo r only thrct: days before
his company sent him to Canada.
Thi rd Voice: Whe re does the woman say Dave is now?

10. Man: Have yo u seen Ann Marie in the past fiftee n


minutes?
Woman: She went to the gas station to have her tank filled.
Third Voice: What does the woman say about Ann Marie?

11. Woman: George and Jetf were not at the meeting.


Man: T hey would have come if they had known about
it.
Third Voice: What does the man say about George and Jeff?

12. Woman: Are the test results posted yet?


Man: Yes. Most of the students scored eighty percent
and above, but Michael is the exception.
Third Voice: What does the man imply about Michael?
546 ANSWERS AND EXPLANATIONS FOR PRACTICE TEST 3

13. Man: Have they fo und o ut who took the answer sheets?
Woman: The humanities professor asked the dean to
question several students.
Third Voice: What does the woman mean?

14. Man: I' m afraid I'll have to work lace again tomorrow
night.
Woman : Why don ' t you ask for some extra help ?
Third Voice: What does the woman suggest the man do?

15. Man : If Henry hadn' t had so much work to do, he


would have come to the concert with us.
Woman: It's too bad he missed such a great show.
Third Voice: What do the speake rs say about Henry?

16. Man: Did James return the books to the library?


Woman: No, he had them renewed.
Third Voice: What does the woman say about James?

17. Man: Has Harry stopped smoking yet?


Woman: He is afraid he'll gain weight.
Third Voice: What do the speakers say about Harry?

18. Man: Does Jonathan spend much time studying?


Woman: He hasn' t studied in weeks. but he'l! do well.
Third Voice: What does the woman imply about Jonathan'!

19. Woman: How was your meal at the banquet?


Man: My meat was so tough 1 could hardly cut it.
Third Voice: What does the man say about his experience at
the banquet?

20. Man: Where could Rick be?


Woman: He must have forgotten about our meeting.
Third Voice: What does the woman say about Rick?
SECTION I: LIS1ENINO COMPREHENSION SCRIPT 547

21. Man: What's in that bag over there?


W o m an: I bought some apples, peaches, p e ars, a n d grapes.
Third Voice: What is the woman talking abcut?

22. Woman: Where did Joe and Nancy go fo r their honey-


moon?
Man: They were going to go to Puerto Rico, but they
couldn't afford it, so they went to St. Augustine
instead for one week.
Third Voice: What does the man say about the couple's honey-
ruoon?

23. Woman: Did you know that the hot dog did not originate
in the United States but in Germany?
Man: Yes, and they've even had something similar to it
in Finland. It's made out of reindeer meat.
Third Voice: Which of the followi ng is NOT true about the hot
dog?

24. Woman: You ought to take it easy fo r a few days.


Man: I have no time to spare.
Third Voice: What problem does the man have?

25. Man: Tiffany is already walking, but Stephanie isn't.


Woman: Tiffany was born before Stephanie was.
Third Voice: What does the woman say about Tiffany and
Stephanie?

26. Man: I am taking my car downtown to be repaired.


Woman: Be sure you get an estimate.
Third Voice: What does the woman advise the man?
548 ANSWERS ANO EXPLANATIONS FOR PRACTICE TEST 3

27. Woman: Why did Professor Nelson get angry with Jane?
Man: She should have worked on her paper last night,
but s he watched TV instead.
Third Voice: What does the man say about Jane?

28. Man: Franklin focused o n the deer and snapped the


shutter.
Woman: What a great shot!
Third Voice: What are the speakers talking about?

29. Woman: I need to complete my paper this weekend.


Man: If I were you, I'd have it typed by a service.
Third Voice: What does the man suggest the woman do?

30. Man: I hear Yolanda ran into Anna downtown last


week.
Woman: I haven't seen either of them for months.
Third Voice: What does the man mean?

PartB

Questions 31rhrough 34 are based on the following conversation.

Man: Did you see that TV program last night about the sky
diver whose parachutes didn't open after he had jumped
from his plane?
Woman: No, I didn't. Did he die?
Man: No. It's really unbelievable how he could have survived
such a free fall, much less live to tell about it 011 television!
Woman: What happened?
Man: Neither of his chutes opened as he plummeted to the
ground. When they found him, they thought he was dead.
Doctors said he'd never walk agai n, but he proved them
wrong.
SECTION t : LISTENING COMPREHENSION SCRIPT 549

Woman: How long was he recuperating?


Man: He spent eighteen months in the hospital while his broken
bones were mending. He was no sooner discharged than
he jumped out of a plane again.
Woman: Gee, some people sure do crazy things!

31. Why was the man in the hospital?


32. Where did the interview take place?
33. What caused the man's accident?
34. What did the man do soon after he was rdeased from ,he
hospital?

Questions 35 through 38 are based on the followm~ ' onversation.

Man: What's the matter?


Woman: I can't sleep lying down. I feel a lot ot pressure in my
chest
Man: Well, there is so me congestion. I want to do some te:-.ts.
Woman: How soon will I get the results?
Man: Oh, you'll have the results before you leave the office, and
I'll prescribe some antibiotics that I believe will help you.

35. What is the probable relationship between these two speakers?


36. When will the wom an receive the results of the tests?
37. What does the man think will help the woman?
38. What is the woman's problem?

Part C

Questions 39 through 42 are based on the following news story.

Two men and a thirteen-year-old boy are safe now after being
rescued from their tiny boat which had been adrift in the G ulf of
Mexico fo r twenty-four hours. After their families had reported
them missing, the Coast G uard began searching, but the group was
550 ANSWERS AND EXPLANATIONS FOR PRACTICE TEST 3

rescued after waving frantically at a private airpla ne flyi ng over-


head . It turned o ut that they had drifted only seven and a half miles
from where their engine had broke n down.

39. How many people were in the. boat?


40. How were the boaters finally rescued?
41. Why did the authorities begin to search fo r the boat?
42. How far had the boat drifted?

Questions ./-1 through -!-5 are based on lhe following commentary by a


doctor to a patient.

Mr. Davis, I've just finished reading your x-rays. arid I would like
to discuss them with you. You have os te~Jart hritis in the middle of
your back and scoliosis. which is a curvature oi the spine. I can also
feel the muscle spasms and pinched nerves from yo ur shoulders to
the base of your spine. While th is may sound terrible to you. it is not
life-threatening, nor is 11 something to worry abo ut. At present.
there 1s no cure fo r these problems. bUL you can control them with
proper treatment. Fi rst of all. we need to adjust your diet a little.
The nurse will provide you with information o n some foods to avoid
completely and o thers whose consumption should be restricted.
Herc is an explanation of some back exercises that you can do. They
will help to stretch and strengthen the muscles and to relieve the
pain . Use a hearing pad and an ice pack to alleviate soreness in the
joints. I'm going to prescribe some muscle relaxers and pain killers.
Take them as indicated. In about SIX weeks we'll see how you '.I re
progressing and then begin some chiropracti<· treatment. J'd like to
see you again \n three weeks. Please av~ the nurse make an
appointment for theP.

43. What is the purpose of this talk?


44. What did the speaker do before talking with Mr. Davis?
45. According to the speaker. -.vhar is sco1iosis?
SECTION I: LISTEN ING COMPREHENSION SCRIPT 551

Questions .J6 through 50 are based on the following talk about Salvador
Dali.

Ladies and gent lemen. please move in a little closer as we begin


o ur tour th is afternoon. Today you will enjoy the la rgest collection
of Salvador Dali's works under o ne roof. They include several
hundred oil paintings, d rawings, and watercolo rs. more than a
thousand graphics, and a variety of sculptures as well as jewelry. As
vou will see . Da li was multitalented. He designed furniture. created
~xq uisi te wor ks with tine jewels, and concocted perfumes with
tantal izing arom:!:;. He JevelopeJ his talents over a span of six
decades. leaving an indel ible imprint on the world of art.
Herc we see some o f his early paintings. mostly la ndscapes of the
town of his hirth-Figueras in Cata lo nia. Spain-and the seascapes
of <.1 neighboring town called Cadaques. While these represe nt a
more tra d itional art fo rm, it was at this time tha t Dal i·s controver-
sial career began. He was erpelled fro m a prestigious art school in
Madrid beca use he Jisagreed wi th his p rofessors on the ir tech-
niq ues. He o nce threw hi mself down several flights of steps just to
get attention. At the age of twenty-one, he had his first o ne-man
show.
Four years later in Paris. he fell madly in love with Gala Eluard,
the wife of a French poet. She bec:ime his lover. and later they
married. She was the inspiration and model fo r many of his works.
The Disco very of.r!.merica by Christopher Columbus, his mo numental
masterpiece, shows Gala appearing on Columbus's banner. She also
served as his model for o ther works, such as the Crucifixion,
Ecumenical Council. and Hallucinogenic Toreador.
Dali's the mes varied fro m one period to the next, but many
contained recurring images of am s, crutches, limp watches, grasshop-
pers, and sexual symbols. All of these were, in some way, a carryover
from his childhood a nd adolescence. He often placed familiar and
outrageous imaginary objects side by side. A num ber of paintings,
such as Slave Market; Old Age, A dolescence. !11/ancy; Hallucinogenic
Toreador; and Lincoln in Dalivision portray dou ble i01ages. Depend-
ing on how you look at these works, you can see two entirely
d ifferent views.
Surrea iistic paintings are what Dali is best known fo r, and in most
of them he left everything tO the viewer's inte:-pretation. As you
552 ·\NSWERS ANO EXPLANATIONS FOR PRACTICE TEST 3

wander around on your visit today, look at the paintings up close


a nd then move back about twen ty feet and ponder them again from
that distance.
Before leaving, stop at our gift shop to browse and perhaps co
purchase some of the Dali memorabilia-poste rs, books, clothing,
perfume. and post cards. On your next trip to St. Petersburg, come
back to visit us. We are open Tuesday through Saturday fro m 10
A.M. to 5 r. M .• on Sunday from noon to 5 P.M., and we are closed on
Mondays and holidays.

46. Who is the spea~~er ?


47. Which of the following was not mentioned as a Dali creation?
48. Who was one of Dali's freque nt models?
49. Wha t was the artist 's most renowned art fo rm ?
50. What ki nd of paintings were the artist's earliest?

EXPLANATIONS FOR PRACTICE TEST 3

SECTION 2 : STRUCTURE AND WRITIEN EXPRESSION

Part A

l. (A) C hoice (B) includes improper word choice and order. (C) is
incorrect because it is not possible to say "win him the case."
Correct fo rm is "win the case for him. " (D) is incorrect because
minimum is a noun and cannot modify another noun (chance);
it is not clear to whom him refers, and the orde r is not correct.

2. (D ) Choices (A) and (B) are in error because it is not correct to


say " an attribute is when " or "an attribute is how"; an attribute
is a static quality. Choice (C) would be correct if it were "the
way he gives lectures."

3. (D ) Choice (A) uses incorrect vocabulary choice. Remembrance


has a sentimenta l meaning; it should be reminder. Also, the verb
rise, not raise should be used. (B) should read the following
SECTION 2: STRUCTURE AND WRllTEN E XPRESSION 553

month, not the month following. (T he adjective precedes the


noun.) Also, rise, not raise, is required. (C) is incorrect because
the verb contained must be followed by a noun, not a verb.

4. (D) Choice (A) incorrectly uses the before a singula r country


name. (B) uses improper word order and also uses the incor-
rectly. (C) is in error because French when used as an adjective
cannot be made possessive.

5. (B) T he expression should read, " It was not until . .. that." To


use when (choice A) is redundant.

6. (C) In choi:~ (A) there is improper use of the past conditional.


(B) includes improper vocabulary choice. A flat tire does not
happen. (D) makes improper use of the reflexive itself A tire ,
being inanimate, could not flatten itself.

7. (B) T he correct structure is adverbial (never) + auxiliary (have )


+ subject (so many women) + verb (received ).
8. (D) Choice (A ) is incorrect because homework is a non-courit
noun and few cannot be used with non-count nouns. (B) is
incorrect because a complete sentence is required after because.
Choice (C) is verbose. Also, because of cannot be followed by a
complete sentence.

9. (B) Know how in this sentence means " to have a practical


understanding of something." It is not correct to use to after
know how unless it is followed by a verb.

10. (D) Choices (A ), (B), and (C) are all incorrect past conditions.

11. (C) Choice (A) includes improper word choice. One other
should be another. Choice (B) uses also. Also does not mean the
same as again, which is the meaning conveyed by the sentence.
(D) uses incorrect word order. It should say fifty dollars more.
554 ANSWERS ANO EXPLANATIONS FOR PRACTICE TEST 3

12. (B) Choices (A) and ( D) are incorrect because the adjective
a ware must be followed by of before a noun o r noun phrase.
Choice (C) uses of, but the place where is redundant.

13. (A ) Choices (B), (C), and (D) are all too info rmal for written
English and are verbose.

14. ( C) Still. yet. and ulready are misused in the other .mswer
choices.

15. (C) Choices (A) and (D) o mit the word how, which must follow
know before a verb. 0 .11,; cho ice (C) uses parallel construction
(ho w tO manage . . :.ell . .. satisfy ).

Part B

NOTE: 0 = noth ing, indicating that this word or phrase shouid be


deleted.

16. (A) sho uld be liadn 't sent. A past wish must be followed by the
past perfect.

17. (D ) sho uld be so am I. For atfinnacive agreement use ~o +


auxiliary + subject.

18. (A) shouid be such a beautiful. Ca use and erfect: such + a .,...
adjective + singular counc noun.

19. (D ) should be to use. Allow + indirect object + infinitive.

20. (A) should be to stimulace. Use try + infinitive.

21. (A) should be is. Mumps is a non-count noun.

22. (B) should be had gone. T he past perfect is necessary to show


that this action (going to the supermarket) occurred before the
o ther actio n (coming home).

---- ---
\
'iECTION 2: STRUcnJRE AND WRITTEN EXPRESSION
SSS

23 ID) ~hould be for. Use for + duration of time.

24. ( 8) "hould be reports. Each + singular verb.


25. ( 0) "Should be is. Athlete is the subject and is singular.

26. : B) , hould be l.l. Both and as well as are redundant if they are
~sed together; use either both . .. and or as well as alone.

27. (Cl <>hould be were. This is a present wish. The verb be must be
m , he plural past te.11se form in a present wish because it is
~ontrary to fact.

28. <.B) should be his. Possessive forms must be used before a


gerund.

29. 1 B) ">hould be such a way. Cause and effect: such + a + singular


count noun + that.

30. •C) ~hould be dancers. After one of the there must be a plural
noun.

31. (A > :-.hould be fewer. Fri.ends is a plural count noun, so less is


incorrect.

32. (B) ·hould he four-day. Four-day here functions as an adjective


modifying the noun journey, so it cannot be plural.

33. (Cj .;hould be has. The subject, influence, is singular and thus
requires a singular verb, has.

34. i C) 'lhould be among. Use between whe n there are only two
r~n tit ies, among when there are more than two.

35 . ( A ) hould be Much. Population is a non-count noun, so many


cannot modify it.

36. (B) should be apparently. Verbs are always modified by adverbs,


no! >tdjectives.
556 ANSWERS AND EXPLANATIONS FOR PRACTICE TEST 3

37. (C) should be in. After one of the + superlative + noun + . .. ,


use in + singular count noun.

38. (B) should be to have missed. This is a perfect infinitive.

39. (B) should be an. U e an before words beginning with vowel


sounds.

40. ( B) should be worse. This is a double comparative: the harder . ..


the worse. Worst is superlative.

SECTION 3: READING COMPREHENSION

1. (C) Paragraph 2, sentence 2, says that ·'a serious eye infection


forced her to abandon the idea." Choice (A) is contradicted by
the information given. She did get admitted to medical school
because the first paragraph says "she was finally accepted."
Choices ( B) and ( 0) are true statemen ts. but they have nothing
to do with her not becoming a surgeon.

2. (A) Paragraph I says that it was ·'nearly impossible" for a


woman 2t this time to become a doctor. This answer ca n also be
inferred from the fact that she was the first female physician in
the United States.

3. (A) Paragraph 2 tells us that she graduated in L849 and


paragraph 3 that the hospital was opened in 1857. 1857 - l849
= 8.
4. (B) The question asks fo r the one choice that was not a first in
E lizabeth Blackwell's life. The passage states that she did noc
become a surgeon because of a n eye infection.

5. (C) Paragraph 2 tells us that she graduated in l849, and


paragraph 1 that she was born in 1821. 1849 - 1821 = 28.

6. (B) The sentence states that s he had to " abandon" her dream
of being a surgeon because of a serious eye infection. That
indicates that "abandon" means " give up."
SECTION 3: READING COMPREHENSION 557

7. (A) Answer choice (B) is incorrect because she did not abandon
her plans to become a " doctor," o nly her plans to become a
"surgeon ." Ans wer c hoices (C) a n d ( D) a re details but not t he
main idea of t he entire passage.

8. (C) The passage states that she " managed to o pen a ne':"
hospital," and then says "Besides being the first fe male physi-
cian and founding her own hospital ... " This means that opening
and founding are the same thing. The a nswer is further hinted at
later in the last se ntence, where it says " she also established,"
which indicates that founding and establishing are synonyms.

9. (C) Answer choice (A) prevented her from becoming a sur-


geon, not a doctor. She may not have had a lot of money (she
taught school and gave music lessons fo r money), but she
evidently did pay her t uition (B). Her sister was also a doctor, so
there is no reason to think she was not supported by her family
(D ) . However. it requ ired many letters to medical schools
be fo re she fo und o ne that would accep t women, so (C) is the
best answer choice.

10. (C) Line 11 specifica lly states that a serious eye infection forced
Blackwell to abandon the idea of becoming a surgeon.

11. (C) We are told in paragraph 1 t hat endocrine gland s have no


ducts and release t heir products directly into the bloodstream .

12. (D ) The se ntence states "The thyroid gland collects . .. , which


it stores .. ." T he pronoun precedes a verb (stores) and thus is a
subject, so "it" relates back to the "thyroid gland" in the subject
position of the sentence.

13. (C) Paragraph 3, sentences 2 and 3, says that cretinism occurs in


children as a result of hypothyroidism, o r underactive thyroid
gland.

14. (A) Paragraph 3, se ntence 3, tells us that myxedema occurs in


adults and ca uses the growth process to slow down. We can
infer that this would result in sluggishness, or lethargy.
558 ANSWERS AND EXPLANATIONS FOR PRACTICE TEST 3

15. (A) " Former" means t he first of two, while " latter" refers to the
second. The two items mentioned in the previous sentence are
"underactive thyroid" and "overactive thyroid." The sentence
also sta tes chat "underactive thyroid" is the same as "hypothy-
roidism." So " former" refers to an " underactive thyroid," also
known as "hypothyroidism."

16. (B) We are told in paragraph 4, sentence 1, that a goiter is an


enlarged, or swollen, thyroid gland.

17. (A) The first paragraph states that "exocrine glands secrete
their products through ducts, but endocrine glands, or ductless
glands, release their products directly into the bloodstrea m."
Thus the answer is (A).

18. (B) The sentence states that glands of this type "secrete" and
the other " release." "Emit" is a synonym for " release" or
"secrete."

19. (D) Paragraph 3 states that an "underactive thyroid causes


hypothyroidism."

20. (B) The passage does not address a ll glands, only some specific
types of glands, so choice (A) is incorrect. The passage does not
involve secretion in general, so (C) is incorrect. T he passage
deals with both an underactive and an overactive thyroid gland,
not just an overactive one. so choice (D) is wrong.

21. (B) The "function" of a thing is its purpose or the job it is to


perform. The function of the thyroid gland-to collect iodine
and produce and store thyroxine-is described only in lines 7-9.

22. (A) Paragraph 2 says that "animals can perceive environmental


changes . . ." The fact tha t observing animal behavior can
predict earthquakes does not indicate tha t the number of
deaths nor the location of the epicenter can be predicted, so
answer choices (B) and (C) are incorrect. The entire reading
regards learning of an impending earthquake in advance and
not after it has occurred, so answer choice (D) is incorrect .

-------~-
SECTION 3: READING COMPREHENSION 559

23. (B) Reasons for the animals' perceptions are not specifically
given in the reading, but we can assume that animals are able to
predict these occurrences because they have some instincts that
humans do not possess. No other choice is reasonable or is
suggested in any way by the reading.

24. (A) Even if you are not familiar with the word "evacuate," you
wou ld know that to keep the death toll down, people would
have to be moved away, or "removed," from the area.

25. (0) The reading gives examples of other animals, such as


llamas, seabirds, and chickens.

26. (0 ) The root word "center" in " epicenter" is the clue that
"core" is the word nearest in meaning.

27. (B) It can be inferred that if scientists can predict earthquakes,


they will have e nough warning to lead people to safety, th us
lowering the dea th rate.

28. (A) The phrase " although hundreds of thousands of people


were killed" implies that the event was destructive.

29. (C) This is an inference question. The passage is about


earthquakes and animals, so choice (B) is too broad for the
main idea. Earthquakes can' t be prevented by observing animal
behavior, so choice (A) is incorrect, and choice (0) was not
mentioned in the passage.

30. (C) In the thi rd paragraph, the author states that "Unusual
animal behavior preceding earthquakes has been noted for
centuries." Later in the same paragraph, the author states that
such behavior was observed " throughout the eighteenth and
nineteenth centuries."

31. (A) Paragraph 2, · sentence 1, states that the boy was found
"wandering in the woods." While it is true that the boy growled
at people, choice (C), we are not told that he growled at the
doctor when he was fo und.
560 ANSWERS AND EXPLANATIONS FOR PRACTICE lCST 3

32. (C) The word "offspring" means " children." " Litter" is used to
indicate the offspring of multiparous animals (animals that give
birth to a number of offspring each pregnancy).

33. (B) Paragraph 2, sentence 4, says that "the doctor won the boy' s
confidence and began to work with him. " You should infer that
the a bility to work with him was the result of the boy's
confidence (trust).

34. (B) " Utter" does mean "absolute." but not in this context, so
choice (A) is incorrect. The word is fo und in a list of the
doctor's accomplisments with the b O) , and "screaming a number
of words" most likely would not be considered a positive
development, so choice (C) is incorrer.t. Since reading may be
done silently, and " utte r" means to pronounce words using the
voice, choice (B) is betterthan (D ).

35. (B) Sentence 1 indicates that wolves have been said to care fo r
human children as fa r back as 700 a.c . Choice (C) is true.
" Domesticating" means 10 ·'tame" or " make fit for living in
human society." The doctor was successful in getting the boy to
clothe and feed himself and speak and write to some degree.
Choice (D) is true because " utter a number of words" does not
indicate that he could speak " perfectly."

36. (D) In this sentence, the word " preposterous" is being con-
trasted with the word " credible." Since " credible" means
" believable," you can determine that " preposterous" means the
opposite, " absurd" (totally u nbelievable).

37. (B) Answer choice (A) is incorrect because the passage 1s about
some specific children (Romulus, Remus, and another boy),
and not children in general. No ge neral statement is made
about children raised by wolves. Choice (C) is only one part of
the reading. Romulus and Remus are discussed in paragraph 1,
but another child is discussed in paragraph 2. Choice (D ) is
incorrect because it confuses the attempted drowning of Romu-
lus and Remus with the French doctor, who is mentioned in
paragraph 2.
SECTION 3: READING COMPREHENSION 561

38. (B) Rome did not exist when Romulus and Remus were
children because they were the founders (A). A French doctor
encountered a boy wandering in the woods, but that happened
2.500 yea rs after the lives of Romulus and Remus (C). The
twins were ordered to be drowned, but they were not (D).
Legend has it th:H Romulus and Remus founded the city of
Rome.

39. (A) The reading states that Romulus and Remus were the
" legendary twin founders of Rome" o n line 3.

40. (A) The last sentence of paragraph l states that V parahaemo-


lyticus is found in highest abundance in inshore waters, particu-
larly near harbors (a harbor is similar to a bay).

41. (A) ·'Inshore·· means ·'close to shore" or ··near the coast. "

42. (A) The e ntire reading passage concerns Vibrio parahaemolyti-


cas. The word ·'it" in paragraph l, sentence 2. rerers to that
organism, and so does the word "it" in line 13. T he preceding
sentence refers to the organism by name and then calls it an
"organism." When it refers to it as an "organism." it discusses
" isolatio n. " T hus, the only thing ··it" can be referring to is the
" o rganism.·•

43. (B) The last sentence of paragraph 2 states that a Japanese


scientist has not isolated V. parahaemolyticus as frequently in
winter as during warmer months. November is the coldest
month listed for the north Pacific.

44. (B) The first sentence of paragraph 3 gives diarrhea as the most
common symptom.

45. (D) The word " this" refers to "diarrhea, the most common
symptom," which appears just before it.
562 ANSWERS ANO EXPLANATIONS FOR PRACTICE TEST 3

46. (C) An incubation period means the time between a microorgan-


ism 's entry into a body and the exhibiting of the first sympu~11 s.
The first sentence of paragraph 3 says that the first sympto m
occurs " within four o r five days.··

47. (B) The fact that a person becomes ill after eating "contami-
nated" food sho uld lead you to the conclusion that it mea ns
"tai nted." If you don't know the meaning of " tainted." it 1s still
possible to e liminate the o ther three choices from the way
"contaminated" is used.

48. (D) Paragraph 2, sentence l, states, "the distribution of the


bacteria in sea wate"r was dependent on the wa ter temperature,"
but "independent of the sodium chloride content." Sodium
chloride is salt.

49. (C) Since stomach cramps are given as a symptom of the


infection, you can assume they would be unpleasant. Choice ( C)
is the most logical. Although high temperature would a lso be
unpleasant. it would not norma1ly be associated with rhe
stomach.

50. (C) " Isolation" means "separation." Another form of the word,
"isolated," appears in the following sentence.

-,
PRACTICE TEST 4

ANSWER KEY FOR PRACTICE TEST 4

After some answers in this answer key, you will find numbers in
italic type. These are page numbers in Part III where you will find
review material for t hese questions. Although any one question may
involve several different rules and concepts, these page numbers
refer to important areas you should review if you have missed a
question or are not sure of the material involved. Make full use of
these page number references and of the index to direct your
personal review.

Section 1: Listening Comprehension


l. (A) 11. (C) 21. (B) 31. (B) 41. (A)
2. (D) 12. (B) 22. (C) 32. (D) 42. (A)
3. (C) 13. (C) 23. (C) 33 . (B) 43. (D)
4. (B) 14. (D) 24. (B) 34. (D) 44. (C)
5. (C) 15. (C) 25. (D) 35. (A) 45. (B)
6. (A) 16. (A) 26. (D) 36. (D) 46. (A)
7. (A) 17. (D) 27. (B) 37. (B) 47. (C)
8. (A) 18. (C) 28. (A) 38. (C) 48. (D)
9. (B) 19. (C) 29. (A) 39. (A) 49. (B)
10. (A) 20. (D) 30. (A) 40. (D) 50. (C)

563
564 SCORING PRACTICE TEST 4

Section 2: Structu re a nd Written Expression


l. (B) 57, 59. 205, 229 21. ( D) 48-50
2. ( B ) 70 22. (B) 82
3. (A) 72-73 23. (D )
4. (C) 148-149 24. (D ) 48-50
5. (C) 125-127 25. (A )
6. (D ) 150-151 ' 26. (A) 45
7. (A ) 148, 175 27. (A) 103
8. (D ) 45, 99 28. (B) 69-70
9. (A) 45, 145 29. (D)
10. (D) 118 30. (B) 85
11. (C) 94 31. (B) 45, 68
12. (D) 68, 142-143 32. (B ) 175
13. (A) 153-154 33. (A) 89-90
14. ( A) i83-i84 34. (A ) 94-95
15. (C) 52-53 35 . (B) 79, 104
16. (C) 47 36. (C) 62
17. (A) 75-76 37. (A) 113, 118
18. (D ) 135 38. (A) 129
19. (C) 39. (C) 139- 140
20. (C) 52-53 40. (D ) 86, 289

Section 3: Reading Comprehe nsion


1. (D) 11. (A ) 21. (D) 31. (C) 41. (D )
2. (B) t2. (D ) 22. (A) 32. (A) 42. (D )
3. (B) 13. (A) 23. (C) 33. (D) 43. (A)
4. (D) 14. (B) 24. (D) 34. (A) 44. (D )
5. (A) 15. (D ) 25. (C) 35. \D) 45. (D )
6. (C) 16. (B) 26. (C) 36. (A ) 46. (C)
7. (D) 17. (A) 27. (D) 37. \A) 47. (A)
8. ( B) t8. (C) 28. (B) 38. (C) 48. (C)
9. ( D) 19. (C) 29. (B) 39. (B) 49. (A)
10. ( B) 20. (D ) 30. (A) 40. (C) 50. (A )
SCORING PRACTICE TEST 4 565

PRACTICE TEST 4: ANALYSIS-SCORING SHEET

Use the c han below to s pot your s trengths a nd weaknesses in


each test section and to arrive at your total converted score. Fill in
your number of correct answers for each <,ection in the space
provided. Refer to the Converted Score Sheet on page 496 to find
your converted score for each section and enter those numbers on
the chart. Find the sum of your converted scores, multiply that sum
by 10, and divide by 3.
Example: 1f raw scores are then converted scores are
Section 1: 33 51
Section 2: 26 49
Section 3: 38 53
Sum of Converted Scores 153
Times 10 = l ,530
Divided by 3 = 510 =Total Converted
Score
This will give you the approximate score that you would obtain if
this were an actual TOEFL. Remember that your score here may
possibly be higher than the score that you might receive on an actual
TOEFL simply because you are studying the elements of the test
shortly before taki ng each test. The score is intended only to give
you a general idea of approximately what your actual score will be.

Total Total Converted


Possible Correct Score
Section l :
Listening Comprehension I 50
I

~
Section 2:
Structure and Written Expression
Section 3:
Reading Comprehension 0

TOTALS
I 140 I
Sum of Converted Scores
Times 10 = - --
Divided by 3 = _ _ _ = Total Converted Score
566 ANSWERS AND EXPLANATIONS FOR PRACTICE TEST 4

SECTION 1: LISTENING COMPREHENSION SCRIPT

Part A

l. Man: You're so late. I thought you'd never get here.


Woman: My car broke down o n the highway, and I had to
walk.
Third Voice: Why does the woman say she had to walk?

2. Man: We missed you at Dale's party last night.


Woman: I'm not going to any celebrations with that group
because t hey're so tightly knit that they brush
everyone else off.
Third Voice: Why does the woman say she didn 't attend Dale's
party?

3. Woman: Bill. are you still planning to buy that nice red
sports car you looked at last week?
Man: I'm afraid that's impossible because haven' t
been able to come up with the cash, and someone
else has already made a down payment o n it.
T hird Voice: What does Bill say about buying a car?

4 . Man: Gail is supposed to be here at t he meeting


tonight. Where is she?
Woman: She came down with the ttu and had to stay home.
Third Voice: Why does the woman say Gail didn' t attend the
meeting?

5. Man: How many people will be corning to the reunion


on Saturday?
Woman: We had to cross fifteen names off our original list
of one hundred.
Third Voice: How many people does the woman say they
expect to attend the reunion?

--- ~
<;ECTIO'.' !·LISTENING COMPREHENSION SCRIPT 567

6. Woman: You look happy this morning.


:vlan: l j ust came fro m my adviser 's otfice and fou nd o ut
that t he •;allege board has done away with the
foreign language requirement fo r graduation.
Third Vmce: What does the man mean?

7 . \fan: i hear (hat Rusty's car is being repossessed by the


nn anct: company.
Woman : Yes, he·s fa llen behind on the payments.
Th1rn Vn1ce: Whar does the woman mean?

8. Woman: Whar did you do in class today?


Man: The leacher went ove r last Friday's lesson.
rl-i1r(i Voice: Wh;it does the man say t he teacher did in class?

9 . Wornan : ·' lot ge tting t hat job was a big letdown.


\ ifan: 0011 · • worry. Something bette r will come along.
Thu~i Vn1ce: What a re the speakers talking about ?

10. Man: Ho" do the Finleys feel about moving to New


Me>aco?
Woman: The~ ·re ri;;:ally looking forwa rd to it.
Thirct Voice: What does the woman say about the Finleys'
1·eact1on t0 moving?

l l. \fan Where a re yo u going in such a rush ?


'Vnm an: I have to deposit my paycheck before the bank
::loses. or else I won't have a ny funds to pay these
bills.
ThircJ Voice: What does the woman mean?

12. Woman: Dan, now was your visit with your sister's friends?
Man: l hardly knew the people.
Third Vnice: Wh at did t he man say a bout his siste r's friends?
568 ANSWERS AND EXPLANATIONS FOR PRACTICE TEST 4

13. Woman: You need to cut down on your sugar intake.


Man: I find it very hard to resist.
Third Voice: What does the woman suggest the man do?

14. Man: Was anyone seriously injured i 11 the accident?


Vloman: It looks as if all the victims will pull through.
Third Voice: What does the woman say about the victims of
the accident?

15. Woman: Did many people buy tickets for the rock con-
cert?
Man: So many people showed up to purchase the
tickets o n opening day that they were sold out by ·
noon.
Third Voice: What does the man mean?

16. Woman: Did Phil apply for the accounting position that
was advertised in the paper?
Man: Despite his inexperience in the field, Phil applied
for the job.
Third Voice: What does the man say about Phil?

17. Man: Who solved that difficult physics problem?


Woman: No one but Gary knew how to solve it.
Third Voice: What does the woman say about the problem ?

18. Man: What sort of response did you get on your req uest
for additional funding?
Woman: The response to my initial request was gratifying.
Third Voice: What does the woman mean?

19. Man: Will Amy be attending tonight's meeting?


Wo man: She plans to attend the meeting in spite of the
homework she needs to complele for tomorrow.
Third Voice: What does the woman say about Amy's plans?

---.-
SECTIO N I: LISTE NING COMPREHENSION SCRIPT 569

20. Man: Are you going to wait for Gil?


Wo man: He would rather that I didn't wait for him, but I
plan to anyway.
Third Voice: What does the woman mean?

21. Man: Where is Diane?


Woman: She ran o ut of milk and went out to get some.
Third Voice: Where does the woman say that Diane is?
22. Woman: 1 want to go lO lhc oonccn wnii;ln, o"'' i t :iuu-1 11 .. 1
seven, and I have to work until five. There wo n't
be enough time to go home for dinner.
Man: I've got an idea. I'll pick you up after work a::'.i
we'll eat downtown. That'll give us plenty of time
to get to the concert.
Third Voice: If the speakers follow the man's suggestion, what
will happen?

23. Man: I understand Oscar's been a little depressed.


Woman: He's got so many bills that his wife says he'll never
get ahead.
Third Voice: What does the woman mean ?

24. Woman: Where are your keys?


Man: I can't find them, but l'm sure they'll show up
soon.
Third Voice: What does the man say about his keys?

25 . Man: Did you and Stanley go to the concert last week?


Woman: We would have attended if the tickets had not
been too expensive.
Third Voice: What does the woman mean ?
570 ANSWERS ANO EXPLANATIONS FOR PRACTICE TEST 4

26. Man: I'm sorry to bother you, but I can't see when you
hold the banne r up .
Woman: Sorry. J d id n't realize it blocked your view.
T hird Voice: What will the woman probably do?

27. Woman: I thought Melanie was going to wear tha t pre tty
red wool coat you bought her.
Man: She couldn't wear it because it made her break
o ut in a rash.
Third Voice: W ha r do W'C learn {c..,rn l OIS conversation?

28. Woma n: Are you hungry now?


Man: I could sure go fo r a steak and salad.
Third Voice: What does the ma n mean?

29. Wo man: Does Jeanette like foo tball?


Man: She rarely misses a game.
Thi rd Voice: What does the man say abo ut Jeanette?

30. Woma n: How is business?


Man: O ur best agent hasn't sold a single policy this
week.
Third Voice: W hat does the man mea n?

Part B

Questions 31 through 34 are based on the follo wing conversation.

Man : I can't believe it. Tod ay I went shoppin g at the sto re near
my house instead of my usual store, and the prices we re
fa ntastic !
Woman: Is it o ne of those no-frills sto res?
Man: No, they just had some good sales, and the produce
looked better tha n it has recently at my regular store.
Woman: What kinds of things were o n sale?
SECTION I: LISTEN ING COMPREH ENSION SCRIPT 571

Man: J got a dozen large eggs fo r 85ri. beer fo r $2.75, tuna fish
fo r 99ri. a nd bleach fo r 80ri. l bought a loc of food fo r less
th en $50.
Woman: Where is this st0re? l might try it too.
Man : It's the o ne o n the corner of 16th Avenue and Main
Street.

3 1. To wha t type of store did the man go?


32. Which of the fol lowing irems did the man NOT buy on sale?
33. Whac was o ne adva ntage of this store over the ma n's regular
sto re?
34. Which of the followi ng is true about the man's shopping
experience?

Questions 35 th rough 38 are based on the following conversation.

Wo man: Good morning, Friendly Travel Age ncy. May I help you?
Man: My wife and I are pla nning a trip to San J uan and are
interested in a package deal. What can you recommend?
Wo man: Well, sir, we have a very special seven-day cruise including
San J uan and anocher port of call, tourist class accommo-
datio ns, a ll incl usive for S699 each from Miami.
Man: We don' t care much for cruises.
Woman: In that case. how about a twe lve-day trip. including air
fa re. accommodations at the Venus Hotel, several trips to
the outer islands, and two meals per day for o nly $749
each?
Man: That sounds more like what we had in mind. What else
does it include?
Woman: A sightseeing tour of O ld San Juan and El Yunque Rain
Forest, and an afternoon at Luquillo Beach.
Man: My wife would certainly love that!
Woman: When would you like to leave?
Man: March 15th.
Woman: Shall I make reservations fo r you ?
572 ANSWERS AND EXPLJ\NATIONS FO R PRACTICE TEST~

Man: Yes. Please make the m now. My name is James Morrison,


that's M-0-do uble R-I-S-0-N. I'd like to charge them to
my VISA card. My numbe r is 4555-2000-936 1-8788. T he
expiration date is in June.
Woman: After verifying your credit, we'll mail your tickets directly
to you. You should have them in three days. That will give
you plenty of time before your departure.

35. Where are the man and his wife going?


36. What is the man's name?
37. How does the man plan to pay fo r the trip?
38. When does the man wish to leave?

Pan\_;

Questions 39 through 41 are based on the following talk about roller


skating.

In an effort to fight the soaring costs of gasoline and publi:;


transportation, many athletic students have taken to rolic:; skating.
This means of transportatio n is creating traffic problems and is
presenting a safety hazard for skaters as well as motorises in college
and university areas throughout the country.
If skaters do not return to the sidewalk, but insist on causing a
dilemma for drivers and risking their own safety, the police will
issue the violators $15 citations for disregarding a city as well as a
state o rdinance. In the past monrh, seven careless students have
been injured, three seriously, as they darted into o ncomirig traffic.
Many of them refuse co wear helmets and are suffering head injuries
as a result. O ne student was thrown 50 fee t and suffered a
concussion requiring a three-week hospital stay.

39. What problem has caused students to take up skating <ts d

means of transportation?
40. What law fo rbids roller skating in the streets?
41. Which of the following is NOT true?
'iECTION I: LISTENING COMPREHENSION SCR IPT 573

Questions 42 through 46 are based on the follo wing talk about early life
in Non h A m erica.

When the early settlers, especially the English, arrived in the New
World, the hardships and dangers awaiting them were totally
unexpected. Had it not been for some friendly Indians, the colonists
never would have survived the terrible winters. They knew nothing
about planting crops, hunting animals, building shelters. nor making
clothing from animal skins. Life in England had been much simpler,
.:1nd this new life was nae like what the Spanish explorers had
reported.
The settlers introduced iron tools, muskets for hunting, domesti-
cated animals, and political ways to the Indians. In exchange, the
<;ettlers learned to build canoes for water transportation and to use
snowshoes and toboggans for winter craveling. The Indians taught
them to blaze trails through the foresc, co hunt iarge an imals and
trap smaller o ne s, and co spear fish in the lakes and streams. T he
natives also introduced to the settlers typical foods such as turkey,
;;orn, sq uash, beans, and pumpkin.
The early settlers did everything possible in order to make their
new settlements resemble the homes they had left behind.

42. Which of the following did the new settlers teach the Indians?
43. What does the speaker imply about corn, squash, and pumpkin?
44. How did the Indians teach the settlers to travel in the winter?
45 .Which of the following was NOT introduced to the settlers by
~ he lndians?
46. Which of the following is NOT true?

Questions 47 through 50 are based on the following announcement by a


bus d river.

Welcom e aboard the Luxury Cruise bus to Dallas. Baton Rouge,


a nd Atla nta. We are scheduled to arrive in Dallas at 1:45 this
:Uternoon . There will be a fifteen-minute rest stop at thac time. We
will have a thirty- minute dinner stop in Baton Rouge at 6:45 fo r
those of you who are continuing on to Atlanta. We should arrive in
)74 .i.NSWERS AND EXPLANATJONS FOR PRACTICE TEST •

Atlanta at 1:45 tomorrow morning. Please remember the number ot


your bus for reboardi ng. That number 1s 3224.
fh 1s coach is •.11r-condiuoned for your 1.:omfort. Please remember
that smoking oi cigarettes is perminea o nly in the last six rows .• ind
the smoking of any o ther material 1s prohibited, as is the drinking of
.ilcoholic beverages.
Thank you for traveling with us. Have a pleas:.mt tnp .

.+7. At what time and in what city wiil the passengers have a
fifteen-minute rest stop?
.+8. Which of the following 1s permitted in the last six rows?
.+9. What is the number of the bus?
50. At what time is the bus supposed to arrive at its final
dr.::Miuation '?

EXPLANATIONS FOR PRACTICE TEST -'

SECTION 2: STRUCTURE AND WRITIEN EXPRESSION

Pan A

1. (B) Choice (A) is verbose, using with caution rather than


cautiously. Also, it would make no sense to look cauticu~!y after
crossing the street. Choice ( A ) also use!> simp le past when past
pen·ect is required. Choice (C) is passive and the sentence does
not call fo r a passive meaning. Also. using the adjective cautwus
would indicate that look in che sentence 1s a stative verb
meaning appear. and that is not the meaning of rhe sentence.
(D) uses an incorrect sequence of tenses. The ve rb crossed is in
the past. '

2. (B) The subject notebook and repon is plural, and choicl.!s (A),
(C), and (D) all contain singular verbs.

3. (A) The plural verb need is required here because if there 1s a


plural noun after nor. the vero must be plural.

-----~-
SECTION 2: STRUCTURE AND WRITTEN EXPRESSION 575

. + . (C) C hoice (A) is incorrect because the superlative, not the


comparative, must be used when more than two are expressed.
Choices (B) and (D) are incorrect because the definite article
the must be used before the superlative.

5. (C) The correct form is would rather+ [verb in simple formJ .

6. (D) Eighc-century-old is fu nctio ning as an adjective and cannot


be plural.

7. (A) Use the comparative when only two entities are involved.
Choice (B) incorrectly uses the superlative. Choice (C) would
be correct if the smaller of them began a new sentence, but it is
11ut correct after the comma. ( D ) incorrectly uses the relative
pronoun that, which cannot be used with the preposition.

8. (D) Choice (A) includes an incorrect sequence of te nses; do


shoul"cl. be did to agree wirh endured. (B) uses negative agree-
ment, and [he sentence is positive. Choice (C) includes incor-
rect use of affirmative agreement. Also. the correct plural of
child is children.

9. (A) This is a multiple number comparative. Choice (C) is also a


multiple number comparative, but one earns money. Money is a
non-count noun, and thus the sentence requires much.

10. (D) This is a past condition. The correct sequence is had studied
... would have been able. (B) is verbose.

11. (C) The correct structure for an embedded question is question


word + subject + verb. Choices (A) and (B) incorrectly include
did. and choice (D) incorrectly uses the infinitive co put.

12. (D) Choice (A) is incorrect because the subject facilities is


plural and requires a plural verb. Choice (B) uses an incorrect
comparative. It should be becter chan. Choices (A), (B), and (C)
all use an illogical comparison. T hey seem to compare the
facilities with the new hospital. Choice (D) is correct; those of=
the facilities o/
576 ANSWERS AND EXPLANATIONS FOR PRACTICE TEST J

13. (A) Choices (B), (C), and ( D) are all missing necessary
prepositions, because of, on account of, as a result of (notice
that a necessary article was left o ut here as well).

14. (A) In this subjunctive construction use suggest that + [verb in


simple form] .

15. (C) Choice (A) incorrectly uses others, which implies that there
are more than one other. The sentence says there are two
teachers. Choice (B) incorrectly uses another, which indicates
the indefinite. A specific is required here. (D) is incorrect
uecause in this sentence other requires the article the.

PartB

16. (C) s hould be an. Use an before a word beginning with a vowel
sound.

17. (A) should be were guarding. Use a number of+ plural verb.

18. (D) should be universally. An adjective (understood) is always


modified by an adverb, never by another adjective.

19. (C) should be by incorporating. This indicates the method by


which they convey and preserve their thoughts.

20. (C) should be other systems. Other cannot be plural when it


appears before a noun.

21. (D) should be in the time of This sentence calls for specific time,
the time of Socrates.

22. (B) should be themselves. The word theirselves does not e,rjst.

23. (D) should be survival. A noun. not a gerund, is necessary here


after the preposition for.

--~
SECTION 2: STRUCI'URE AND WRITTEN EXPRESSION 577

24. (D) should be fear. In this sentence fear is indefinite and cannot
be modified by the definite article the.

25. (A) should be on a child's level o r on a childish level. Before a


noun. child must be possessive (child's) or it must be in adjective
fonn (childish).

26. (A) should be these. These is the plural of this. The plural form is
required here before the plural noun officials.
27. (A) should be hardly ever. Hardly never is a double negative and
should be avoided.

?fl. (B) should be is. Airpollution is a singular subject and requires a


singular verb.

J.Y. (D) :.ltuuld be c lose to the city. C lose to mea ns near.

30. (B) sho uld be of flying. The adjective capable req uires the
preposition of+ [verb + ing].

31. (B) should be was. News is a non-count noun and requires a


singular verb.

32. (B) should be which or thac. Thar which is redundant here


because they are both relative pronouns. One or the o ther
should be used, but not both.

33. (A) should be Joel 's. Use the possessive before a gerund.

34. (A) should be what happened. For embedded q uestions. use


question word + subject + verb. This is a subject question. so
the question word (what) is also the subject.

35. (B) should be us. Use the object pronoun after a verb. The
sentence is an indirect command.

36. (C) should be for. Use for + duration of time.


578 ANSWERS AND EXPl.ANATIONS FO R PRACTICE TEST 4

37. (A) should be would have. The conditional perfect uses would+
have -'- [verb in past participle]. Would of is never- correct.

38. (A ) should be supposed co. Use be + supposed co (means


should ).

39. ( C) should be from . Always use different f rom.

40. (D) should be of financing. T he noun means requires the


preposition of~ [verb + ing].

SECTION 3: FiCAOING COMPREHENSION

1. (D) Lic!"..::.r.s are not considered to be plants, so choice (A) is


incorrect. In mutualistic associations. both life forms benefit.
The passage explains that the algae are parasitized in lichens. so
choice ( B) is incorrect. C hoice (C) is wrong because the
association is between a fungus and an alga, not two fu ngi.

2. (B) The sentences followi ng the sentence in which " hardy" is


used describe some of the hostile places lichens can thrive, so
you should realize that ''har dy" means ·' tough." or " d urable."

3. (B) T he passage states that lichens were mistakenly thought to


be mutualistic associations where both panicipants benefit, so
choice (B) correctly defines the relationship.

4. (D ) Any of the answer choices couid describe the relationship


of the fu ngi and algae in lichens; however, you should know that
"intimate"' means in "close "' association .

5. (A) in the last paragraph. it's explained that insects glue lichens
on their exoskeletons for camouflage.

6. (C) Paragraph 2 expiains that lichens were o nce thought to


re present murualistic relationships, but this was tested by
growing different lichen iungi and algae apart.
SECTION 3: READING COMPREHENSION 579

7. (D) According to the passage, a ·' hostile" environment is one in


which few other o rganisms can flourish, so " inhospitable"
would be the best answer.

8. (B) The sentence in which lichens are described as being


endolithic goes on to explain that lichens have been found
inside of rocks in Antarctica.

9. ( D) ·' Reducing soil <!rosion" is the only one of the fo ur answer


choices that was stated in che reading passage.

10. (B) Paragraph 2 explains that experiments were conducted that


showed the fungi parasitize the algae in the lichen relationship,
but riu1C (his was nor whac scientists had originally thought.
firs t paragraph says that termites and ants have similar
l l . ( A) T he
communal habits, but that they are physically different. Choice
(B) is incorrect because there is no comparison of ants' and
termites· bodies in the passage. Answer choices (C) and ( D) are
not s uggested in the reading.

12. (D) The word "communal" is related to ·'commune" and


··community," both of which relate to how living things function
m ··social" groups.

13. (A) We are told that the reproductive termites have eyes. but
that the workers are blind and the soldiers are eyeless. Choices
(B), (C), Jnd ~D ) are true because o nly the reproductives fly,
and fty only o ne time. and soldiers are larger than workers.

14. (B) The sentence states that term ites and ants are alike in some
respects. ·'although physically the two insects are distinct." The
word "although " should lead you to understand that ·'distinct"
is the opposite of "like"; thus it means ·'different."

15. (D) The passage indicates chat each "class" has its own job, and
the word ''class" is used throughout the reading passage to
describe different ·'types" and "categories. "
580 ANSWERS AND EXPLANATIONS FOR PRACTICE TEST~

16. (B) The male and female reproductives, it is implied in


paragraph 2, fly only to develop a new colony. Choice (A ) is not
t rue becaus e t he reading in dicate s t hat a pair of reproductives
flies alo ne. Choice (B) is not true because the author states in
paragraph 3 that the worke rs make up the majority of the
colony. Choice (D) is not correct because a worker is smaller
than a soldier and does not have the hard head and strong jaws
and legs of a soldier.

17. (A ) To ·'fo und" means to "establish."

18. (C) A "cell" is an enclosed " compartment. "

19. (C) Answer choice (A) is incorrect because the majority of the
reading is not concerned wit h the destruction of ho uses. Choice
(B) is incorrect because only a portion of the reading is related
to how termites work together. Answe r choice (D ) is incorrect
because the reading passage is not significantly concerned with
the relationship of these rwo rypes of termites. Choice (C) is a
general statement about the topic of the reading.

20. (D) The sentence says, ·' Like those of ants, termite colonies
consist of different classes, each ... " The word " each" refers to
the word immediately before it, ·'classes. "

21. (D) In sente nces 2 and 3 of paragraph 2, all of the other choices
are given as purposes of the pretrial confere nce.

22. (A) The passage starts out indicating that there are problems in
the court system. The sentence containing the word "ameliorat-
ing" should lead you to understand that it means becoming
" better" or ·'improving."

23. (C) The sentence states that one suggestion is to allow districts
with too many cases to borrow judges fro m those that do not
have a " backlog," which should lead you to understa nd that
" backlog" means too many cases. or an " overload. "
SECTION 3: READING COMPREHENSION 581

24. (D) The word "viable" means "workable" or "practical."


Although the pretrial conference, according to the reading, has
no t been as beneficial as had been hoped, the small-claims
court is given as a viable suggestion for improvement. Also, the
last paragraph suggests that more innovations will be proposed
in a continuing effort to find remedies. Nowhere in the passage
is it suggested that all states should follow California's example
(A ), that the legisla ture should fo rmulate fewe r laws (B), or
that no one cares (C). I n fact, the e ntire reading concerns
suggested remedies of those who are concerned.

"'5. (C) Paragraph 1, sentence 2, says, " ... and the litigants, or
parcies, have to wait. . ." This indicates that " litigants" is
another way of saying "parties in a lawsuit" in this context.

26. (C) The last sentence of paragraph 3 indicates that a litigant


waives (gives up) his or her right to a jury trial and the right to
appeal.

27. (D) The reading passage indicates that one of the problems is
costs, which should lead you to understand that " staggering"
means " very high," "shocking,'' or " astounding."

28. (B) The sentence indicates that small-claims courts can be


beneficial, which should lead you to understand that "dispatch"
means speed, or " haste."

29. (B) T he second sentence of paragraph l says that " costs are
staggering" (overwhelming) and litigants " have to wait some-
times many years. " Also, the last sente nce of the reading says
that the problems " must be remedied if the citizens who have
valid claims are going to be able to have their day in court."

30. (A) You can ga the r from the entire passage that the "situation"
needs to be " fixed" or "improved." To " correct" the situation is
closest in meaning to "remedy."
582 ANSWERS AND EXPLANATIONS FOR PRACTICE TEST J

31. (C) Lines 12-23 discuss pretrial conferences. The sentence


beginning "The theory behind pretrial conferences . .. " shows
the apparent benefits of the pretrial conference, and . t h~
sentence beginning with " Unfortunately" is the sentence md1-
cating that pretrial conferences may not work.

32. (A) The last sentence of paragraph 2 tells us that 705 survivors
were rescued by the Carpachia . The last sentence of paragraph 1
tells us there were 2.227 passengers at the start of the voyage.
Roundi ng off, 700 divided by 2.000 is about one chird \.700 X
3 = 2,100 is another' way to estimate) that survived and were
rescued, so it could noc be true that only one third of the people
perished (died).

33. ( D) The panic of the people. the fi re o n the ship, and the speed
at which the ship was moving are all mentioned as contributing
to the disaster. The Carpachia, however, was the rescue ship.

34. ( A) Paragraph 2. sentence l. and paragraph 3 indicate choice


(A) is true. ··only rwo days at sea" and ··rwo days of sailing glory
on its maiden voyage" both indicate that it had traveled o nly
two days.

35. (0) To " extinguis h'" 1s to put o ut a ti.r e; "'unextinguishe d"


means that the fire was not stopped (was ··unquenched").

36. (A) In this context. ··maide n·· means ··fi rst" o r ··i naugu ral."
Paragraph 3 states that the "'S.S. Titanic had e n1oycd o nly two
days of sailing glory.··

37. (A) From the reading you can infer that people beiieve d the
S.S. Titanic was ··unsinkable," so yo u could assume that they
"called " ll " unsinkable." T he ~ hip was christened (C) and
probably listed ( D) in the naval registry as the S.S. Titanic, its
formal name. Ships are launched, not " initiated" (B).

38. (C) Answer choice (C) is the o nly correct :mswer. The S.S.
Titanic sank; therefore, it was not seawo rthy {A). Choice (B) is
incorrect because the Carpathia successfully rescued o ne third
SECTION 3: READING COMPREHENSION 583

of the passengers. Choice (D) is incorrect because the cause of


the d isaster was the ship's striking an iceberg.

39. (B) The last sentence of paragraph 1 states that the owners
provided less than one half the number of lifeboats and rafts
necessary for all of the passengers.

40. (C) The passage states, "Explorations and detailed examina-


tions of the base of the structure reveal many intersecting lines.
Further scientific study indica tes tha t these represent a type of
timeline of events." None of the other three answer choices is
supported by the passage.

41. (D) The prefix "extra-" means "outside" or " beyond." " Terres-
trial" refers to the earth, ~u " extraterrestrial" refers to beings
from somewhere beyond earth. The use of the word "even" in
the sentence might help you to conclude that "extraterrestrial
beings" are something out of the ordinary.

42. (D ) The author implies that there are a number of passages in


order to protect the tomb and its treasures.

43. (A) " Intersecting" lines are lines that cross one another, choice
(A).

44. (D) The passage states that researc hers have fou nd that the
intersecting lines represent historical and future events.

45. (D) To "prophesy" is to tell of the future, so choice (D ) is


correct. The " future generations" in the sentence is a contex-
tual clue.

46. (C) The passage is essentially a listing of the amazing things


about the G reat Pyramid, so choice (C) is accurate. Choice (A)
covers a fraction of the passage. That the Great Pyramid was a
massive construction project is briefly described; however, the
passage is not about construction problems, so choice (B) is
incorrect. Choice (D) is too limited because it refers only to the
buria l chamber rather than the entire pyramid.
584 ANSWERS AND EXPLANATIONS FOR PRACTICE TEST 4

47. (A) The last sentence of paragraph 1 says that they based their
calculations on astronomical observations (observation of the
celestial bodies).

48. (C) Paragraph 1, sentence l, tells us that it was built "as a tomb
for Pharaoh Cheops." Although the E~ptian s did observe the
solar system (A), a tomb would have some connection with
religious observances (B), and the pyramid was an engineering
feat (D), none of these are given as the reason for the pyramid's
construction.

49. (A ) The passage implies that the pyramiCJ is one of the seven
wonders of the world for many reasons. Two of the reasons are
1) the aligr.ment of the pyramid's fou r si:!.:s with true north,
south. east, and west and 2) the timeline on the base that
stretches into the fucure.

50. (A ) A "feat" is a notable ·'achievement," so " accomplishment"


is the correct answer. In the sentence. the adjectives describing
"fear'' are " incredible" and "engineering, ., both refe rring to
the perfect alignment of the Great Pyramid with the compass
points. That contextual information should help you eliminate
the other answer choices.
PRACTICE TEST 5
ANSWER KEY FOR PRACTICE TEST 5

After some answers in this answer key, you will find numbers in ·
italic type. These are page numbers in Part III where you will find
review material for these questions. Although any one question may
involve several different rules and concepts, these page numbers
refer to important areas you should review if you have missed a
question or arc not sure of the material involved. Make full use of
these page number references and of the index to direct your
personal review.

Section 1: Listening Comprehension


l. (C) 11. (C) 21. (B) 31. (B) 41. (C)
2. (D) 12. (B) 22. (A) 32. (B) 42. (A)
3. (B) 13. (A) 23. (C) 33. (A) 43. (D )
4. (D) 14. (B) 24. (C) 34. (C) 44. (8)
5. (C) 15. (B) 25. (A) 35. (B) 45. (C)
6. (B) 16. (A) 26. (B) 36. (A) 46. (A)
7. (B) 17. (C) 27. (A) 37. (D) 47. (D)
8. (D) 18. (B) 28. (A) 38. (D ) 48. (A)
9. (B) 19. (A ) 29. (D) 39. (C) 49. (D)
10. (A) 20. (A) 30. (C) 40. (C) 50. (A)

585
586 SCORING PRACTICE TEST 5

Section 2: Structure and Written Expression


1. (C) 175 21. (C)
2. (C) 229 22. (A) 123-124
3. (B) 23. (A) 48-50
4. (C) 188 24. (A) 94
5. (D) 135, 204-205 25. (A) 84
6. (C) 185-186 26. (D) 153-154
7. (C) 221-223 27. (B) 84
8. (A) 104 28. (B) 69-70
9. (C) 118 29. (A) 153-154
10. (B) 183-184 30. (B) 68-69
11. (A) 173 31. (D) 140
12. (B) 87, 209 32. (C) 68-69
13. (C) 81, 84-85, 89-90 33. (C) 69-69
14. (D) 227- 228 34. (B) 68-69
15. (A) 59-61, 204- 205, 221-223 35. (C) 135
16. (B) 71 36. (D) 171
17. (D) 191-193 37. (A) 227-228
18. (C) 193-194 38. (B) 129
19. (B) 183-184 39. (B) 104
20. (B) 86 40. ( B) 52-53

Section 3: Reading Comprehension


l. (C) 11. (B) 21. (B) 31. (D) 41. (A)
2. (D) 12. (C) 22. l D) 32. (A) 42. (D)
3. (B) 13. (A) 23. (B) 33. (D) 43. (C)
4. (C) 14. (A) 24. (C) 34. (B) 44. (B)
5. (D) 15. (B) 25. (D) 35. (C) 45. (C)
6. (D) 16. (B) 26. (D) 36. (D) 46. (A )
7. (A) 17. (A) 27. (D) 37. (D) 47. (A)
8. (A) 18. (D) 28. (C) 38. (D) 48. (D)
9. (C) 19. (C) 29. (A) 39. (D) 49. (B)
10. (B) 20. (A ) 30. (B) 40. (B) 50. (D )
SCO RING PRACTICE TFST ) 587

PRACTICE TEST 5: ANALYSIS-SCORING SHEET

Use the chart below to spot your strengths dnd weaknes es m


each test section and to arrive at your total converted score. Fill in
your number of correct answers for each section in the space
provided. R efer to the Converted Score Sheet on page 496 to find
your converted co re fo r each section and enter those numbers o n
the chart. Find t he sum o f yo ur conve rted scores. multiply that sum
by 10. and divide by 3.
Example: If raw sco res are then conven ed scores are
Section l : 33 51
Section 2: 26 49
Section 3: 38 53
Sum of Converted Scores 153
Ti.mes 10 = 1.530
Divided by 3 = 510 = Total Converted
Score
This. will give you {he approximate score that vou would o btain 1f
this were an actual f OEFL. Remember that your ~:core here may
possibly be higher than the score that yo u might receive on an actual
TOEFL simply because you are studying the element~ o f the cest
shortly before taking each test. The score is intended only to give
you a general idea of approximately what your actual score will be.

Total Total ConYerted


Possible Correct Score
Section 1: I
Listening Comprehension 50 I i
Section 2: !
40
Structure and Written Expression I
Section 3:
50
Reading Comprehension
I
TOTALS 140
:
S um of Converted Scores
Times JO=
- --
Divided by 3 =
- -- = Total Convened Score
588 ANSWERS AND EXPLANATIONS FOR PRACTICE TEST 5

SECTION 1: LISTENING COMPREHENSION SCRIPT

Part A

1. Woman: There were tears of laughter on the faces of


everyone in the theater.
Man: The play certainly raised some eyebrows, but it
was nothing less than hilarious.
Third Voice: According to the man and woman, how did the
audience react to the play?

2. Man: Th~ Green Dolphin sounds like a nice place tv


eat.
Woman: OK, let's go there. I hear that they have a
complete menu and a warm atmosphere.
Third Voice: Where are the man a nd woman going?

3. Man: Do you think your grandfather heard our plans


for the surprise party?
Woman: No, he's partially deaf.
Third Voice: Why does the woman say her grandfather doesn't
know about the party?

4 . Woman: Why didn' t you have your geology ctass today?


Man: Only three out of a class of twenty-five showed
up. Since the professor had planned to present a
complex demonstration, he decided to cancel the
class until everybody was present.
Third Voice: Why does the man say the geology class didn't
meet today?
SECTION l: LISTENING COMPREHENSION SCRIPT 589

5. Wo man: Since it 's the rush hour, let's take the·subway.


Man: O K. It's not as direct as the bus, but it's faste r and
there ' ll be no c hance of a traffic jam .
Third Voice: Why do the man and woman decide to take the
subway?

6. Man: I heard Doug got a ticket yesterday.


Woman: He did. He drove down a one-way street the
wrong way.
Third Voice: What does the woman say about Doug's receiving
a ticket?

7. Woman: Do you know Susan Flannigan?


Man: The name rings a bell, but I' m not sure.
Third Voice: What does the man mean?

8. Man: Roy doesn't stand a chance of winning a gold


medal in the O lympics.
Woman: True, bu t he's doing his best.
Third Voice: What do the speakers mean?

9. Woman: Somebody needs to change the cartridge in t he


cop y machine.
Man: Don't look at me!
Third Voice: What does the man mean?

10. Woman: I need to go to Chicago next week. What do you


have available?
Man: There are three nonstop flights fro m Atlanta to
Chicago each week.
Third Voice: What does the man say about the flights from
A tlanta to C hicago?
590 ANSWERS AND EXPLANATIONS FOR PRACTI CE TEST 5

11. Man: Mr. Roberts is preparing for his upcom ing vaca-
tio n.
W o man: Yes, ht:'s looki ng forward to it.
Third Voice: What does the woman mean?

12. Ma n: Why arc Maria's eyes so red?


Wo man: T hey're irritated from rhe chlor ine in the pool.
Third Voice: W hat does the woman say abou t Ma ria?

13. Man: I'm really looking fo rwa rd to moving to the new


build ing.
Wo man: Uh, I'm no t quite sure how to put this. Your
positio n is being eliminated.
Third Voice: What is the woman 's problem?

14. Woman: Did Sandra like the shoes you bought he r fo r her
birthday?
Man: She exchanged them fo r a different pair.
Third Voice: What does the man say that Sand ra did with the
shoes?

15. Wo man: It seems tha t everyone will be going on the field


trip.
Man: Don' t be too sure. Nor everyone has turned in a
consent fo rm.
Third Voice: What does the man imply?

16. Man: Why didn't Janet finish her homewo rk?


Woman: Her glasses broke, so she could n't read her
assignment.
Third Voice: What does the woman say happened to Janet?
SECTION I: LISTENING COMPREHENSION SCRIPT 591

17. Man: Leslie is taking biochemistry and advanced calcu-


lus next semester.
Woman: She's got to be out of her mind!
Third Voice: What does the woman imply about Leslie?

18. Man: We're way over budget on this project.


Woman: They must have miscalculated the cost of the new
equipment.
Third Voice: What does the woman say about the project?

19. Woman: I thought Naomi couldn't afford to go to the


conference.
Man: She couldn't have attended if her boss hadn't
paid her way.
Third Voice: What does the man say about Naomi's attending
the conference?

20. Man : Does June like the new television programs this
fall?
Woman: She dislikes television, but her husband watches
it nightly.
Third Voice: What does the woman mean?

21. Woman: It sure is a long way up to the peak.


Man: Especially on such a hot day!
Third Voice: What does the man mean?

22. Woman: Did you hear about the house that the Kehoes
bought in the country?
Man: Yes, and Chuck said that they got a very good
deal on it.
Third Voice: What does the man mean ?
592 ANSWERS AND EXPLANATIONS FOR PRACTICE TEST 5

23. Man: The program director said that we'd have to


postpone the outing until Saturday because of
inclement weather.
Woman: It's a shame. T he food has al ready been ordered
and will probably spoil.
Third Voice: Why does the man say the outing was postponed?

24. Man: I thought you said that Rob went to Sebring High
School.
Woman: No, he used to attend Clark High School, but
after graduation last year, he enrolled in Melrose
Community College where he's presently study-
ing.
Third Voice: Where does the woman say Rob goes to school?

25. Man: Have you bought Jerry's birthday gifts yet?


Woman: I've found the baseball shoes. a shirt, and a game ,
but not the bicycle.
Third Voice: Which of the following items has the woman
NOT bought?

26. Man: Do you need some help, miss?


Woman: Yes. could you give me a hand with these pack-
ages?
Third Voice: What does the woman mean?

27. Woman: Has Louise found another job yet?


Man: She's searching fo r a new job as a typist.
Third Voice: What does the man say about Louise?

28. Woman: Did Haivey know about the physics test when he
skipped class the other day?
Man: No, had he known about the test, he wouldn't
have missed class.
Third Voice: What does the man mean?
SECTION l: LISTENING COMPREHENSION SCRIPT 593

?.9. Man: I understand that Joe is not doing well in school.


Woman: It's a pity that he hates to study.
Third Voice: What does the woman mean ?

30. Woman: What is Scott doing with his children while he's in
school?
Man: He's trying to find a nursery near the university.
Third Voice: What does the man say about Scott ?

Part B

;2uewom 31 through 34 are based on rhe following con versation.

Woman: f've been hearing some strange noises under the hood for
(he past two weeks. What do you think is wrong?
'Vian: Well, your radiator is leaking, your fuel pump is broken,
dnd your carburetor is dirty.
Woman: '"low long will the repairs take?
\fan: ' can probably have it as good as new in fo ur days.
Woman: flow much will all of this cost?
\fan: \ bo ut $195.
Woman: Do you accept these ten-percent discount coupons fo r
.vork over $150?
\fan: Yes, we do. If you leave it now, I'll have it ready by Friday
itfternoon.

n . Wha< .to the speakers imply about what the woman will
probably pay?
32. How long will it take to complete the repairs?
13. What can we assume the man does for a living?
_;4. Which of the following was NOT mentioned as a problem?
594 ANSWERS AND EXPLANATIONS FO R PRACTICE TEST 5

Questions 35 through 38 are based on the following conversation.

Woman: Have you heard that Na ncy's b os s wa nts her to accept a


six-week assignment in Acapulco?
Man: Yes, but what exactly will she be doing there all that time?
Woman: Her boss wanes her to write a feature story on the regional
arts and crafts. She'll be photographing and interviewing
the local artists.
Man: She' ll r eally like that, especially since all of her expenses
will be paid and she can practice her Spanish.
Woman: Yes. but most of all. she'll get to spend her leisure hours
soaking up the sun on those lovely beaches.
Man: At night she' ll be eating that great food and listening to
the mariachi music. When will she be leaving?
Woman: Since she doesn' t need a passport, it'll probably be m
about a week.
Man: That doesn't give her much time to get organized.

35. Which of the following was NOT mentioned as a reason for


Nancy's enjoying her new assignment?
36. How soon will Nancy be leaving?
37. What is the one thing Nancy will NOT need for this trip?
38. What does the man suggest Nancy can do at night?

PartC

Questions 39 through 43 are based on the following talk about a


medical miracle.

Robert Edwards was blinded in an automobile accident nine


years ago. He was also partially deaf because of old age. Last week,
he was strolJing near his home when a thunderstorm approached.
He took refuge under a tree and was struck by lightning. He was
knocked to the ground and woke up some twenty minutes later,
lying face down in water below the tree. He went into the house and
lay down in bed. A short time later, he awoke; his legs were numb
SECTION I: LISTENING COMPREHENSION SCRIPT )
~95

and he was trembling, but, when he opened his eyes, t.e could see
the clock across the room fading in and out in fro nt of him. When
his wife entered, he saw her for the first time in nine years. Doctors
confirm that he has regained his sight and hea ring, apparently from
the flash of lightning, but they are unable to explain the occurrence.
The only possible explanation offered by one doctor was that, since
Edwards lost his sight as a result of trauma in a terrible accident,
perhaps the only way it could be restored was by another trauma.

39. What caused Robert Edwards's blindness?


40. What was the first thing that Edwards saw afte r being struck by
lightning?
41. Which of the following statements is NOT true?
42. What was Edwards doing when he was struck by lightning?
43. What was the reason given by one doctor that Edwards
regained his sight?

Questions -14 through 47 are based on the following talk about


Delaware.

Delaware is considered the first state of the United States


because it was the firs t to accept the Constitutio n, in December,
1787. It is a very small state, second o nly to Rhode Island. Another
impo rtant fact about Delaware is that nylon. tha t lightweight. yet
'itron g fiber of the twenrieth century, was invented there. l n colo nial
days. Delaware was part of the " bread basket" area, ra ising wheat.
corn. a nd ocher grains fo r national consumption .
ln 1638. a group of Swed ish settlers set up a colony alo ng the
Delaware River and lived there peacefully until 1655 wht:n the
Dutch. who disliked the Swedes, settled there. Later, it was taken
over by the English. and finally became independent in 1776.

44. What important twentieth-century fiber was invented in Dela-


ware?
45. Why is Delaware co nsidered the first state of the United States?
46. Which of the following did NOT at any time control the
Delaware territory?
47. Why was this area known as the "'bread basket"?
596 ANSWERS AND EXPLANATIONS FOR PRACTICE TEST 5

Questions 48 through 50 are based on the following talk about animals.

Adaptation is the process by which living things a djust to changes


in their environment-ways of finding food, protecting themselves
from their enemies, and reproducing. The protective adaptations
vary with each species of animal, depending on its individual needs
and environment.
Many animals possess colors that help them blend in with their
surroundings. Polar bears and Arctic foxes can easily move undetec-
ted amidst the winter snows. Many butterflies' colors make it
difficult to find them among the trees. Chameleons can change
colors to disguise themselves on rocks, trees, and wood chips.
Snakes bite; wasps and bees sting; skunks emit a pungent odor;
and porcupines release painful quills into their attackers.

48. Which of the following was NOT mentioned as possessing a


protective device?
49. What makes porcupines unique?
50. Which of the following protective devices was NOT mentioned
in this talk?

EXPLANATIONS FOR PRACTICE TEST 5

SECTION 2: STRUCTURE AND WRITTEN EXPRESSION

Part A

1. (C) Choice (A) is incorrect. What is not a relative pronoun, and


thus cannot follow a noun in this way. Choice (B) incorrectly
uses the pronoun who, which may be used only for people. The
noun immediately before it is proposal. (D) is verbose.

2. (C) Choices (A), (B), and (D) are all verbose.

3. (B) To speak of societal classes we have only the following


choices: lower class, lower-middle class, middle class, upper-
middle class, and upper class.
SECTION 2: STRUCTURE AND WRIITEN EXPRESSION 597

4. (C) Choices (A) and (B) are incorrect because the correct form
is know how + [verb in infinitive]. Choice (D) is verbose, using a
poor choice of vocabulary in "way of efficiency in study. "

5. (D) Choice (A) uses improper word order. Also, easier should
be easily (the adverb) to modify the verb, and "with hopes to be
able" should be " with hopes of being able." (B) uses an
improper sequence of tenses; can should be could. And, as in
(A), easier should be easily. Choice (C) is incorrect because the
proper idiom is hope of, not hope for.

6. (C) Cl1oice (A) sho uld read not only ... but also. (B) is
redundant. You should not say both . . . us well as, and the
choicP. 10es not include the necessary noun (ability, skill, talent,
etc. ) after artistic. (D ) is verbose and uses poor vocabulary
choice.

7. (C) Choices (A), (B), and (D) lack parallel structure. Correct
structure is will + [verb in simple form]: will wash . .. iron ...
prepare . .. dust.

8. (A) The correct form for the negative indirect command is verb
+ indirect object + not + infinitive.

9. (C) The past condition requires if . .. past perfect ... modal +


perfective.

10. (B) For the subjunctive use insisted that+ [verb in simple form] .
Choice (A) would be correct if it did not include that he, which
is redundant when used with that his patient.

11. (A) Choice (B) is incorrect because it says Let ... to enter. It
must be let + [verb in simple fo rm], "let the photographers
enter." In choice (C),permitting is in the gerund form, and a verb
in the past perfect is needed. Also permit, like allow, must be
followed by the infinitive, not the simple form. (D) uses
incorrect word order; the verb is after the complement.
598 ANSWERS AND EXPLANATIONS FOR PRACTICE TEST 5

12. (B) Choice (A) is incorrect because there is no antecedent for


the pronoun they. Choice (C) is verbose and should read either
capable of completing o r able to complete. (D) is also verbose and
uses improper word choice. You canno t "trust" abiliry.

13. (C) Choice (A) is incorrect because the committee members


did not resent the president; they resented his not infonning
them. If the sente nce meant that they rese nted the president. it
would have to say," . .. resented the president /or not informing
... " (B) is not correct because this wording would also indicate
that they reseml!d the president himself, but resent he re must be
followed by (verb + ing]. (D ) is verbose. It also should use fail+
infinitive (faild to mform ). Also, in choice (D ), rhemselves is an
improper use of the re flexive; rhem would be correct.

14. (D) This sentence involves the use of an adverbial at the


beginning of a sentence. Correct fo rm is adverbial -"- auxiliary +
subject + ve rb. The auxiliary did is in the main sentence before
the subject Anlzur. so (D) is the only possible answer. as it
begins with the adverbial only.

15. (A ) Choice tB) is incorrect because it does not use parallel


structure. Active voice ... active voice is needed. Choice (C)
- makes imprope r use of the past progressive . (D) is verbose and
makes improper use of the present perfect. Correct sequence of
tense is scurried . . . heard.

PartB

NOTE: 0 = nothing, indicating that this word or phrase should be


deleted.

16. (B) should be has. Neither must be followed by a singular verb.

17. (D) should be raise. Use raise + complement (his test score is the
cor:-.plement). Rise does not take a complement.
SECTION 2: LISTENING COMPREHENSION SCRIPT 599

18. (C) should be lying. Use lay + complement. There is no


complement in this sentence, so the verb lie, not lay, is required.

19. (B) Should be go. The correct subjunctive form is suggest that +
[verb in simple form] .

20. (B) should be going. Look forward to+ [verb+ ing].

21. (C) sho uld be 0. The preposition of is not necessary after the
preposition off.

22. (A) should be jog. Correct usage is used to + [verb in simple


form] (Mr. Anderson used to jog . .. ) or be used to + [verb + ing]
(Mr. Anderson was used to jogging . . . ).

23. (A) should be volume. Use noun + cardinal number or the +


ordinal number + noun. It is correct to say volume four or the
Jounh volume.

24. (A) should be he could have. This is an embedded question:


question word + subject + verb.

25. (A) should be to defend. Use try + infinitive.

26. (D) should be because of Use because + sentence and because


of+ noun phrase. 171e students' confusion is only a noun phrase.

27. (B) should be to suppon. Intend + infinitive.

28. (B) should be is. Congressman is a singular subject and requires


a singular verb.

29. (A) should be because of Use because of + noun phrase. Note


that " that had devastated the area" is a relative clause;
therefore, "the torrential rains" is only a noun phrase, not a
sentence.
600 ANSWERS AND EXPLANATIONS FOR PRACTICE TEST 5

30. (B) should be is. Lack is a singular subject and requires a


singular verb.

31. (D) should be than the first. The correct comparison is better
than.

32. (C) s hould be has. Cultivation is a singular subject and requires


a singular verb.

33. (C) should be is causing. Decision is a singular subject and


requires a singular verb.

34. (B) should be have been. Species (in this sentence) is a plural
subject and requires a plural verb. Species may also be singular,
but if tha t had been the case in this sentence. underutilized
would have been preceded by an.

35. (C) should be diligently. The verb had worked should be


modified by an adverb, not an adjective.

36. (D) should be pulled. The correct construction is have +


complement + [verb in past participle] . This is the rule for
passive causatives.

37. (A) should be Hardly had he. For an adverbial at the beginning
of a sentence use adverbial + auxiliary + subject + verb.

38. (B) should be change. Had better + [verb in simple form].

39. (B) should be not to. For the negative indirect command use
verb + not + infinitive. · ·

40. (B) should be others. A n means one; here others must be plural
because it is functioning as a pi onoun. It is never possible to say
anothers.
SECTION 3: READING COMPREHENSION 601

SECTION 3: READING COMPREHENSION

l. (C) A " disservice" is a harmful action. Sentence 2 says that


science has " made m any foods unfit to eat." T he read ing later
gives nitrates and ni trites as harmful su bstances that have been
added to food.

2. (D ) ·'P rone" in this context means the different cultures are


more ·'likely" to contr act certain illnesses because of their food
choices. Choice (B) can be elimina ted because it's the opposite
of " likely." Choice (C) doesn't m ake sense, and choice (A)
means lying face down , the other defin ition of " prone."
" Predisposed" is the syno nym for " pro ne" in this use.

3. (B) Paragraph 2, sem e nce 2, says that nitrates are used as colo r
preservers in meat.

4. (C) In the last sentence of the third paragraph, the letters FDA
follow the title Food and D rug Administration.

5. (D) " These" is specifying the "carcinogenic additives" that


follow it. "These carcinogenic additives" refers to the previous
sentence's " nitrates and nitrites" that caused cancer. There-
fore, " nitrates and nitrites" is the answer.

6. (D ) "Carcinogenic" means "cancer-causing." Paragraph 2,


sentence 2, states that nitrates and nitrites cause cancer. T he
following sen tence begins, " Yet, these carcinogenic addi-
tives .... " You can assume that the wo rd "these " refers to the
cancer-causing additives m entio ned in the previous sentence.

7. (A) Paragraph 3, !;entences 3 and 4, tell us that drugs are not


always administered for medicinal reasons.

8. (A) The root "add" should lead you to choose answer choice
(A).

9. (C) The whole passage discusses illnesses and the benefits of a


healthy d iet. Answer choices (A) and (B) ar e too general.
Choice (D ) is incorrect because the passage does not deal only
with " avoid ing" injurious substances.
602 ANSWERS AND EXPLANATIONS FOR PRACTICE TEST 5

10. (B) The word "fit" is contrasted with "unfit." The fact that the
sentence says "science has made enormous steps in making
food more fit to eat" should provide a clue that " fit" is
"suitable."

11. (B) The passage states "That food is related to illness is not a
new discovery" in line IO and goes on to describe a 1945 study.

12. (C) Choice (A) is not correct because the passage states that
the Egyptians left " no written accounts. " Modern embalmers
still using these methods (B) are not mentioned at all, nor is
chemical analysis (D). Sentence 4 does state specifically Lhat
"scientists have had to examine mummies and establish their
own theories," choice (C).

13. (A) The subject of this paragraph is the " embalmers."

14. (A) This is an inference question. The reading does not


specifically describe the embalming process in any of these
ways. However, you can assume that the process was not "short
and simple" (B) because in some cases it took seventy days. A
process would not be "strict and unfaltering" (C); those would
be qualities more likely ascribed to a person. There is nothing at
all in the reading to suggest that the embalming would be either
"wild" or " terrifying" ( D). Because of the several steps involved
and the time mentioned, however, it would seem logical that the
process is " lengthy and complicated," choice (A).

15. (B) Choice (A) is incorrect because the passage is discussing


the treatment of bodies that are already dead. To embalm (C) is
to preserve against decay. To rejuvenate (D) means to restore
youth. " Decay" is nearest in meaning to " deteriorate."

16. (B) You are asked fo r the o ne choice that is not true. Choice
(A) is true because the Egyptians " firmly believed in the
afterlife." Choice (C) is true. The compounds are listed as
being made up of salt, spices, and resins. Choice (D ) is not
specifically mentioned, but you should assume that it has been
difficult to determine the process since there are no written
SECTION 3: READING COMPREHENSION 603

accounts available. Choice (B), however, is false. Sentence 5


says, "up to seventy days for the pharaohs and nobility and only
a few days for th e poor, " so e mbalming did not always take
seventy days to complete.

17. (A) It was important to the Egyptians that corpses did not decay
(sentence 2), so choice (A) is correct. Scaring robbers away (B)
is not mentioned in the passage. Encasing a body in a sarcopha-
gus (C) ,vas part of the entire burial ritual for a pharaoh or
noble, but it followed mummification and was nor the reason for
it. Amulets are described as protecting the body from harm on
its journey (0), not the mummification process.

18. (D) In the first paragraph, it's stated that food, clothing, jewels,
and tools provide for the deceased's material needs. In the
-;econd paragraph, the need for protection on the long journey
to the afterlife is mentioned. It can be inferred from these two
statements that the Egyptians believed material items were still
needed by the deceased because they were on a long journey to
the afterlife.

19. (C) ·' Amulets" are charms that protect against injury or evil.
·'Curses,'' choice (D ), can be eliminated because the entire
passage relates the respect the Egyptians had for the deceased.
Curses are also not tangible. There are no clues to eliminate
choices (A) and (B) if you do not know the meaning of
''amulets."

20. (A) Substitute the answer choices for "accomplished" in the


-;entence. 'Performed" is the only verb that makes sense.

21. (B) The passage states that " the embalming process might have
taken up to seventy days for the pharaohs and only a few days
for the poor. ..

!2. l 0 ) The econd sentence states that some tapeworms attach


themselves to the intestinal wall; thus they do not float freely.
604 ANSWERS AND EXPLANATIONS FOR PRACTICE TEST 5

23. (B) The excretory system is responsible for removing waste


from a body. Only choice (B) relates to elimination.

24 . (C) Paragraph 2, sentence 3, explains that a hermaphrodite has


both male and female sexual organs.

25. (D ) The subject of the previous sentence is " some tapeworms."


So " others" means "other tapeworms."

26. (D) Euphoria is not mentioned as a symptom. Irregular


appetite, nervousness, and anemia, which mean the same as
answers (A), (B), and (C), are mentioned.

27. (. D) We are told in paragraph 1 that some tapeworms attach


themselves to the intestinal wall to feed, while others float
freely and absorb food through their body walls.

28. (C) The reading gives general information about a particular


parasite. the tapeworm. Choices (A) and (D) are too broad in
scope, and choice (B) is too narrow.

29. (A) Sentence 2 of paragrap h l mentions that some tapeworms


attach themselves by means of suckers in their heads.

30. (B) "'A tapeworm consists of numerous segments" is the


statement, so choices (A) and (D ) do not make sense. The only
organs mentioned in the passage are hermaphroditic sexual
organs, and they are located in each segment, so choice (C)
does not make sense. "'Segments" is nearest in meaning to
"sections."

31. (D) "' Foresaw" means to have known beforehand, which is


nearest in meaning to " predicted. "

32. (A) T he third se ntence of paragraph 1 says that the prize was
established to recognize "worthwhile contributions to human-
ity."
SECTION 3: READING COMPREHENSION 605

33. (D) The last sentence of paragraph 3 says that Americans have
won "numerous science awards."

34. (B) Choice (A) is true. The awards vary fro m $30,000 to
$125,000. We are told specifically that politics sometimes plays
an important role in the selection (C) and that some people
have won two prizes, although chat is rare (unusual). If it is rare,
then only a few will have done so (D). Choice (B) is not true.
The date December 10 is not important in commemorating
Nobel's invention, but rather the anniversary of his death.

35. (C) Paragraph 1, sentence 4, says that there were originally five
awards, and economics was added in l968. The total, then, is
six.

36. ( D) An inventor of dynamite would most likely be working in


the field of science.

37. (D) The contributions that the Nobel Prize winners make to
humanity are most likely "valuable."

38. (D) Nobel's original legacy was S9,000,000 (paragraph 2).

39. (D) Choices (A) and (B) are details of the passage, not the main
idea. Choice (C) is not stated in the passage. Leaving $9,000,000
to support people who make valuable contributions to humanity
is in itself a great contribution to humanity (D).

40. (B) A "legacy" is the property or money given to another at


death, so "bequest" has most nearly the same meaning.

41. (A) You are asked to choose the best summary of the passage,
which means the statement that best tells the general idea.
Choice (B) is the opposite of what the reading says. Choices (C)
and (D) may be crue, but they are too specific to give the general
idea of the e ntire passage. And while you might assume that
verbalization is the fastest form of communication (D), the
reading does not mention this.
606 ANSWERS A ND EXPLANATI ONS FOR PRACTICE TEST 5

42. (D ) "These" is an adjective describing " symbols," and " sym-


bols" is a term that can be used to describe t he motions of sign
language explained two sentences earlier.

43 . ( C ) The deaf, although they cannot hear, sometimes can speak,


bu t the mute, by definition, cannot sp eak. Therefore, they could
not themselves use oral communication.

44. (B ) Blind people cannot see, so c hoices (A), (C), and (D) wo uld
not be used by them. Braille 1s read with t he fi ngertips
(p a ragraph 3).

45. (C) There are nine forms of communica tion listed in the
reading: oral speech. sign la nguage, body langu ~Je , Braille,
signal flags, Morse code. smoke signals. road maps. and picture
signs.

46. (A) C hoices (C) and (D) a re described in the following


sentences, so they can be eliminated. To "wink" is to close on e
eye briefly, a nd to ·' b link" is to close bo th eyes b riefly.

47. (A) The last sentence of paragr aph l says that these symbols
(sign language) cannot be used internationally fo r spelling.

48. (D ) Sentence 2 describes the expression of thoughts a nd


feelings as t he reason for communicatio n, which is introduced
in sentence 1.

49. (B) Since the passage focuses o n co mmunica tion. it would be


appropriate to use th at word in the title. Choice (B) covers the
passage's topic while choice (D) is too narrow.

50. (D) A nonlinguistic code would most likely be used by a


telegrapher.
PRACTICE TEST 6
ANSWER KEY FOR PRACTICE TEST 6

After some answers in this answer key, you wili find numbers in
italic type. T hese are page numbers in Part III where you will find
review material for these questions. Although any one question may
invo lve several different rules and concepts, these page numbers
refer to important areas you should review if you have missed a
question or are not sure of the material involved. Make full use of
these page number references and of the index to direct your
personal review.

Section 1: Listening Comprehen sior1

l. (C) 11. (B) 21. (C) 31. (D) 41. (A)


2. (B) 12. (C) 22. (A) 32. (A) 42. (A)
3. (C) 13. (A) 23. (B) 33. (C) 43. (B)
4. (A) 14. (B) 24. (B) 34. (C) 44. (B)
5. (D) 15. (D) 25. ( B) 35. (D) 45. (D)
6. (B) 16. (C) 26. (A) 36. (B) 46. (D)
7. (B) 17. (D) 27. (C) 37. (D) 47. (D )
8. (B) 18. (A) 28. (C) 38. (D) 48. (B )
9. (C) 19. (C) 29. (C) 39. (B) 49. (B) .
10. (C) 20. (B) 30. (A) 40. (C) 50. (A)

607
608 SCORING PRACTICE TEST 6

Section 2: Structure and Written Expression


l. (D) 135, 229 21. (B) 59
2. (B) 22. (B) 59--00
3. (A ) 146 23 . (B) 65--06
4. (A ) 221-223, 229-230 24. (C) 89-90
5. (A) 59-61 25. {A) 64
6. (B) 224-226 26. (C) 205
7. ( D) 27. (B) 68
8. (D) 28. (B) 69-70
9. (B ) 94 29. (B) 72-73
10. (B ) 118 30. (C) 212
11. (C) 31. (A)
12. (C) 229 32. (A) 89-90
l3. (D) 33. (A) 273
14. (A) 229 34. (C) 171
15. (B) 192, 219-220 35. (A) 185-186
16. (D ) 221-223 36. (C) 289
17. (D) 221-223 37. (C) 123- 124
18. (B ) 82 38. {A ) 59, 61--02
19. {C) 205 39. (C) 52-53
20. (D) 157 40. (D) 45

Section 3: Reading Comprehension


1. (C) 11. (C) 21. (D ) 31. (A ) 41. (A)
2. (A) 12. (B) 22. (A) 32. (C) 42. (D)
3. (C) 13. (A) 23. (B) 33. (B) 43. (B)
4. (C) 14. ( B) 24. (A) 34. (B) 44. (B)
5. (C) l5. (A) 25. (A) 35. (D ) 45. (B)
6. (A) l6. (A) 26. (C) 36. (A) 46. (C)
7. (C) 17. (A) 27. (B) 37. (A) 47. (D)
8. (A) 18. (A) 28. (B) 38. (C) 48. (A)
9. (B) 19. (A) 29. (B) 39. (B) 49. (D)
10. (C) 20. (C) 30. (C) 40. (A) 50. (B)
SCORING PRACTICE TEST 6 609

PRACTICE TEST 6: ANALYSIS-SCORING SHEET


Use t he c hart below to spot your strengths and weaknesses in
each test section and to arrive at your total converted score. Fill in
your number of correct answers for each section in the space
provided. Refer to the Converted Score Sheet on page 496 to find
your converted score for each section and enter those n umbers on
the chart. Find the sum of your converted scores, multiply that sum
bv 10, and divide by 3.
Example: If raw scores are then converted scores are
Section 1: 33 51
Section 2: 26 49
Section 3: 38 53
Sum of Converted Scores 153
Times 10 = 1.530
Divided by 3 = 510 = Total Converted
Score
This will give you the approximate score that you would obtain if
this wer e an actual TOEFL. Remember that your score here may
possibly be highe~ than the score that you might receive on an actual
TOEFL simply because you are studying the elements of the test
shortly before taking each test. Thi! score is intended only to give
you a general idea of approximately what your actual score will be.

Total Total Converted


Possible Correct Score
Section 1:
Listening Comprehension
50

Section 2:
40
Structure and Written Expression
Section 3:
50
Reading Comprehension

TOTALS 140

Sum of Converted Scores


Times 10 = - - - -
Divided by 3 = - - - - = Total Converted Score
610 ANSWERS AND EXPLANATIONS FOR PRACTICE TEST 6

SECTION 1: LISTENING COMPREHENSION SCRIPT

Part A

1. Woman : Mark can't stand rare meat.


Man: I know. I ordered medium well. We'll send it
back.
Third Voice: What do the speakers mean?
.... Man: Ugh, this milk is sour!
"'·
Woman: It should be good. The expiration date is five days
away.
Third Voice: What are the man and woman talking about?

3. Woman: Have you seen that movie a bout the girl who had
sixteen different personalities?
Man: No, and I don 't plan to. lt sounds scary.
Third Voice: Why does the man say he doesn 't want to see the
movie?

4. Man: I haven' t seen you wear that lovely necklace


before.
Woman: It was packed away until last week. It's a family
heirloom.
Third Voice: What does the woman say about the necklace?

5. Woman: The neighborhood convenience store was held uo


last night.
Man : Yes, l heard it o n t he radio this morning.
Third Voice: What does t he woman say happened at the
convenien ce store last night?
SECTION I: LISTENING COMPREHENSION SCRIPT 611

6. Woman: Do you think this skirt goes well with this blouse?
Man: Yes, but I t hink your red dress would be more
elegant for the reception.
T hird Voice: What does the man say about the woman's c hoice
of clothing?

7. Woman: ~wish Jack were coming to visit us.

Man: He won't be able to because it's out of his way.


Third Voice: What do the speakers mean?

8. Man: I hope you will back me up o n this new curricu-


lum proposal.
Woman: You can count on me!
Third Voice: What does the woman mean ?

9. Man: Miss. can you give me change for a dollar?


Woman: I'm sorry, -;ir. I'm not allowed to give c hange
without a purchase. If you go across the hall,
you 'll find a change machine m fro nt of the
jewelry store.
Third Voice: Where does the woman suggest that the man get
change?

10. Man: It wasn't supposed to rain today.


Woman: It'll let up, won't it?
Third Voice: What can be inferred about the weather?

11. Man: I should submit an offer on the house today,


shouldn' t I?
V.'0man: You'll have to decide that for yourself.
Third Voice: What does the woman mean?
612 ANSWERS AND EXPLANATIONS FOR PRACTICE TEST b

12. Woman: I hear your son is working part-time at the


department store.
Ma n: Yes. He works Monday, ·w ednesday, and Friday
from three to seve n and all day Saturday.
Third Voice: Which days does the man imply t hat his son does
NOT work?

13. Man: Are you prepared fo r the test, Cindy?


Woman: I fo und it extremely difficult to lea rn all tha t
material.
Third Voice: What does the woma n say about her p reparation
fo r the test?

14. Woman: Did Bob study yesterday?


Man: He would have if it hadn' t been such a nice day.
Third Voice: What does the ma n say Bob did yesterday?

15. Woman: Why was Susan so la te the o ther night?


Ma n: She could hardly find a sea t in the dark t heater.
Third Voice: What does the man say happened to Susan?

16. Wo man: We re there a lot of students in the class?


Man: Before th e class bega n, a dozen students were in
the room. but soon the number doubled.
Third Voice: What does the ma n mean ?

17. Wo man: How was your class?


Ma n: Contrary to what I had expected, the professo1
canceled it.
Third Voice: Wha t does the man mean ?
SECTION I: LISTENING COMPREHENSION SCRIPT 613

18. Man: How would you ~ ike your two pounds of pork
chops sliced?
Woman: Medium thin will b e fine.
Third Voice: Where does this conversation probably take place?

19. Man: Which of the boys is Henry Adams?


Woman: The one with che 5reen sweater, using rhe
crutches.
Third Voice: What does the woman imply about Henry Ad-
ams?

20. Woman: That famous science fiction writer Isaac Asimov's


new book is coming out in I uly.
lvl-ro. ~
We c>rnbably won't be able to find a library copy
umil Scpte-mber .
Third Voice: When does the woman say Asimov's book will be
published?

21. Man: Where's Katie?


Woman: I don't know, but 1f she doesn't get here soon, we
will probably be late.
Third Voice: What does the woman say about Katie?

22. Man: What are you doing t his weekend?


Woman: I can't remember whether Tony and I are going to
a party on Friday o r Saturday night.
Third Voice: What are t he woman and Tony planning to do?

23. Woman: Do Helen and her husband go fishi ng together?


Man: Helen dislikes going fis hing with her husband,
even though she goes quite often.
Third Voice: What does the man mean?
614 ANSWERS AND EXPLANATIONS FOR PRACTICE TEST 6

24. Man: Have you seen Karl?


Woman: Yes. I was puzzled by the expression on his face.
Third Voice: What does the woman mean?

25. Man: Clint looked worn out.


Woman: He must have been exhausted after t hat run .
Third Voice : 'Whal UOCI> tile woman say about Clint?

26. Man: When can I see Dr. Jones?


Woman : She should be free to see you after noon romor-
row.
Third Voice: When does the woman say the man can have an
appointment? 1
27. Woman : Sally ' ~ not vc:ry ~ociab le. IS she ?

Man: In spite of her bad habits. she has a lot of friends.


Third Voice: What doe~ the man ~ay abo ut Sally?

28. Man: Ramona, are you and your husband going to buy
a ne w house this year?
Woman: Houses are so expensive now that we simply can "t
afford to buy o ne.
Third Voice: What does t he woman say about the cost of
houses?

2.9. Wo ma n: Can I still register fo r t he economics class?


Man: It's too late. Registration closed t he day before
yesterday.
Third Voice: What does t he man say .:tbout registration for
classes?

30. Man: Why isn't Elaine going to the country with us?
W oman: I fo rgot to tell her about our change in plans.
Third Voice: What do t he speakers say about E laine?
SECTION I; LISTENING COMPREHENSION SCRIPT 615
,,
Part B

Questions 31 through 34 a re based on the following conversation.

Man: Well, that's the last straw. I've reached 200 pounds, I'm
out of breath, and nothing fits me anymore. How in the
world can I break this cycle ?
Woman: I lost thirty pounds last year, and I can' t tell you how much
better I feel.
Man: I lost fifteen o nce but gained it right back. You look great.
How did you do it?
Woman: It requires a change in lifestyle, permanently. I cut down
o n fat in take and o ther problem foods, including sweets
and alcohol, and 1 got into a regular program of exercise.
Man; I'm a member of a gym, but it's so boring. I feel
inadequate next to those muscular and slim bodies.
Woman: That's how they got perfect, by going to the gym and
watching what they ate. If you don't want to go there,
swim o r ride a bicycle. Take a brisk walk after each meal.
Here, let me lend you these two books. One has informa-
tion on the fa t con:ent of certain foods-sb you know
what to avoid. And the other has menus and recipes for a
great number of healthy, low-fat meals. But remember,
the change in food is not enough. You must get the
exercise too.
Man: I guess I'll just have to draw the line and do without the
foods I love, as you recommend. No more heavy meals,
desserts, or nightly cocktails. I'll also have to force myself
to get that exercise. Maybe I' ll even go back to the gym.
Woman: Right. And brown bag it instead of going o ut to lunch at
work. Stock up on harmless goodies. If you get hungry,
nibble on some celery or carrots. If you follow the recipes
in this book, you'll see that dieting doesn' t have to be
unpleasant.

31. What is the man's problem?


32. How does the woman suggest that he solve his problem?
33. What does the woman suggest that he do when he's hungry?
?i4. What does the woman mean 'o"j the e:x.\)tessi.on " brown bai i.t"'?
616 ANSWERS AND EXPLANATIONS FOR PRACTICE TEST 6

Questions 35 through 38 are based on the following conversation.

Man: Yesterday, we were discussing the fa mo us poet, Gabriela


Mistral. Who can tell me somethin g about her?
Woman: She was from Chile a nd Jived in poverty in her early years.
She became a teacher a nd then a writer.
Man: What was her claim to fame?
Woman: She wo n the Nobel Prize in literature in 1945. She was the
tirst Latin American woman to do so.
Man: Why <•re her works so significant?
Woman: She addresses social issues such as maternity, love,
children, gender eq uity, a nd the plight of the downtrod-
den. Her the mes have been nourished by her own
personal sorrow.
Man: Did she teach only in her native country?
Woman: No, she was invited by the governments of Mexico, Spain.
France, the United States, a nd Germany to teach in
several universities.
Man: She was well loved by her colleagues and readers a nd has
left us a treasure of poetic works.

35. What is the probable relationshio between the man and t he


woman?
36. Where was Gabriela Mistral born !
37. What was Gabriela Mistral's profession?
38. According to this conversation. which country was NOT men-
tioned as o ne where Gabriela Mistral had taught?

Pact C

Questions 39 through 43 are based on the foll.owing ialk about ear~y air
transponation.

Al most two centuries ago. humans e njoyed their fi rst airborne


ride in a cloth balloon. Passengers rode in a basket fastened below
the balloon. These br ave adve nturers depended solely on the wind
velocity a nd direction "to move t he m about because of the lack of a
steering mechanism.
SECTION I: LISTENING COMPREHENSION SCRIPT 617

In 1852, a French clockmaker flew t he first controllable balloon a


distance o f seventeen miles. Germany began producing and usin g
airships about forty-six years la te r with its famous zeppelins, na med
in honor o f their inventor, Count von Zeppelin. The la rgest and
probably most famous of Germany's airships was the Hindenberg,
which could travel at eighty-five miles per hour.
Later, t he two countries bordering o n the E nglish Channel, G reat
Britain and France, built smaller airships caJied " blimps." The
latter airships were intended for patrolling the coast a nd observing
su bmarine activity, while the fo rmer served as passenger a nd cargo
ships.
After the Hindenberg burned in 1937, more and mo re people
shied away from this form of transportation.

39. Why was it difficult to fty in th e air-filled balloons of two


hundred years ago?
40. Which of the following countries was NOT involved in the
produ ctio n of airships?
41. Who flew t he firs t co ntrollable balloon?
42. Whic h country used these airships for passenger a nd cargo
t ra nsport?
43. Why did fewer people travel on airships after 1937?

Questions 44 through 46 are based on the following talk about animal


life spans.

All living, self-propelled beings do not e njoy t he same life span.


Scientists have discovered tha t the faster a livin g thing grows a nd
moves during its life, the shorter its life will be. Animals producing
many offspring will have shorter lives than those that produce o nly a
few. Larger animals live longer th a n smalle r ones. Some species live
several weeks, while o thers can enjoy more t han a one-hund~ed­
year existence.
D isease and o ther e nvironmental conditions a re capable of
wiping o ut a particular species in a given area.

44. Based o n the in forma tion in this talk, which of the following will
p robably live only a short time?
618 ANSWERS AND EXPLANATIONS FOR PRACTICE TEST 6

45. Which of the following can be expected to live the longest?


46. Which was NOT mentioned as a cause for shortening an
animal's life span?

Questions ·./7 through 50 are based on rhe following lecture about


dinosaurs.

It was not until 1822 that scientists learned about the existence of
dinosaurs. Tha nks to an E nglish doctor an<l his wife, tbe door was
opened to this zoological study. Reasoning that the reptiles'
tremendous size mus t have-made them terrible t.--reatures, scientists
combined two G reek words, dezmos, meaning terrible, a nd sauros,
meaning lizarcls, to form the word dinosaur.
After many years of study, they deccrmined thac these beasts
roamed the .!arch for millions uf years, and ceased to exist some
sixty million year s ago.
Unbelievable as it may seem. not all dinosaurs were carnivorous.
thac is, meac eating. Many were herbivorous, or vegetarian.
By reas!>embling the bones fo und at excavation sires, scientists
have been able to reconstruct the skeletons and learn a great deal
about the Jinosaurs· living conditions. They have learned that
dinosaurs inhabited not only the land. but also t he water a nd sky .

.i7. By what name did scientists refer to these creatures?


48. When <lo scientists believe that the last of the dinosaurs
disappeared?
.i9. How have scientists been ab le co learn of the living conditions of
these a nimals?
50. Which of the following is NOT true of these animals?
SECTION 2: STRUCTURE AND WRITIEN EXPRESSION 619

EXPLANATIONS FOR PRACTICE TEST 6

SECTION 2: STRUCTURE AND WRITTEN EXPRcSSION

Part A

l. (D) C hoice (A) i~ incorrect because the verb studied should be


modified by a n adverb. badly. (B) is in e rror because it is never
.;orrect to say wise with a noun or verb to mea n in relation to. (C)
uses badly, which is an adverb and cannot modify the noun
srudent.

2. (B) The t!xpression second only t0 here means that Harvard's


programs are the best, and this unive rsity's programs a re
second best.

3. (A) This is a double co mparative. The correct fo rm is the more


... the less.

4. (A) Choice (A) contains correct parallel structure: d 11ruh1/iry


(noun ) ... economy (noon). Choice (B) is verbose. Lasrs ,, :m:~
time means it is durable. It is not necessary to use so I1uny
words. Whe n there is a short e r a nswer t hat means the same ana
1s gramma tically correct. choose the shorter a nswer. Choice ( C)
would be co rrect if it said "its durability an d economy" (noun /
noun). C hoice (D) uses economy-wise. It is always in correct to
use wise with a noun in this way.

5. (A) Past progressive: when ... simple past ... past progressive.
Choice ( D) would be correct if it said had been sleeping.

6. (B ) The fo rm should be su bject + verb -r indirect object -r


direct object. There should be 110 preposiuon.

7. ( D ) The correct expression 1s tn grearer numbers. This is an


.:xpressio n that vou ~ho uld memorize.
620 ANSWERS AND EXPLANATIONS FOR PRACTICE TEST 6

8. (D) Always after the phrase it was not until must appear the
word that. To use when here would be redundant.

9. (B) For an embedded question, use question word + subject+


verb.

10. (B) Past condition. When the conditional perfect is used in the
result clause, the past perfect must be used in the if clause.

11. (C) There are o nly a few possibilities for expressing age in
English: ( 1) when he was sixteen (years old), (2) at (age)
sixteen, and (3) at the age of sixteen. Choice (C) follows rule (3)
and is the only correct answer.

12. (C) Choice (A) is verbose and too informal for written English.
Choice (B) uses make. . expressions, which has lo do with facial
fea tures, not sp eech, anc.. makes no sense here. In choice (D)
with their minds open (having open minds) is an idiom meaning
to be willing to have no biases. When used in this context with
talk, it is not logical.

13. (D ) Much less is used in this context in a negative sentence to


indicate that the second item mentioned is disliked even more
than the first. The students dislike reading novels and dislike
reading textbooks even more.

14. (A) Choices (B) and (D) are verbose. Eagerly is much more
concise than either of these c hoices. Choice (C) uses eager-
nessiy, which is not a word. It is not possible to add a n adverb
affix (-ly) to a noun affix (-ness) .

15. (B) Choices (A), (C), and (D) are all verbose. In addition,
choice (D) includes the wrong verb (raised}; (A ) and (D )
incorrectly use the time when. lt should be the time that because
the time when is redundant; c hoice (C) uses incorrect word
order.
SECTION 2: STRUCTURE AND WRITIEN EXPRESSION 621

PartB

NOTE: 0 = nothing, indicating that this word or phrase should be


d\!leted.

16. (D) should be 0. Parallel structure would be adjective / noun,


adjective / noun, adjective / noun. The phrase working at is not
necessary because the verb is in the main clause.

l 7. (D) should be cruelty. Parallel structure requires noun (strength),


noun (power), and noun (cruelty).

18. (B) should be themselves. The form theirselves does not exist.

19. (C) should be would receive. The sequence of tenses should be


past . . . past.

20. (D) should be such a short time. Cause / effect: such + a +


adjective + singular count noun.

21. (B) should be turned. He finished taking classes in 1978;


therefore, the verb must be in t he past tense.

22. (B) should be was hunying. Past progressive: when . . . past tense
... past progressive.

23. (B) should be had stolen. Use the past perfect: had + [verb in
past participle].

24. (C) should be dog's. Use the possessive form before a gerund.

25. (A) should be has been hoping. Use the present perfect
progressive: has been + [verb + ing].

26. (C) should be had been driving. The correct sequence of tenses
is after driving . . . had been driving. After driving is past in this
sentence. and has been driving is present.
622 ANSWERS AND EXPLANATIONS FOR PRACTICE TEST 6

27. (B) should be is. The Depanment of Foreign Languages is


singular so the ve rb must be singular.

28. ( B) should be is. ,.lccompanied by is a prepositional p hrase a nd


therefore is not pa rt of t he su bject. The subject is winner, which
is singular.

29. (B) should be knows. Neither is singula r a nd requires a singular


ve rb.

30. ( C) sho uld be your. Those ofyou is t he su bject. so we must keep


the sam e person pronoun fo r the possessive (your).

31. (A) sho uld be zoology book. It is not correct to say a book of
- - - -- for textbooks.

32. (A) sho uld be Mana 's. Use the possessive fo rm befo re a geru nd.

33. ( A) should be lend or loan. This is incorrect vocabulary choice.


(I a m t he receiver and Jane is the give r.)

34 . (C) should be eat. Causative: make + [verb in simple form].

35. (A) sho uld be not only composes. The acto r does two different
things. using two different verbs, composes a nd sings. Not only
must precede the fi rst verb because there a re two ve rbs.

36. (C) should be of rlbour is an inco rrect prepositio n with sample.


ft sho uld be a sample of something.

37. (C) should be used. Use be + used to + ge rund and used to +


sim ple fo rm.

38. (A) should be called. Last nif?ht is a specific time a nd requires


the simple past tense. not the prese nt µeriect.

39. (C) should be miters. Another is smgular. Others must be plural


co agree with their.

..o. (D) should be;ewer. Movies is a count noun, so 1t req uiresje1ver


001 less.
SECTI ON 3: READING COMPRE H ENSION IJ23

SECTION 3: READING COMPREHENSION

L (C) Parag r a ph 2 tells us th at he p u b lish ed th e New York Weekly


Journal, a newspa pe r his wife continued to publish while he was
imprisoned.

2. (A) Pan•graph 2, last sente nce, specifically speah of "corrupt


gove rnme nt officials."

3. (C) Paragra ph L last se nte nce, says tha t the righr (freedo m 1Jf
the press) was adopted in 1791 an d tha t t he Z enge r trial was in
1735. 1791 - 1735 = 56.

4. (C) You a re asked fo r t he o ne cho ice tha t is no r true. Cho ice


(A) is true. It is mentio ned in paragraph 2. C hoice (B) is true.
We a re told tha t the jury was ··persu aded" by Ha milton. A nd it
sho uld be obvious tha t they were figh ting ·'for freedom"
because the e ntire reading conce rns freedom of the press.
C hoice (D) is true . We a re told in pa ragraph 2 cha r the king sent
corrup t officials -'to gove rn th e colon ies .. ,

5. (C) Pa ragra ph '.!, se nte nce 3, specifically sta tes t ha t he was


arrested for " writing a story about the crown-a ppo mted ~over­
nor of New York. "

6. (A) Choice (A) is the a nswer because the governor was crown
appointed. a nd if the judge wanted to stay in office, he would
represent the king. Choice l B) is incorrect because it was not
men tio ned in t he passage. C hoice (C) is incorrect because the
Cons titutio n was adopted 56 years after the trial. Cho ice (D ) is
incor rect because th e governor is described as "crown a p-
pointed." Cho ices IC) a nd (D) are tricky because of the tacts
(C) a nd the voca bulary "crown a ppointed " (D ).

7. l C) S ubstitu te th e a nswer choices for "'defying" in the sente nce.


"Disregard ing" is closest in meaning to ''defying." If yo \J a re
not fam iliar wit h the word .. defying, ·• you can inter that it means
"disregarding' · because che passage says the judge o rdered a
gutl ty verdict, b ut the JUry fou n d Zenger not guilty.
624 ANSWERS AND EXPLANATIONS FOR PRACTICE TEST 6

8. (A ) Zenger's wife continued to publish the newspaper every


day, including articles abo ut corrupt governme nt officials,
which was the very act th at landed her husband in prison. Tha t
context should tell you that "dutifully" is closest in meaning to
" faithfully."

9 . (B) The passage is about Peter Zenge r and the right to publish
information freely, so look for those ke y wo rds a nd ideas.
Andrew Hamilton played an important ro le in the trial;
however, he did not give Americans freedom of the press, so
answer choice (A) is not correct. Choices (C) and (D ) focus on
law ra tJier than publishing, so tht.y can be eliminated.

10. (C ) Line 9 specifically states that the governor was "crown


a ppointed," which means he was a ppo inted by a mona rch.

11. (C) Paragraph 2 says. ''In A.D. 800, C harlemagne ... initiated
the Caro lingian ren aissa nce . .. " Altho ugh the Roman civiliza-
tion is mentioned in the passage . .ind is older, it is not given as
an example of a civiliza tion involving rebirth.

12. ( B) Paragraph 2 says that d uring the Carolingian period,


modem cities were "patterned on Roman a rchitecture." You
could eliminate choices (C) a nd (D ) immediately, as the
question asks fo r a "city" and these two choices are not city
names.

13. (A ) Choices (B), {C), and (D ) are specifically mentioned in the


reading. If you know tha t "status quo" mea ns the ·'existing
condition," you could realize t hat this would not be appropriate
for a time of rebirth.

14. (B) This is an inference question. Fro m the fact that we a re told
of the .. a ble rule" of Yaroslav, we can assume that he was in
general a competent leader . Also, given his name, Yaroslav the
Wzse. o ne would not assume he was demented, inept, or c ruel,
all negative qualities.
SECTION 3: READING COMPREHENSION 625

15. (A) The word " carryover" means most nearly the same as
" remnant. " One can assume from the context that the libraries
were something that came from Alexandrian Egypt. The o nly
o ther choice that is close is (B), " residue." But " residue" has
the connotation of a chemical process and thus is not the best
choice.

16. (A ) Education and learning are mentioned in the descriptions


of a ll three renaissance periods. C hoice (C) is incorrect, and
choices (B) and (D ) are not common to all three periods.

t 7 . ( A ) " Renaissance" means " rebirth" in the three civilizations.


specifically a rebirth of Greek a nd Roman culture. Art tech-
niques, patterns of architecture, a nd educational systems are all
mentioned as being borrowed a nd improved upon.

18. (A) The Italian renaissance lasted from 1400 until 1600, or 200
ye a rs. The Carolingian renaissance lasted throughout the 800s,
or 100 years. T he K.ievan renaissance also lasted a century, or
100 years. Therefore, the answer is (A), the Italian renaissance.

19. (A ) The Italian renaissance was most recent and lasted twice as
long as the o ther two periods of rebirth. It also spread
throughout Europe, and t he New World through exploration.

20. (C) Choice (A ) is incorrect because all of the cultures improved


on what they knew of previous cultures, rather than merely
copying what predecessors had accomplished. Choice (B) is
incorrect because Russia is not in western Europe. Choice (D )
is incorrect because other cultures, namely the Greek a nd
Roman, were studied. The ways in which the several cultures
advanced during their periods of rebirth is t he focus of the
e ntire passage.

21. (D) Sentences 2 and 3 in paragraph l state that the foods made
from gelatin are jellylike (which would be easy to chew) and high
in protein, so we can assume that gelatin would be beneficial for
elderly and ill people.
626 ANSWERS AND EXPLANATIONS FO R PRACTICE TEST b

22. (A) If you don ' t know the meaning of " fluid. " this question
could be difficult. T he two sentences following ·'fluid" tell you
that it is concentrated, chilled, ~ Heed, :ind finally dried and
ground. r\ " fluid" 1s closest in meaning to ·'liquid." Whe n the
' 'liquid" in this passage is concentrated a nd chilled, ir becomes
a firmer subst ance.

23. ( B) Paragraph 2. sentence :.. says that the grease must fi rst be
eliminated, so it probably dot:5 not a id in producing gelatin . The
reading passage specifically c-intradicts c hoice (A ) bec au ~e it
says in t he firs t sentence of pa1agrap h 2 t hat processi ng ge latin
made from b'.)r.es ·'varies slightly from 1har of gelatin made
from skin." T he reading indicat."!S th a t a much more detailed
procedure of oroducing gelatin 's req uired than what is de-
scribed in choice (C). Answer chc ice (D) is incorrect because
t he reading indicates gelatin is ··dned a nd ground. " Only after
the grinding would gelatin be in powder fo rm.

24. (A) The whole reading deals with t he p rocess of making gelatin .
C hoice (B) is too gen eral, a nd t he o ther choices suggest only
details o f the passage.

25. (A) This is the only answer choice not listed in the passage.

26. (C) The passage states that gelatin is beneficial to the consumer
because it is high in protein.

27. (B) Substitute the answer choices for ·'ground" in t he sentence.


One would not "putrify" (C) o r ''dirty" (D ) a commercial or
food product. so e limina te those choices. The gelatin has
already been chilled. or ·•refrigerated," c hoice (A), in the
'1rcvious sentence. That leaves· " pulverized," choice ( B). To
··pulverize" is to ·'grmd."

28. ( B) The last sentence says that gelatin is tasteless, so choice (A)
is incorrect. Gelatin is not easy to make. so choice (C ) is not
true. One can infer that gelatin does melt a t high temperatures
because it has to b e refrigerated to be sliced and ground, so
choice (D ) is untrue.
SECTION J: READING COMPREHENSION 627

29. (B) The -;entence states, " In the processing of gelatin made
from bones (which varies slightly from that of gelatin made from
skin), the grease must first be e liminated." " That" refers to the
"'processing" at the beginning of the sentence.

30. (C) Paragra ph 1. sentence 2 , says that the technology "allows


scientist!> to introduce genetic material (or genes) from one
organism into another." The key word in this question is
" primarily." Choice (D) is a small part of the technology.
Choice IA) 1s a result o i the tech nology. Choice (B) is the
function of the foreign gen e a nd also involves only a part of the
technolo g:,·.

11 (A\ ~ub-.titutc t he .rnswer c hoices fo r '"profound" in the


~e ntence . From the context. you can infer that recombinant
DNA technology is incredible. The development is not ·'bor-
ing, ·· choice (B), or "'secret. " c hoice (D) . The techn ology may
be ''dangerous." choice (C), but that is not the focus of this
'icntence. ·' Profound " means "'significa nt" in this use.

32. (C) In the sentence. ''isolation" applies to a piece of DNA fro m


an o rganism under ~ tudy. "'D estruction" (A ) of the ONA
doesn' t make o;ense. nor does the "'study" of what is already
bei ng studied. cho ice (D). " Duplica tion" is addressed in the
last half of t he sentence by "artifici ally syn thesized." "Isola-
tion" is closest in meaning to "segregation."

33. (B) Paragraph 1. sentence 5, tells us that Escherichia coli is a


bacterium into which the recom binant molecule can be intro-
duced. It is not itself p roduced by DNA technology.

34. (B) The word "artificially" means " not naturally," or "syntheti-
cally."

35. (D) In t he sentence, a piece of DNA is being " ligated" to a


fragment of bacterial DNA. The only answer choice that makes
sense is (D).
628 ANSWERS AND EXPLANATIONS FOR PRACTICE TEST 6

36. (A ) Sentence 6 states that " the foreign gene will not only
replicate in the bacteria, but also express itself." Thus choice
(A) is the o ne choice that is not true. Choices (B), (C), and (D)
are specifically mentioned in the passage.

37. (A ) The " fragment" in the sentence is some bacterial DNA.


Choices (B) and (C) do not make sense, and it's highly unlikely
that scientists would be dealing with a " large piece" (D) of
DNA. "Particle" is the best answer cho ice.

38. (C) The sentence states t hat bacterial DNA has the "capacity"
to replicate itself independently. DNA doesn 't have " h<'r-
mones," choice (A), " technology," choice (B), or " space,"
choice (D). It does have the "ability" to replicate, however, so
that is the best choice.

39. (B) The last sentence of paragraph 1 states that the expression
of a gene requires the processes of transcription and transla-
tion. Choice (D) is not true because, while the reading states
that Escherichia coli may be produced in large amounts in
synthetic media, it does not say t hat it requires synthetic media.

40. (A ) Sentence 4 says, "This piece of DNA is then ligated to a


fragment of bacterial DNA which has the capacity to replicate
itself independently." Sentence 5 continues, "The recombinant
molecule thus produced . ... " This means that two different
molecules are ligated (joined) to produce " a recombinant
molecule." Choice (B) is not true, since. although several
technologies are combined, the reading does not say that the
technologies are recombined. Choice (C) is not true because
reco mbination of molecules is at the level of DNA, and not at
the level of their products, the proteins. Choice (D) is not true
because Escherichia coli is used to obtain expression of the
recombinant molecule. but it is a "common intestinal bacte-
rium," and not a recombinant.

41. (A) The o nly answer choice that makes sense for the synonym
of "replicate" is " reproduce."
SECTION 3: READING COMPREHENSION 629

42. (D) Mars is 55 percent the size of Earth; therefore, it is smaller.

43. (B) The first sentence of paragraph 2 tells us that the canals
were thought to have been discovered by ~chiaparelli.

~- (B) " Supposedly" modifies Schiaparelli's discovery of man-


made canals o n Mars. The next sentence states that the
man-made canal theory was proved to be a myth. The context
casts doubt o n the discovery, so " presumably" is closest in
meaning to "supposedly."

45. (B) If Mars is "commonly called" the Red Planet, that is how it
is " nicknamed."

46. (C) A " myth" is e ither something untrue or a culture's wo1ld


view. In this use, " myth" is closest in meaning to " legend,"
which is a popular story or theory that is unverified.

47. (D) This is the o nly choice that is nor true. The last paragraph
states that some polar ice a nd permafrost were found, indicat-
ing that a t one time there were significant quantities. Now,
however, o nly traces are left, not large quantities.

48. (A) Most of the passage is spent telling the reader what Viking I
discovered when it landed on Mars and how the data refutes
some of t he beliefs people had about Mars. The other th ree
answer choices are incorrect.

49. (D) The sentence states that weather changes are being
"monitored," so the word nearest in meaning to that is
"observed."

50. (B) The fact that the volcanoes o n Mars a re believed to be


''dormant," though some show signs of activity, should lead you
to choose "inactive" as the synonym for "dormant" and the
condition of most of the volcanoes.

You might also like